Enarm 8a Parte

91
ENARM by JD-MD ENARM 2007 by JD-MD 8ª PARTE 701.- Para la activación de de la vitamina D por el riñón se requiere la acción de una: Peptidasa Hidroxilasa Desmolasa Isomerasa Anhidrasa La vitamina D3 (cholecalciferol), se produce en la piel de los mamíferos a partir del 7 dihidrocolesterol por acción de la luz solar. En el hígado, la vitamina D3 es convertida en 25 hidroxicolecalciferol (calcidiol, 25-OHD3). El 25 hidroxicolecalciferol es convertido, en los túbulos proximales de los riñones, al metabolito más activo 1,25 dihidrocolecalciferol, que también se denomina calcitriol o 1,25-(OH)2D3. La enzima implicada es la 1-alfa hidroxilasa mitócondrica. 702.- Caso clínico seriado. Una mujer de 22 años acude a consulta y refiere perdida de 10 kg de peso en dos meses, dolor en la cara anterior del cuello, caída del cabello y exoltalmos. Primer enunciado: El diagnóstico más probable es: a) Bocio tóxico difuso b) Tiroiditis c) Hipotiroidismo d) Cáncer de tiroides e) Bocio nodular 703.- Segundo enunciado: Lo más probable es que en los exámenes de laboratorio de esta paciente se encuentre: a) TSH elevada, T4 elevada y T3 elevada b) TSH baja, T4 normal y T3 normal c) TSH elevada, T4 normal y T3 normal d) TSH baja, T4 elevada y T3 elevada e) TSH baja, T4 baja y T3 baja El caso clínico nos muestra datos clásicos de hipertiroidismo causada probablemente por una tirotoxicosis, el dato clave del diagnóstico es el exoftalmos que es causado por la enfermedad de Graves, y no por otras causas de hipertiroidismo. 704.- Un hombre de 65 años, hipertenso y fumador crónico presenta dolor torácico terebrante de aparición súbita, irradiado a la cara posterior del tórax. El diagnóstico más probable es: MX-Biomedical Research Group / JD-MD Bioinformatics Labs JD-MD Medicine and Technology 1

Transcript of Enarm 8a Parte

Page 1: Enarm 8a Parte

ENARM by JD-MD

ENARM 2007 by JD-MD 8ª PARTE

701.- Para la activación de de la vitamina D por el riñón se requiere la acción de una:

PeptidasaHidroxilasaDesmolasaIsomerasaAnhidrasa

La vitamina D3 (cholecalciferol), se produce en la piel de los mamíferos a partir del 7 dihidrocolesterol por acción de la luz solar. En el hígado, la vitamina D3 es convertida en 25 hidroxicolecalciferol (calcidiol, 25-OHD3). El 25 hidroxicolecalciferol es convertido, en los túbulos proximales de los riñones, al metabolito más activo 1,25 dihidrocolecalciferol, que también se denomina calcitriol o 1,25-(OH)2D3. La enzima implicada es la 1-alfa hidroxilasa mitócondrica.

702.- Caso clínico seriado. Una mujer de 22 años acude a consulta y refiere perdida de 10 kg de peso en dos meses, dolor en la cara anterior del cuello, caída del cabello y exoltalmos. Primer enunciado: El diagnóstico más probable es:

a) Bocio tóxico difusob) Tiroiditis c) Hipotiroidismod) Cáncer de tiroidese) Bocio nodular

703.- Segundo enunciado: Lo más probable es que en los exámenes de laboratorio de esta paciente se encuentre:

a) TSH elevada, T4 elevada y T3 elevadab) TSH baja, T4 normal y T3 normalc) TSH elevada, T4 normal y T3 normald) TSH baja, T4 elevada y T3 elevada e) TSH baja, T4 baja y T3 baja

El caso clínico nos muestra datos clásicos de hipertiroidismo causada probablemente por una tirotoxicosis, el dato clave del diagnóstico es el exoftalmos que es causado por la enfermedad de Graves, y no por otras causas de hipertiroidismo.

704.- Un hombre de 65 años, hipertenso y fumador crónico presenta dolor torácico terebrante de aparición súbita, irradiado a la cara posterior del tórax. El diagnóstico más probable es:

a) Tromboembolia pulmonarb) Ruptura del músculo papilarc) Neumotórax espontáneod) Aneurisma disecante de la aorta e) Infarto agudo del miocardio

The large majority of abdominal and thoracic aortic aneurysms are asymptomatic when they are discovered incidentally on a routine physical examination or imaging study. Patients with thoracic aortic aneurysms may experience chest pain or, less often, back pain. Vascular complications include aortic insufficiency (sometimes with secondary heart failure), hemoptysis, and thromboembolism. An enlarging aneurysm may produce local mass effects due to compression of adjacent mediastinal structures, producing symptoms such as coughing, wheezing, dyspnea, hoarseness, recurrent pneumonia, or dysphagia.

705.- Caso clínico seriado. Una mujer de 23 años, con embarazo de 35 semanas, asiste a consulta prenatal programada y refiere disnea de medianos esfuerzos, astenia y edema bimaleolar. Su presión arterial es de 100/60 mmHg y la frecuencia cardíaca, de 84 latidos por minuto. Se ausculta soplo sistólico en el mesocardio, sin irradiaciones, que deja de escucharse al cambiar de posición. La evaluación fetal es normal. Primer enunciado: el diagnóstico clínico más probable es:

MX-Biomedical Research Group / JD-MD Bioinformatics LabsJD-MD Medicine and Technology

1

Page 2: Enarm 8a Parte

ENARM by JD-MD

a) Insuficiencia cardíacab) Hipertensión pulmonarc) Embarazo normal d) Síndrome neurovegetativoe) Síndrome de hipotensión supina

706.- Segundo enunciado: la conducta mas apropiada en este momento consiste en:

a) Administrar vasodilatadoresb) Proporcionar medidas para el cuidado de un embarazo normal c) Administrar anticolinergicosd) Administrar expansores del volumen plasmáticoe) Administrar digitalicos

MATERNAL WELL-BEING AS A SIGN OF FETAL WELL-BEINGIn modern obstetric practice, fetal well-being has been determined mainly by direct monitoring and testing, but it is important not to overlook the status of the mother when determining fetal well-being. Maternal health is obviously crucial to fetal development and must be continuously assessed during pregnancy.Maternal Height & Weight: Maternal height and prepregnancy weight along with the rate and amount of weight gain during pregnancy are important in fetal development. Women who are underweight or of short stature tend to have smaller babies, and are at risk for low birthweight and preterm delivery. A teenage mother is compromised if her diet is not adequate to meet her own growth requirements as well as those of her fetus. In such circumstances, women less than 157 cm (5 ft) tall and especially those weighing less than 45 kg (100 lb) should be encouraged to gain at least a minimum of 11–12 kg (25 lb), if not more. Inadequate progressive weight gain may reflect nutritional deficit, maternal illness, or a hormonal milieu that does not promote proper volume expansion and anabolic state. Often, this is associated with poor fundal growth and a small fetus and placenta, suggesting fetal growth retardation. Weight gain and fundal height should be closely monitored during pregnancy.Blood Pressure: Blood pressure levels may provide a clue to subtle circulatory compromise. Normally, the mean arterial pressure drops somewhat from prepregnancy or early pregnancy values during the middle trimester. It is important to note this decline so that it does not mask a subsequent rise in blood pressure that may signal the onset of hypertension. In the third trimester, blood pressure recordings taken in the supine position may be higher than those taken in the recumbent position; this may also indicate the onset of hypertension. Normal patients may have a significant drop in blood pressure in the supine position (supine hypotensive syndrome). This is corrected when the patient is in the left lateral position.Fundal Height: Fundal height should be measured and recorded at each visit after 20 weeks. Measurements should be made with a centimeter tape (McDonald's technique) from the pubic symphysis to the top of the uterine mass over the curvilinear abdominal surface. Progress is especially important in the third trimester, when fetal growth retardation is most easily determined.Fetal Heart Tones: FHTs can usually be heard by 10–12 postmenstrual weeks using a hand-held Doppler device. This may be helpful when gestational age is in doubt or in the presence of threatened abortion or other abnormal observations in the late first trimester. Attention should be paid both to rate and rhythm and to any accelerations, decelerations, or irregularities. Significant abnormalities may be further assessed by ultrasonography, fetal echocardiography, or electronic fetal heart rate monitoring, depending on gestational age. Term gestation can be assumed 18–20 weeks after FHTs have first been heard with the standard unamplified obstetric stethoscope. However, prudence demands use of other clinical landmarks in determining gestational age.Edema: At each prenatal visit, abnormal or potentially abnormal findings should be noted, and a careful record should be made of any unusual events that have occurred since the last visit. Transient episodes of general edema or swelling should be noted. Lower extremity edema in late pregnancy is a natural consequence of hydrostatic compromise of lower body circulation. Edema of the upper body (eg, face and hands), especially in association with relative or absolute increases in blood pressure, may be the first sign of preeclampsia. Subtle changes may precede the more obvious picture (eg, finger rings may become too tight; this is a convenient index of early difficulties). A moderate rise in blood pressure without excessive fluid retention may suggest a predisposition to chronic hypertension.Fetal Size & Position: Manual assessment of fetal size and position is always indicated after about 26 weeks. The fetus may assume a number of positions before late gestation, but persistence of an abnormal lie into late pregnancy suggests abnormal placentation, uterine anomalies, or other problems that should certainly be investigated further. If an abnormal lie persists, consider external version after 37 weeks. Suspected abnormal fetal size should also be investigated, and failure to palpate fetal parts easily may suggest polyhydramnios (also confirmed by uterine measurements that deviate from expectations).

MX-Biomedical Research Group / JD-MD Bioinformatics LabsJD-MD Medicine and Technology

2

Page 3: Enarm 8a Parte

ENARM by JD-MD

707.- Una paciente de 28 años, con antecedente de infección pélvica presenta súbitamente dolor abdominal, taquicardia y temperatura de 38º C en la exploración física se encuentra aumento de volumen del anexo derecho. El diagnóstico clínico más probable es:

a) Piosalpinxb) Embarazo ectópicoc) Absceso tubo-ovarico d) Quiste torcido del ovarioe) Quiste paratubario

Las secuelas de la enfermedad pélvica inflamatoria incluyen embarazo ectópico, infecundidad, dolor pélvico crónico, hidrosálpinx, absceso tuboovárico y dispareunia crónica. La rotura del absceso tuboovárico puede causar sepsis.

708.- La hormona responsable de la actividad del complejo mioeléctrico migratorio intestinal es la:

a) Motilina b) Gastrinac) Colecistocininad) Secretinae) Bradicidina

La motilina origina contracciones del músculo liso intestinal, y parece ser un regulador de la motilidad interdigestiva preparando al intestino para la siguiente comida.

709.- La limitación de la actividad de las personas mayores de 65 años se debe más frecuentemente a:

Alteraciones visualesEnfermedades cardíacasProblemas auditivosEnfermedades reumáticasEnfermedades cerebro-vasculares

La limitación debida a enfermedades degenerativas articulares (clasificadas como enfermedades reumáticas) causa el mayor impacto en la limitación de la actividad en los adultos mayores. Las alteraciones auditivas también son un problema importante de limitación de la actividad, 30% en mayores de 65 años y más del 50% en mayores de 85 años la sufren.

710.- La diabetes, la hipertensión arterial y la obesidad son factores de riesgo relacionados con el cáncer:

Cuello uterinoOvarioMamaEndometrioMiometrio

Endometrial carcinoma is the most common malignancy of the female genital tract in the United States. For 2000, 36,100 new cases and 6,500 deaths were anticipated. It is predominantly a disease of affluent, obese, postmenopausal women of low parity, although an increasing proportion of younger patients with endometrial cancer has been reported. In the United States, black women have approximately a 40% lower risk for development of cancer of the corpus uteri, but approximately a 54% greater risk of dying of the disease, mainly because of late diagnosis. Over the last few decades, age-standardized incidence rates have risen in most countries and in urban populations. Developing countries and Japan have incidence rates four to five times lower than Western industrialized nations, with the lowest rates being in India and south Asia. Since the mid-1980s, careful surgical staging has more accurately defined patterns of spread and has allowed more individualization of treatment, particularly with respect to adjuvant radiation. Any factor that increases exposure to unopposed estrogen (e.g., hormone replacement therapy, obesity, anovulatory cycles, estrogen-secreting tumors) increases the risk of endometrial cancer, whereas factors that decrease exposure to estrogens or increase progesterone levels (e.g., oral contraceptives or smoking) tend to be protective. The impact of these factors differs in various populations, and in northern Italy, established risk factors account for only approximately 50% of the cases.

MX-Biomedical Research Group / JD-MD Bioinformatics LabsJD-MD Medicine and Technology

3

Page 4: Enarm 8a Parte

ENARM by JD-MD

711.- Si la prueba de Schilling es normal en un paciente que padece anemia megaloblástica, se debe buscar una alteración en el:

EstomagoDuodenoYeyunoIleonColon

A normal Schilling test in a patient with documented cobalamin deficiency may indicate poor absorption of the vitamin when mixed with food. This can be established by repeating the Schilling test with radioactive cobalamin scrambled with an egg.

712.- Un lactante de tres meses previamente sano inicio su cuadro clínico hace 5 días con rinorrea hialina y tos seca ocasional. Dos días después se exacerbó la tos y presentó paroxismos severos acompañados de cianosis y estridor al final del acceso. El diagnóstico más probable es:

LaringotraqueitisNeumonía intersticial por Chlamydia trachomatisTos ferinaBronconeumoniaSíndrome coqueluchoide

Pertussis, or whooping cough, is an infection of the respiratory tract caused by Bordetella pertussis. Occasionally, a similar clinical syndrome is caused by Bordetella parapertussis, the adenoviruses, or Chlamydia. Following inhalation, B. pertussis organisms attach to the epithelial cells of the respiratory tract. Multiplication of the bacteria leads to infiltration of the mucosa with polymorphonuclear leukocytes and lymphocytes. Inflammatory debris in the lumen of the bronchi and peribronchial lymphoid hyperplasia obstruct the smaller airways, causing atelectasis. Although pertussis can be divided into three stages for discussion, a clinically distinct syndrome does not evolve until the disease has progressed to the second stage. Initially, the symptoms mimic a viral URI. This first stage (catarrhal), characterized by a mild cough, conjunctivitis, and coryza, lasts for 1 to 2 weeks. An increasingly severe cough heralds the onset of the second stage (paroxysmal), which continues for 2 to 4 weeks. After a prolonged spasm of coughing, the sudden inflow of air produces the characteristic whoop. Vomiting often occurs after such an episode. When not coughing, the child has a remarkably normal physical examination, except for an occasional subconjunctival hemorrhage. During the third stage (convalescent), the intensity of the cough wanes. The WBC count in children usually reaches a level of 20,000 to 50,000/mm3 with a marked lymphocytosis, but such changes are not often seen in infants less than 3 to 6 months old. Although a chest radiograph occasionally shows the characteristic “shaggy” right heart border, more often the lung fields appear clear. B. pertussis can be identified by fluorescent antibody staining of mucus obtained from the nasopharynx or, less commonly, recovered by culture of this material. The fatality rate for pertussis is approximately 1% for patients in the first month of life and 0.3% for those between age 2 and 12 months. Complications often occur during a bout of pertussis. The most immediately life-threatening complication is complete obstruction of the airway by a mucous plug, leading to respiratory arrest. Although secondary bacterial pneumonia has a more insidious onset, it occurs in 25% of children with pertussis and accounts for 90% of the fatalities from pertussis. Seizures are seen in 3% of patients, and encephalitis in 1%. Sudden increases in intrathoracic pressure can cause intracranial hemorrhages, rupture of the diaphragm, and rectal prolapse. Except for occasional situations in which fluorescent antibody testing is immediately available, the diagnosis of pertussis rests on clinical grounds. Children with an unmistakable paroxysmal cough followed by a whoop should be assumed to have the disease. When the clinical picture is unclear, a WBC count and chest radiograph may be useful. The radiograph helps eliminate other causes of a severe cough (e.g., foreign body, bacterial pneumonia, cystic fibrosis, tuberculosis), and the WBC count provides confirmatory evidence if a leukocytosis with marked lymphocytosis is found. Because of the grave risk of complications, all children less than 6 months of age diagnosed firmly as having pertussis should be observed in the hospital. Older children who show signs of respiratory compromise, such as cyanosis during paroxysms of coughing, or who develop complications also require admission. Treatment includes erythromycin (40 mg/kg per day for 14 days), maintenance of adequate hydration, and a level of respiratory support appropriate to the severity of the disease. Clarithromycin and azithromycin are alternative choices. Household and other close contacts require chemoprophylaxis with erythromycin (40 mg/kg per day for 14 days). Children younger than 7 years who are unimmunized or have received fewer than four doses of pertussis vaccine should have their pertussis immunization initiated or continued as soon as possible after exposure. Children who are fully immunized for age but have received only three doses require a fourth dose. Those who have had four doses need a booster unless the last dose has been within 3 years or they are more than 6 years old. DTPa is preferred.

MX-Biomedical Research Group / JD-MD Bioinformatics LabsJD-MD Medicine and Technology

4

Page 5: Enarm 8a Parte

ENARM by JD-MD

713.- Caso clínico seriado. Una niña de 4 años de edad presenta aproximadamente veinte pápulas en el tronco, la cara y las extremidades. Son translucidas; tienen forma de domo y umbilicación central. La madre refiere que la paciente no ha tenido fiebre, Primera pregunta: El diagnóstico clínico más probable es:

VaricelaMolusco contagiosoInfección por enterovirusEscabiosisPrurigo por insectos

714.- Segunda pregunta. El agente etiológico más probable es:

a) Virus cocksakie Ab) Varicela zoterc) Poxvirusd) Sarcoptes scabeie) Diptero sp.

El molusco contagioso es una enfermedad de la piel y de las mucosas, producida por un poxvirus. Se caracteriza por la presencia de pápulas blancas dispersas. No se afectan las palmas de las manos ni las plantas de los pies. La enfermedad es más frecuente en niños y adultos con una alteración de la respuesta inmune. La transmisión interpersonal se realiza por contacto directo o indirecto.

715.- El factor qué más contribuye al desarrollo de la osteodistrofia renal en pacientes con insuficiencia renal crónica es:

a) Producción alterada de 1,25-dihidroxivitamina D b) Hipocalcemia c) Hipofosfatemiad) Perdida de vitamina De) Aumento de la actividad osteoclástica

Renal osteodystrophy is characterized by secondary hyperparathyroidism, which is due to hyperphosphatemia, hypocalcemia, marked parathyroid hypertrophy, and bony resistance to the action of parathormone; by inadequate formation of 1,25-dihydroxyvitamin D in the kidney resulting in osteomalacia in adults and rickets in children; and for as yet obscure reasons, by areas of osteosclerosis. Tertiary hyperparathyroidism is said to exist when high parathormone levels persist despite normal or high levels of serum calcium. This condition is secondary to the marked increase in parathyroid mass with abnormal and inadequate suppression of parathormone secretion. Metabolic acidosis also contributes to the bone disease by titration of protons for calcium in bone matrix. High parathormone levels and high cytosol calcium concentrations probably contribute to uremic encephalopathy, myocyte dysfunction, and an impaired bone marrow response to erythropoietin. Severe syndromes termed calciphylaxis include metastatic calcification in soft tissues and small blood vessels and ischemic necrosis of skin and muscle. In such circumstances, partial parathyroidectomy--removal of 3½ glands--may be required, but secondary hyperparathyroidism is best prevented. Adynamic renal bone disease, which is associated with much-diminished bone turnover, is now being seen and requires bone biopsy for diagnosis. It may reflect skeletal resistance to the action of parathormone. Other joint diseases include secondary gout and pseudogout, which may be associated with chondrocalcinosis. In advanced renal failure a variety of factors have been identified as direct stimulators of parathyroid hormone (PTH) secretion, including hypocalcemia, low levels of circulating calcitriol (the active vitamin D metabolite), and more recently, hyperphosphatemia. However, most patients with mild chronic renal failure exhibit increased serum PTH levels without alterations in serum levels of calcium, phosphorus, and calcitriol. With more advanced nephron loss, the phosphate load of the remaining functioning nephrons progressively increases. This increased load results in inhibition of C1-alpha-hydroxylase, the enzyme responsible for the conversion of 25-hydroxyvitamin D to its active metabolite 1,25-dihydroxyvitamin D (calcitriol). Calcitriol deficiency in turn further decreases intestinal calcium absorption and thus results in hypocalcemia. Calcitriol deficiency in advanced renal failure is associated with a decreased number of vitamin D receptors, in particular, receptors in parathyroid glands. Because calcitriol has been shown to suppress the expression of pre-pro-PTH mRNA, lower circulating calcitriol levels together with a low number of vitamin D receptors in patients with ESRD result in stimulation of both synthesis and secretion of PTH. Low blood ionized calcium levels rapidly stimulate PTH secretion, whereas high calcium concentrations suppress it. The relationship between ionized calcium and PTH follows a sigmoidal pattern. The action of calcium on parathyroid gland cells is associated with modulation of intracellular cyclic adenosine monophosphate (cAMP). The short-term stimulation induced by low calcium is due to release of stored pre-formed hormone and an increase in the number of

MX-Biomedical Research Group / JD-MD Bioinformatics LabsJD-MD Medicine and Technology

5

Page 6: Enarm 8a Parte

ENARM by JD-MD

cells that secrete PTH. More prolonged hypocalcemia induces changes in intracellular PTH degradation with reutilization of degraded hormone and mobilization of the secondary storage pool. Within days or weeks of the onset of hypocalcemia, pre-pro-PTH mRNA expression is stimulated. This effect is exerted through a recently described negative calcium response element located in the upstream flanking region of the gene for PTH. Calcium exerts its effects on parathyroid gland cells through a recently isolated G protein-coupled calcium-sensing receptor located on the cell membrane. Expression of the calcium receptor has been shown to be suppressed by calcitriol deficiency and stimulated by calcitriol administration, thus suggesting an additional regulatory mechanism of the active vitamin D metabolite on PTH production. The decreased number of calcium-sensing receptors with low circulating calcitriol may, at least in part, explain the relative insensitivity of parathyroid gland cells to calcium in patients undergoing dialysis (higher set point). When the glomerular filtration rate reaches levels of less than 25% of normal, the serum phosphorus content rises. At this level of reduced renal function, the ability of the remaining nephrons to increase phosphate excretion is exhausted. Increased serum phosphorus levels further decrease serum calcium through physicochemical binding and suppress C1-alpha-hydroxylase activity, which results in further lowering of the circulating levels of calcitriol. Moreover, a direct stimulatory effect of phosphorus on parathyroid gland cells, independent of calcium and calcitriol, has recently been observed in patients with ESRD. The mechanism of the direct action of phosphorus on PTH secretion has not been fully elucidated. All the mechanisms described above result in increased production of PTH and increased parathyroid gland mass. The size of the parathyroid glands progressively increases with time in dialyzed patients and parallels serum PTH levels. This increase in size is mainly due to diffuse cellular hyperplasia. Monoclonal cell growth may also develop and result in the formation of tumor-like nodules that have less or no vitamin D and calcium-sensing receptors and that promote parathyroid gland resistance to calcitriol and calcium.

716.- En pacientes qué padecen trombosis venosa profunda, el procedimiento más sensible para observar el trombo y analizar el flujo sanguíneo es:

a) Venografiab) Ultrasonido doppler c) Gammagrama venosod) Pletismografíae) Resonancia magnética

Objective tests used in the diagnosis of venous thrombosis include B-mode ultrasonography (commonly with color flow Doppler), impedance plethysmography (IPG), and ascending venography. The only laboratory test of proven value in the diagnosis and management of DVT is the D-dimer test.Real-time B-mode ultrasound, preferably with Doppler assessment, has become the standard technique for the evaluation of patients with clinically suspected DVT. Prospective studies have shown that the single criterion of vein compressibility is highly sensitive and specific for the detection of proximal DVT (sensitivity and specificity, both >95%). Other ultrasound criteria are insensitive, non-specific, or both. Doppler ultrasonography is still not sufficiently sensitive for the detection of isolated calf vein thrombosis; serial testing is required to detect proximal extension. Serial ultrasound, based on the now-confirmed concept that calf vein thrombi are clinically important only when they extend into the proximal veins and are reliably detected by ultrasound, is a useful clinical approach. A positive result is highly predictive of acute proximal DVT and warrants anticoagulation, whereas anticoagulant therapy can be safely withheld in symptomatic patients who have negative results by serial ultrasound. IPG is sensitive and specific for proximal DVT in symptomatic patients, but it is insensitive for calf DVT; a normal result cannot exclude this diagnosis. In patients with clinically suspected DVT, positive IPG results guide therapeutic decisions in the absence of clinical conditions known to produce false-positive results.Although IPG has high sensitivity and specificity for the detection of symptomatic DVT, it lacks sensitivity for the detection of asymptomatic DVT in patients who have had surgery, such as total hip replacement, or in trauma patients. In such circumstances, the only reliable method for detecting DVT is bilateral ascending venography.False-positive IPG results may occur with disorders that interfere with arterial inflow or venous outflow, including severe heart failure, constrictive pericarditis, severe arterial insufficiency, hypotension, and external compression of veins. Most of these disorders are readily recognized clinically. The test cannot be performed on patients who are in plaster casts or who cannot be adequately positioned because of immobilization or pain.Doppler ultrasound and IPG are both highly sensitive and specific in the diagnosis of proximal DVT in symptomatic patients, but Doppler ultrasonography is more sensitive for detecting symptomatic calf vein thrombosis and more accurate for detecting proximal DVT in patients with increased central venous pressure or arterial insufficiency and DVT in patients whose leg is in a removable cast, who are in traction, or who have had distal leg amputation. However, both IPG and Doppler ultrasonography lack sensitivity and specificity for the detection of asymptomatic venous thrombosis in patients who have undergone surgery. Doppler ultrasonography (or even real-time B-mode ultrasonography without Doppler) has generally replaced IPG as the most popular non-invasive test for the detection of venous thrombosis.

MX-Biomedical Research Group / JD-MD Bioinformatics LabsJD-MD Medicine and Technology

6

Page 7: Enarm 8a Parte

ENARM by JD-MD

Venography, the gold standard objective method for the diagnosis of DVT, requires considerable experience to execute and interpret accurately. The most reliable criterion for DVT is an intraluminal filling defect that is constant in all films and is seen in numerous projections. Other abnormalities such as non-filling of a segment of the deep venous system or non-filling of the entire deep venous system above the knee may be caused by technical artifacts. Even in the best of circumstances, it may be impossible to cannulate a vein on the dorsum of the foot, thus making ascending venography impossible. Venography is also associated with pain in the foot while dye is being injected or 1 to 2 days after injection. The procedure may be complicated by superficial phlebitis and even DVT in 1 to 2% of patients with normal results on venography. Other less common complications include dye hypersensitivity or aggravation of renal insufficiency. The risks of venography must be carefully weighed against its benefits, and ultrasonography has largely replaced contrast venography in symptomatic patients because of its widespread availability. D-dimer, which can be measured rapidly by a number of enzyme-linked immunosorbent assays or less reliably by latex agglutination, has a high negative predictive value for excluding suspected VTE. The combination of a negative D-dimer, a non-diagnostic lung scan, and a low clinical probability of DVT or the combination of a normal D-dimer and normal ultrasound or IPG may sufficiently exclude DVT so that further diagnostic testing can be safely limited and anticoagulation avoided.However, the main limitation of D-dimer testing is that patients with suspected venous thrombosis frequently have significant co-morbid disease or are elderly; the majority of these patients have abnormal D-dimer assays. The findings in two clinical trials that a single repeat venous ultrasound at 1 week could safely exclude the diagnosis of DVT has made the use of clinical probabilities and D-dimer testing less relevant.For patients with suspected DVT, screening compression ultrasound should be the first test; patients who are positive should be treated. Patients with a negative screening ultrasound should undergo serial non-invasive leg testing one to three times (minimum of one test at 1 week). If a diagnosis is urgently required, venography may be used. In centers using a D-dimer assay of proven validity, patients with a negative ultrasound and low clinical probability require no further testing or treatment if the D-dimer test is negative. Because patients with a high or intermediate clinical probability still require serial non-invasive leg tests or venography despite a negative ultrasound and negative D-dimer, D-dimer assay adds little to the evaluation of such patients.The diagnosis of upper extremity DVT can be made by compression ultrasonography, with or without color flow Doppler imaging, or by venography. Many patients with suspected upper extremity venous thrombosis have negative objective studies that exclude the diagnosis. Pulmonary embolism frequently complicates upper extremity DVT and may be the initial manifestation; objective testing for pulmonary emboli is required if the diagnosis will alter management. In patients with superficial thrombophlebitis, objective testing should be performed whenever DVT is suspected, particularly in patients with extensive phlebitis

717.- El dato ultrasonografico más importante para precisar la conducta a seguir ante un embarazo prolongado es la:

a) Edad gestacionalb) Medición del diámetro biparietalc) Madurez placentariad) Cantidad de líquido amniótico e) Característica del líquido amniótico

Reduced amniotic fluid (i.e., oligohydramnios) is a frequent finding in prolonged pregnancies. It presents a problem because it can be a marker for fetal compromise and because it puts the fetus at risk for cord accidents. Oligohydramnios has been attributed to placental insufficiency causing the fetus to redistribute blood flow to protect the brain. This, in turn, is considered to cause renal hypoperfusion with reduced glomerular filtration and urine production leading to decreased amniotic fluid. However, a study of 57 women at gestations greater than 41 weeks in which the resistance index of the fetal middle cerebral, renal, and umbilical arteries was evaluated, found no evidence that oligohydramnios was associated with redistribution of blood flow. It is commonly accepted that amniotic fluid volume decreases as gestational age advances beyond term. In at least one study, this was not found to be the case. Among 511 women at 41 weeks or more, only 11.5% had oligohydramnios defined as a four-quadrant amniotic fluid index (AFI) of 5 or less. The average AFI at 41 weeks was 12.4 cm ± 4.2 cm. The best method to accurately estimate the amount of amniotic fluid is also a matter of debate. Commonly used ultrasound techniques include the four-quadrant AFI and the largest vertical pocket. In a study of 190 women at 40 weeks or more in which four ultrasound methodologies were studied for amniotic fluid volume estimation (the four-quadrant AFI, the largest vertical pocket, the largest transverse pocket, and the sums of all pockets), the only technique which demonstrated diagnostic value was use of the largest vertical pocket. A cutoff of 2.7 cm correlated well with favorable outcomes. The traditional cutoff of 2 cm in largest vertical diameter was also found to be an indicator of favorable outcome, but the authors thought that sensitivity could be increased by raising the cutoff to 2.7 cm.

718.- La prueba de tolerancia a las contracciones uterinas esta indicada en casos de:

MX-Biomedical Research Group / JD-MD Bioinformatics LabsJD-MD Medicine and Technology

7

Page 8: Enarm 8a Parte

ENARM by JD-MD

a) Amenaza de parto preterminob) Embarazo postermino c) Inserción baja de la placentad) Cesárea iterativae) Fase activa de trabajo de parto

It is used in prolonged pregnancy. The contraction stress test (CST) was the first test used for antepartum fetal monitoring. The underlying premise is that the fetus with uteroplacental insufficiency would experience late decelerations with contractions in response to hypoxia generated by decreased blood flow to the intervillous space. The non-stress- test (NST) is the first-line screening test at many medical centers. It is quickly and easily performed in an outpatient setting. The NST is based on the knowledge that fetal hypoxia interrupts the pathway between the fetal heart and an intact central nervous system (CNS). The fetus with an intact CNS will have heart rate accelerations with movement or stimulation. The CST has a high false-positive rate and the NST has a false-negative rate of 2.7 per 1,000 in a high-risk population.

719.- Desde el punto de vista histopatológico, la mayoría de los tumores primarios de la próstata corresponden a:

a) Adenocarcinoma de tipo acinar b) Adenocarcinoma de tipo ductalc) Adenocarcinoma de tipo mucinosod) Carcinoma transicionale) Carcinoma escamoso

In approximately 70% of cases, carcinoma of the prostate arises in the peripheral zone of the gland, classically in a posterior location, often rendering it palpable on rectal examination. Characteristically, on cross-section of the prostate, the neoplastic tissue is gritty and firm, but when embedded within the prostatic substance, it may be extremely difficult to visualize and be more readily apparent on palpation. Spread of prostate cancer occurs by direct local invasion and through the bloodstream and lymph. Local extension most commonly involves the seminal vesicles and the base of the urinary bladder, which may result in ureteral obstruction. Hematogenous spread occurs chiefly to the bones, particularly the axial skeleton, but some lesions spread widely to viscera. Massive visceral dissemination is an exception rather than the rule. The bony metastases may be osteolytic, but osteoblastic lesions are frequent and in men point strongly to prostatic cancer. The bones commonly involved, in descending order of frequency, are lumbar spine, proximal femur, pelvis, thoracic spine, and ribs. Lymphatic spread occurs initially to the obturator nodes followed by perivesical, hypogastric, iliac, presacral, and para-aortic nodes. Lymph node spread occurs frequently and often precedes spread to the bones. As we discuss later, metastases to the lymph nodes in apparently localized prostatic cancer have a significant impact on the prognosis.Histologically, most lesions are adenocarcinomas that produce well-defined, readily demonstrable gland patterns. The glands are lined by a single uniform layer of cuboidal or low columnar epithelium. The outer basal layer of cells typical of normal and hyperplastic glands is often absent. Occasionally the glands are somewhat larger, with a papillary or cribriform pattern. The cytoplasm of the tumor cells is unremarkable, but the nuclei are large and vacuolated and contain one or more large nucleoli. There is some variation in nuclear size and shape, but in general pleomorphism is not marked. Mitotic figures are extremely uncommon. When such well-differentiated tumors occur in sharply delimited rounded masses, they have to be distinguished from nodular hyperplasia. In general, malignant acini are smaller and closely spaced, "back-to-back" with little intervening stroma and are lined by a single layer of cells. Not all prostatic cancers, however, are well differentiated. In less differentiated lesions, the tumor cells tend to grow in cords, nests, or sheets. Stromal production may be scant or quite extensive in certain lesions, producing a scirrhous-like consistency to the neoplasm. The most reliable hallmarks of malignancy, especially in well-differentiated tumors, are clear evidence of invasion of the capsule with its lymphatic and vascular channels, perineurial invasion, or both. The perineurial spaces, which are involved in most cases, are not lined by endothelium, and they do not represent lymphatics, as formerly believed.In approximately 80% of cases, prostatic tissue removed for carcinoma also harbors presumptive precursor lesions, referred to as duct-acinar dysplasia, or simply prostatic intraepithelial neoplasia (PIN). These lesions consist of multiple, although sometimes single, foci of glands with intra-acinar proliferation of cells that demonstrate nuclear anaplasia. In contrast to frank cancer, however, there is no invasion, and the dysplastic cells are surrounded by a layer of basal cells and an intact basement membrane. Studies have revealed that many of the molecular changes seen in invasive cancers are also present in PIN. These include aneuploidy as well as loss of heterozygosity at several loci. At certain loci (e.g., 8p12), there is loss of heterozygosity in 64% of foci of PIN and 91% of cancers, compared with 0% in benign tissues. Such data strongly support the argument that PIN is an intermediate lesion between normal and frankly malignant tissue.

MX-Biomedical Research Group / JD-MD Bioinformatics LabsJD-MD Medicine and Technology

8

Page 9: Enarm 8a Parte

ENARM by JD-MD

In keeping with this notion, follow-up studies reveal that in about one third of cases, PIN progresses to invasive cancer within a period of 10 years, presumably after additional genetic changes have accumulated.

720.- En los pacientes diabéticos, la estenosis arterial por ateroesclerosis acelerada se presenta más frecuentemente en las arterias:

a) Popliteasb) Carotideasc) Femoralesd) Renalese) Coronarias

Dada la incidencia de eventos cardiacos en los pacientes diabéticos, la respuesta correcta podría ser la opción E

721.- Un paciente de 32 años acude a consulta por padecer hipertensión arterial. Al examen físico se le encuentra un soplo en la región interescapular y disminución de los pulsos en los miembros pélvicos. La radiografía de tórax muestra indentación del borde inferior del tercer al noveno arco costal. El diagnóstico más probable es:

a) Insuficiencia aórticab) Feocromocitomac) Coartación aórtica d) Arteritis de Takayasue) Estenosis aórtica

Aortic coarctation typically occurs just distal to the left subclavian artery at the site of the aortic ductal attachment or its residual ligamentum arteriosum. Less commonly the coarctation ridge lies proximal to the left subclavian. A bicuspid aortic valve is the most common coexisting anomaly, but VSDs and PDAs are also seen. "Pseudocoarctation" refers to buckling or kinking of the aortic arch without the presence of a significant gradient. The most common complications of aortic coarctation are systemic hypertension and secondary left ventricular hypertrophy with heart failure. Systemic hypertension is caused by decreased vascular compliance in the proximal aorta and activation of the renin-angiotensin system in response to renal artery hypoperfusion below the obstruction. Left ventricular hypertrophy occurs in response to chronic pressure overload. Congestive heart failure occurs most commonly in infants and then after 40 years of age. The high pressure proximal to the obstruction stimulates the growth of collateral vessels from the internal mammary, scapular, and superior intercostal arteries to the intercostals of the descending aorta. Collateral circulation increases with age and contributes to perfusion of the lower extremities and the spinal cord. This mechanism, although adaptive in a patient who has not undergone surgery, accounts for significant morbidity during surgery when the motor impairment results from inadequate protection of spinal perfusion. Aneurysms occur most notably in the ascending aorta and in the circle of Willis. Premature coronary disease is thought to be related to the resulting hypertension. Complications, including bacterial endarteritis at the coarctation site or more commonly endocarditis at the site of a bicuspid aortic valve, cerebrovascular complications, myocardial infarction, heart failure, and aortic dissection, occur in 2 to 6% of patients, more frequently in those with advancing age who have not undergone surgery. Young adults may be asymptomatic with incidental systemic hypertension and decreased lower extremity pulses. Coarctation should always be considered in adolescents and young adult males with unexplained upper extremity hypertension. The pressure differential can cause epistaxis, headaches, leg fatigue, or claudication. Older patients have angina, symptoms of heart failure, and vascular complications. On physical examination, the lower half of the body is typically slightly less developed than the upper half. The hips are narrow and the legs are short, in contrast to broad shoulders and long arms. Blood pressure measurements should be obtained in each arm and one leg; an abnormal measurement is a less than 10 mm Hg increase in popliteal systolic blood pressure as compared with arm systolic blood pressure. The diastolic pressure should be the same in the upper and lower extremities. A pressure differential of more than 30 mm Hg between the right and the left arms is consistent with compromised flow in the left subclavian artery. Right brachial palpation characteristically reveals a strong or even bounding pulse as compared with a slowly rising or absent femoral, popliteal, or pedal pulse. Examination of the eyegrounds can reveal tortuous or corkscrew retinal arteries. Precordial palpation is consistent with left ventricular pressure overload. On auscultation, a systolic ejection sound reflecting the presence of a bicuspid aortic valve should be sought. The coarctation itself generates a systolic murmur heard posteriorly, in the mid-thoracic region, the length of which correlates with the severity of the coarctation. Over the anterior of the chest, systolic murmurs reflecting increased collateral flow can be heard in the infraclavicular areas and the sternal edge or in the axillae. In adult coarctation, the most common finding on the electrocardiogram is left ventricular hypertrophy. Chest radiographic findings are diagnostic. Location of the coarctation segment between the dilated left subclavian artery above and the leftward convexity of the descending aorta below results in the "3 sign". Bilateral rib notching as a result of dilation of the posterior intercostal

MX-Biomedical Research Group / JD-MD Bioinformatics LabsJD-MD Medicine and Technology

9

Page 10: Enarm 8a Parte

ENARM by JD-MD

arteries is seen on the posterior of the third to eighth ribs when the coarctation is located below the left subclavian. Unilateral rib notching sparing the left ribs is observed when the coarctation occurs proximal to the left subclavian artery. Transthoracic echocardiography documents the gradient in the descending aorta and determines the presence of left ventricular hypertrophy. MRI is the best modality for visualizing the anatomy of the descending aorta. Cardiac catheterization should measure pressures and assess collaterals when surgery is contemplated. Repair is considered in patients with gradients greater than 30 mm Hg on cardiac catheterization. Fifty per cent of patients repaired when older than 40 years have residual hypertension, whereas those who have undergone surgery between the ages of 1 and 5 years have a less than 10% prevalence of hypertension on long-term follow-up. Balloon angioplasty is the treatment of choice for focal recoarctation in previously operated patients.

722.- El tratamiento del síndrome metabólico se debe iniciar con el uso de:

a) Pioglitazona b) Glipicidac) Glibenclamidad) Acarbosae) Metformina

Las biguanidas y las tizolidinedionas son los medicamentos que además de ser sensibilizadores de la insulina y tener efectos directos sobre el mecanismo de resistencia a la insulina mejorando el control glucémico, también tienen efectos favorables potenciales en otros componentes del síndrome metabólico cardiovascular.

723.- Un factor qué reduce la velocidad del vaciamiento gástrico es:

a) Consistencia acuosa del quimob) Concentración de lípidos en el quimo c) Aumento de gastrina en el plasmad) Alcalinidad del bolo alimenticioe) Reducción de la liberación del factor intrinseco

Gastric motor activity involves two distinct patterns that are distinguished anatomically and functionally. The first pattern involves the smooth muscle of the proximal one-third of the stomach. Myocytes of this region demonstrate no spontaneous myoelectric activity and instead relax in response to the stretch produced by increasing gastric volumes. This receptive relaxation means that a meal can be accommodated with little increase in intragastric pressure. Relaxation is believed to occur via a vagally mediated reflex and permits the stomach to act as a reservoir for several hours after a meal. Contractile activity in the proximal stomach gradually increases, and food is propelled distally by the pressure gradient. The motility pattern of the distal stomach is quite different. Beginning at a pacemaker site on the greater curvature, a series of myoelectric complexes pass distally at a rate of three times per minute. Action potentials are sometimes superimposed on these spontaneous depolarizations, resulting in a peristaltic wave that propagates distally. Vagal activity, gastrin, and motilin all increase the frequency of the action potentials, and secretin, glucagon, and GIP seem to reduce it. The movement of the meal is influenced by each of the motility patterns. Initially food enters the stomach and is stored in its proximal portion, where it is gradually digested and diluted. As pressure increases proximally, the bolus enters the antrum, where it is propelled toward the pylorus by a peristaltic wave. The pylorus actually closes several seconds before the arrival of the wave, allowing only a small amount of liquid and suspended food particles to enter the duodenum, while the main mass of food is retropulsed back into the proximal stomach. This process, referred to as trituration, serves to grind the food into smaller pieces and mix it further with gastric secretions. The stomach is bounded proximally and distally by sphincters. The lower esophageal sphincter (LES) is a high- pressure zone of the distal esophagus just proximal to the cardia. It normally relaxes only in response to a peristaltic wave from above, allowing the passage of a meal and swallowed secretions into the proximal stomach. Between relaxations, the high pressure prevents the reflux of gastric contents into the esophagus. In contrast with the LES, which has no anatomic correlate, the pyloric sphincter is a thickening of smooth muscle that not only regulates gastric emptying but also prevents reflux of duodenal contents into the stomach. The consequence of duodenogastric reflux after surgical disruption of the sphincter, alkaline reflux gastritis, is discussed later, in the section Postgastrectomy Syndromes.

724.- En un paciente con síndrome nefrótico secundario a mieloma múltiple secundario a mieloma múltiple y que tiene albúmina sérica de 1.5 mg/dl, aumenta el riesgo de que se complique con:

a) Edema pulmonar cardiogénicob) Hipocolesterolemiac) Hipertensión arterial

MX-Biomedical Research Group / JD-MD Bioinformatics LabsJD-MD Medicine and Technology

10

Page 11: Enarm 8a Parte

ENARM by JD-MD

d) Hematuriae) Aparición de eventos trombóticos

La albúmina es la proteína plasmática más abundante y representa el 70-90% de la proteinuria detectada en el síndrome nefrótico. La albúmina filtrada es catabolizada en parte por el túbulo renal, cuya tasa catabólica aumenta, pudiendo representar hasta un 20% de la albúmina filtrada en el síndrome nefrótico. Para compensar las pérdidas, el hígado aumenta la tasa de síntesis de albúmina hasta en un 300% por mecanismos de transcripción. Este incremento se correlaciona con la albuminuria, pero no con la presión oncótica del plasma o con la concentración sérica de albúmina, y se abole si la ingesta proteica está disminuida, lo que explica por qué las dietas hipoproteicas disminuyen la proteinuria, pero no aumentan la concentración de albúmina en el plasma. La hipoalbuminemia (albúmina inferior a 3 g/dl) aparece cuando la proteinuria y el catabolismo renal de la albúmina superan la capacidad de síntesis hepática. El grado de hipoalbuminemia se correlaciona con la magnitud de la proteinuria, aunque no de forma constante, ya que otros factores como la edad, el estado nutricional y el tipo de lesión renal también influyen, lo que justifica que haya pacientes con proteinurias muy elevadas sin hipoalbuminemia. Este hallazgo es característico de algunas lesiones glomerulares que cursan con hiperfiltración, como la nefropatía de la obesidad, la nefropatía de reflujo o la secundaria a reducción de masa renal. La pérdida de la albúmina y otras proteínas favorece la aparición de diversos trastornos que pueden ensombrecer la evolución del síndrome nefrótico

725.- Caso clínico seriado. Una mujer de 22 años presenta dolor vulvar severo y prurito severo desde hace tres días. Al inicio noto irritación seguida del desarrollo de ulceras abiertas, qué son dolorosas y se acompañan de febrícula. A la exploración física se encuentran los labios vulvares eritematosas y se observan ulceras abiertas qué son dolorosas al tacto. Primer enunciado: el diagnóstico clínico más probable es:

a) Sífilisb) Linfogranuloma venereoc) Chancroided) Herpes genital e) Infección por citomegalovirus

726.- Segundo enunciado: para confirmar el diagnóstico se debe practicar:

a) Examen microscópico de las secreciones vaginales con hidroxido de potasiob) Observación de las secreciones vaginales al microscopio de campo oscuroc) Búsqueda de células gigantes en el material de las lesiones ulceradas d) Búsqueda de cuerpos de inclusión de Donovan en el material de las lesiones ulceradas

MX-Biomedical Research Group / JD-MD Bioinformatics LabsJD-MD Medicine and Technology

11

Page 12: Enarm 8a Parte

ENARM by JD-MD

e) Búsqueda de coilocitos en el material de las lesiones ulceradas

Herpesvirus hominis infection of the lower genital tract (herpes genitalis) is the most common cause of genital ulcer disease in the United States. In private practice 10% of women demonstrate serologic prior exposure to the virus. Approximately 85% of primary infections are secondary to herpesvirus hominis type II, with the remainder by type I. Infection occurs through direct contact with secretions or mucosal surfaces contaminated with the virus. The virus enters the skin through cracks or other lesions but can enter through an intact mucosa. The virus initially replicates in the dermis and epidermis. Incubation time is 2–7 days. Prodromal symptoms of tingling, burning, or itching may occur shortly before vesicular eruptions appear. The vesicles erode rapidly, resulting in painful ulcers distributed in small patches, or they may involve most of the vulvar surfaces. Dysuria or other urinary symptoms may develop, including urinary retention. In severe infections, fever, malaise, and bilateral inguinal adenopathy can develop. Herpetic cervicitis causes a profuse watery discharge. Rarely a disseminated infection can follow a primary infection. In other cases the primary infection is asymptomatic. Lesions may persist for 2–6 weeks with no subsequent scarring. Diagnosis is based on clinical presentation and laboratory results. The virus can be cultured from vesicle fluid during the acute phase. However, organisms usually cannot be cultured after healing of the primary vesicles, which occurs by 2 weeks. A scraping taken from the ulcer and stained as a Papanicolaou smear can also demonstrate the characteristic giant cells indicative of viral infection, although they may also be confused with malignant cells. Other cells demonstrate a homogeneous “ground glass” appearance of cellular nuclei with numerous small intracellular, scattered basophilic particles and acidophilic inclusion bodies. Unsuspected disease is occasionally detected on a cervical or vaginal smear with a sensitivity of 50%. Approximately 85% of patients develop IgM antibodies to type II virus within 21 days of exposure. Serologic tests are best utilized to determine whether the patient has been infected in the past. A 4-fold or higher increase in neutralizing complement fixation antibody titers between acute and convalescent sera may be useful to document a primary infection. Only 5% of patients with recurrent infection demonstrate a 4-fold or higher rise in antibody titer. New type-specific serologic tests for herpes simplex virus are available. The serologic type-specific glycoprotein G-based assays should be specifically requested when serology is performed to distinguish between herpes simplex type 1 and 2. Despite the presence of adequate humoral and cell-mediated immunity, reactivation of the virus occurs. Following replication in the skin, the viral particles are transported along the peripheral sensory nerves to the dorsal root ganglion, where latent infection is established. Exogenous factors known to contribute to activation of herpesvirus include fever, emotional stress, and menstruation. Immunocompromised patients are prone to develop extensive local disease and systemic dissemination. Whether frequent coitus promotes recurrent disease is unknown. Type II virus is more likely to recur than type I virus, with men more likely to have recurrent symptoms then women. Approximately 50% of patients will have a recurrence within 6 months of the primary infection. The ulcers tend to be smaller, fewer in number, and confined to one area in the vulva, cervix, or vagina. Healing is generally complete in 1–3 weeks. Virus is not recoverable within 7 days after healing of recurrent lesions. Inguinal adenopathy and systemic symptoms generally do not occur. Primary infection can usually be distinguished from secondary infection based on clinical findings. Extragenital sites, such as the fingers, buttocks, and trunk (eczema herpeticum) have been described. The lesions of herpesvirus infection are self-limiting, and they heal spontaneously unless they become infected secondarily. Symptomatic treatment includes good genital hygiene, loose-fitting undergarments, cool compresses or sitz baths, and oral analgesics. Indications for hospitalization for a severe primary infection include urinary retention, severe headache or other systemic symptoms, and temperature greater than 101 °F (38.3 °C). Immunosuppressed patients are more prone to systemic dissemination and should be carefully managed. Treatment includes intravenous acyclovir for hospitalized patients and oral and/or topical antivirals for outpatient treatment. Recurrent herpes should be treated at the onset of prodromal symptoms or vesicle formation. Studies have indicated a decrease in the frequency and severity of recurrences with antiviral treatment. Once-daily dosing may be considered for frequent recurrent outbreaks with 40–70% of patients being free of recurrence at one year. Avoidance of direct contact with active lesions prevents spread of the disease. However, contact with an individual with subclinical disease can result in some primary infections.

727.- La diabetes insípida central secundaria a un traumatismo craneoencefálico debe ser tratada con la administración de:

a) Manitolb) Carbamazepinac) Hidroclorotiacidad) Desmopresina e) Dexametasona

La desmopresina es el tratamiento de opción en la diabetes insípida central.

728.- El fármaco que actúa indirectamente al fortalecer el efecto de la antitrombina III es:

MX-Biomedical Research Group / JD-MD Bioinformatics LabsJD-MD Medicine and Technology

12

Page 13: Enarm 8a Parte

ENARM by JD-MD

a) Heparina b) Dicumarolc) Indandionad) Warfarinae) Acenocumarina

La heparina fortalece el efecto de la antitrombina III hasta en 100 veces.

729.- La digoxina es particularmente útil para el tratamiento de la insuficiencia cardíaca cuando esta cursa con:

a) Arritmia sinusalb) Extrasistoles ventricularesc) Taquicardia supraventriculard) Fibrilación auriculare) Taquicardia sinusal

La digoxina se utiliza en la insuficiencia cardíaca que cursa con fibrilación articular.

730.- Caso clínico seriado. Un niño de 9 años, de nivel socioeconómico bajo, inicia si padecimiento actual súbitamente con fiebre y dolor en el área hepática irradiado hacia el hombro del mismo lado y se exacerba con la tos, los movimientos bruscos del tronco y la posición de decúbito lateral derecho. Primer enunciado. El diagnóstico clínico más probable es:

a) Neumonía basal derechab) Absceso hepático piógenoc) Hepatitis virald) Fasciolosis hepáticae) Absceso hepático amibiano

731.- Segundo enunciado. El laboratorio muestra leucocitosis (15,000) y anemia normocítica normocromica moderada. En estas condiciones, se debe iniciar tratamiento con:

a) Ribavirinab) Bitionolc) Metronidazol d) Cefotaximae) Ceftriaxona

Amebic liver abscess is rare in children. An increased risk is associated with travel through areas of endemic infection (Mexico, Southeast Asia). Entamoeba histolytica invasion occurs via the large bowel, although a history of diarrhea (colitis-like picture) is not always obtained. Amebic liver abscesses are usually heralded by an acute illness with high fever, chills, and leukocytosis. Early in the course, liver tests may suggest mild hepatitis. An occasional prodrome may include cough, dyspnea, and shoulder pain as rupture of the abscess into the right chest occurs. Consolidation of the right lower lobe is common (10–30% of patients). The distinction between pyogenic and amebic abscesses is best made by indirect hemagglutination test (which is positive in more than 95% of patients with amebic liver disease) and the prompt response of the latter to antiamebic therapy (metronidazole). Examination of material obtained by needle aspiration of the abscess using ultrasound guidance is often diagnostic. Amebic abscesses in uncomplicated cases should be treated promptly with oral metronidazole, 35–50 mg/kg/d, in three divided doses for 10 days. Intravenous metronidazole can be used for patients unable to take oral medication. Failure to improve after 72 hours of drug therapy indicates superimposed bacterial infection or an incorrect diagnosis. At this point, needle aspiration or surgical drainage is indicated. Once oral feedings can be tolerated, a 20-day course of iodoquinol (30 mg/kg/d in three doses) is started as a luminal amebicide. Resolution of the abscess cavity occurs over 3–6 months.

732.- Una adolescente de 14 años, con antecedente de enuresis en la etapa preescolar y nicturia desde los 9 años, hace dos meses inicia su padecimiento con hiporexia, palidez de tegumentos, astenia y adinamia. En los exámenes de laboratorio se encuentra: hemoglobina, 6.5 g/dl; glucosa, 95 mg/dl; creatinina, 14 mg/dl; urea, 306 mg/dl; sodio, 141 mEq/l; potasio, 4.6 mEq/l. El diagnóstico clínico más probable es:

a) Glomerunefritisb) Síndrome uremico

MX-Biomedical Research Group / JD-MD Bioinformatics LabsJD-MD Medicine and Technology

13

Page 14: Enarm 8a Parte

ENARM by JD-MD

c) Insuficiencia renal crónica d) Insuficiencia renal agudae) Síndrome nefrótico

Chronic renal failure (CRF) is defined as an irreversible reduction in GFR. The prevalence of CRF in the pediatric population is approximately 18 per 1 million. The prognosis for the infant, child, or adolescent with CRF has improved dramatically over the past 4 decades because of improvements in medical management (aggressive nutritional support, recombinant erythropoietin, and recombinant growth hormone), dialysis techniques, and renal transplantation. In children, CRF may be the result of congenital, acquired, inherited, or metabolic renal disease, and the underlying cause correlates closely with the age of the patient at the time when the CRF is first detected. CRF in children younger than 5 yr is most commonly a result of congenital abnormalities such as renal hypoplasia, dysplasia, and/or obstructive uropathy. After 5 yr of age, acquired diseases (various forms of glomerulonephritis) and inherited disorders (familial juvenile nephronophthisis, Alport syndrome) predominate. CRF related to metabolic disorders (cystinosis, hyperoxaluria) and certain inherited disorders (polycystic kidney disease) may present throughout the childhood years. The clinical presentation of CRF is quite varied and dependent on the underlying renal disease. Children and adolescents with CRF from glomerulonephritis, including the nephrotic syndrome, may present with edema, hypertension, hematuria, and proteinuria. Infants and children with congenital disorders such as renal dysplasia and obstructive uropathy may present in the neonatal period with failure to thrive, dehydration, urinary tract infection, or overt renal insufficiency. With the widespread use of prenatal ultrasonography, many of these infants are identified prenatally, allowing early diagnostic and therapeutic intervention. Children with familial juvenile nephronophthisis may have a very subtle presentation with nonspecific complaints such as headache, fatigue, lethargy, anorexia, vomiting, polydipsia, polyuria, and growth failure over a number of years. The physical examination in patients with CRF may reveal pallor and a sallow appearance. Patients with long-standing untreated CRF may have short stature and the bony abnormalities of renal osteodystrophy. Children with CRF due to glomerulonephritis (or children with advanced renal failure from any cause) may have edema, hypertension, and other signs of extracellular fluid volume overload. Laboratory findings include elevations in blood urea nitrogen and serum creatinine. Laboratory findings may also reveal hyperkalemia, hyponatremia (if volume overloaded), acidosis, hypocalcemia, hyperphosphatemia, and an elevation in uric acid. Patients with heavy proteinuria may have hypoalbuminemia. A complete blood cell count usually shows a normochromic, normocytic anemia. Serum cholesterol and triglyceride levels are usually elevated. In children with CRF caused by glomerulonephritis, the urinalysis shows hematuria and proteinuria. In children with CRF from congenital lesions such as renal dysplasia, the urinalysis usually has a low specific gravity and minimal abnormalities.

733.- El fármaco qué aumenta el tono del esfínter esofágico inferior y mejora la propulsión esofágica es:

a) Trimebutinab) Sucralfatoc) Cisapridad) Esomeprazole) Bromuro de pinaverio

La cisaprida es un activante integral de la onda peristáltica y se usa en pacientes con ERGE con intolerancia a la metoclopramida.

734.- El cereal que se debe emplear para espesar la formula láctea de un lactante qué presenta enfermedad por reflujo gastroesofágico es:

a) Arroz b) Avenac) Tapiocad) Cebadae) Salvado

El cereal de arroz seco espesa la formula, aunque no tiene utilidad si la lactancia es exclusivamente materna.

735.- Después de haber sufrido un traumatismo craneal, un niño de 10 años tiene 4 puntos en la escala de Glasgow y presenta respiración irregular. En este momento esta indicado:

a) Practicarle intubación endotraqueal b) Admisnitrar naloxonac) Administrarle ringer lactato

MX-Biomedical Research Group / JD-MD Bioinformatics LabsJD-MD Medicine and Technology

14

Page 15: Enarm 8a Parte

ENARM by JD-MD

d) Administrarle manitole) Administrarle dexametasona

Head injuries account for approximately half of all trauma-related deaths. An initial impact injury to the brain produces varying degrees of mechanical neuronal and axonal injury. Secondary brain injury occurs from potentially treatable factors, such as intracranial hemorrhage, cerebral edema, ischemia, hypoxia, hypotension, anemia, and increased intracranial pressure (ICP). Optimal emergency department management is paramount in helping to minimize secondary brain injury, thus decreasing the overall mortality and morbidity rates. Traumatic brain injury (TBI) results from either direct or indirect forces to the brain. Direct injury is caused by the force of an object striking the head or a penetrating injury. Indirect injuries result from acceleration-deceleration forces that result in the movement of the brain within the skull. TBI can be classified as mild, moderate, and severe. Mild TBIs include patients with a Glasgow Coma Scale (GCS) of 14 or greater. Approximately 3 percent of patients presenting with mild TBI may “talk and deteriorate” within 48 h postinjury. Patients may be asymptomatic, with only a history of head trauma, or may be confused and amnestic of the event. Patients with high-risk physical examination findings include those with a skull fracture, large subgaleal swelling, focal neurologic findings, distracting injuries, or intoxication. Moderate TBI (GCS 9 to 13) accounts for approximately 10 percent of all patients with head injuries. Overall, 40 percent of moderate TBI patients have a positive computed tomography (CT) scan, and 8 percent require neurosurgical intervention. Roughly 10 percent of these patients will deteriorate and progress to severe TBI. Severe TBI (GCS less than 9) accounts for approximately 10 percent of head injury patients. The mortality rate of severe TBI approaches 40 percent. Out-of-hospital medical personnel often may provide critical parts of the history, including mechanism and time of injury, presence and length of unconsciousness, initial mental status, seizure activity, vomiting, verbalization, and movement of extremities. For an unresponsive patient, family and friends should be contacted to gather key information, including past medical history, medications (especially anticoagulants), and recent use of alcohol or drugs. Clinically important features of the neurologic examination include assessment of the mental status and GCS; pupils for size, reactivity, and anisocoria; cranial nerve function; motor, sensory, and brainstem function; deep tendon reflexes; and any development of decorticate or decerebrate posturing.Standard protocols for evaluation and stabilization of trauma patients should be initiated (see Chap. 151). A careful search for other significant injuries should be made, since up to 60 percent of patients with severe TBI have associated major injuries.1. Administer 100% oxygen, and secure cardiac monitoring and two intravenous (IV) lines. For patients with severe TBI, endotracheal intubation to protect the airway and prevent hypoxemia is the top priority. Orotracheal rapid sequence intubation should be utilized. When properly performed, it assists in preventing increased ICP and has a low complication rate. When performing rapid sequence intubation, it is imperative to provide adequate cervical spine immobilization and use a sedation-induction agent.2. Since hypotension can lead to depressed cerebral perfusion pressure, restoration of an adequate blood pressure is vital. Resuscitation with IV crystalloid fluid is indicated. Once an adequate blood pressure is maintained, IV fluids should be administered cautiously to prevent cerebral edema. Hypotonic and glucose-containing solutions should be avoided.3. Once a positive head CT scan has been identified, immediate neurosurgical consultation is indicated.4. All patients who demonstrate signs of increased ICP should have the head of their bed elevated 30° (provided the patient is not hypotensive), adequate volume resuscitation to a mean arterial pressure greater than 90 mmHg, and maintenance of adequate arterial oxygenation. Mannitol 0.25 to 1.0 g/kg IV should be administered. Hyperventilation is no longer recommended as an intervention to lower ICP because of its potential to cause cerebral ischemia. Hyperventilation should be reserved as a last resort for decreasing ICP; if used, it should be implemented as a temporary measure and the pCO2 monitored closely.5. For posttraumatic seizures, prophylactic phenytoin use for patients with severe TBI is recommended. Seizures should be treated with benzodiazepines, such as lorazepam or diazepam, and phenytoin at a loading dose of 18 mg/kg IV infused at a rate no faster than 50 mg/min.6. Patients with an initial GCS of 15 that is maintained, normal serial neurologic examination results and a normal CT scan may be discharged home. Those with a positive CT scan require neurosurgical consultation and admission. Patients with an initial GCS of 14 and a normal CT scan should be observed in the ED for at least 6 h. If their GCS improves to 15 and they remain completely neurologically intact, they can be discharged home. All patients who experience a head injury should be discharged home with a reliable companion who can observe the patient for at least 24 h, carry out appropriate discharge instructions, and follow the head injury sheet instructions.

736.- El órgano qué más frecuentemente se lesiona en las heridas penetrantes de abdomen es el:

a) Estomagob) Hígadoc) Bazod) Intestino delgado e) Colon

MX-Biomedical Research Group / JD-MD Bioinformatics LabsJD-MD Medicine and Technology

15

Page 16: Enarm 8a Parte

ENARM by JD-MD

El intestino delgado es el órgano lesionado con más frecuencia después de lesiones penetrantes cuya frecuencia fluctúa entre 5 y 20% de los pacientes que requieren exploración quirúrgica.

737.- Una niña de 10 años ingresa al hospital por presentar un cuadro clínico de tres semanas de evolución caracterizado por edema generalizado. Las pruebas de laboratorio informan: hemoglobina, 10.5 g/dl; 5,000 leucocitos por mm3; urea, 15 mg/dl; creatinina, 2 mg/dl; albumina sérica, 2.5 g/dl; globulinas sericas, 4 g/dl y colesterol total, 280 mg/dl. El diagnóstico más probable es:

a) Síndrome nefrótico b) Glomerulonefritis agudac) Lupus eritematosod) Hipotiroidismoe) Insuficiencia renal

Nephrotic syndrome is primarily a pediatric disorder and is 15 times more common in children than adults. The incidence is 2–3/100,000 children per year, and the vast majority of affected children will have steroid-sensitive minimal change disease. The characteristic features of nephrotic syndrome are heavy proteinuria (>3.5?g/24?hr in adults or 40?mg/m2 /hr in children), hypoalbuminemia (<2.5?g/dL), edema, and hyperlipidemia. Most children (90%) with nephrotic syndrome have a form of the idiopathic nephrotic syndrome. Causes of idiopathic nephrotic syndrome include minimal change disease (85%), mesangial proliferation (5%), and focal segmental glomerulosclerosis (10%). The remaining 10% of children with nephrotic syndrome have secondary nephrotic syndrome related to glomerular diseases such as membranous nephropathy or membranoproliferative glomerulonephritis. The underlying abnormality in nephrotic syndrome is an increase in permeability of the glomerular capillary wall, which leads to massive proteinuria and hypoalbuminemia. The cause of the increased permeability is not well understood. In minimal change disease, it is possible that T-cell dysfunction leads to alteration of cytokines, which causes a loss of negatively charged glycoproteins within the glomerular capillary wall. In focal segmental glomerulosclerosis, a plasma factor, perhaps produced by lymphocytes, may be responsible for the increase in capillary wall permeability. Although the mechanism of edema formation in nephrotic syndrome is incompletely understood, it seems likely that, in most instances, urinary protein loss leads to hypoalbuminemia, which causes a decrease in the plasma oncotic pressure and transudation of fluid from the intravascular compartment to the interstitial space. The reduction in intravascular volume decreases renal perfusion pressure, activating the renin-angiotensin-aldosterone system, which stimulates tubular reabsorption of sodium. The reduced intravascular volume also stimulates the release of antidiuretic hormone, which enhances the reabsorption of water in the collecting duct. Because of the decreased plasma oncotic pressure, fluid shifts into the interstitial space, exacerbating the edema. This theory does not apply to all patients with nephrotic syndrome, however, because some patients actually have increased intravascular volume with diminished plasma levels of renin and aldosterone. Therefore, other factors, including a primary renal avidity for sodium and water, may be involved in the formation of edema in some patients with nephrotic syndrome. In the nephrotic state, serum lipid levels (cholesterol, triglycerides) are elevated for two reasons. Hypoalbuminemia stimulates generalized hepatic protein synthesis, including synthesis of lipoproteins. In addition, lipid catabolism is diminished, as a result of reduced plasma levels of lipoprotein lipase, related to increased urinary losses of this enzyme. The idiopathic nephrotic syndrome is more common in males than in females (2:1) and most commonly appears between the ages of 2 and 6 yr. It has been reported as early as 6 mo of age and throughout adulthood. The initial episode and subsequent relapses may follow minor infections and, occasionally, reactions to insect bites, bee stings, or poison ivy. Children usually present with mild edema, which is initially noted around the eyes and in the lower extremities. Nephrotic syndrome may initially be misdiagnosed as an allergic disorder because of the periorbital swelling that decreases throughout the day. With time, the edema becomes generalized, with the development of ascites, pleural effusions, and genital edema. Anorexia, irritability, abdominal pain, and diarrhea are common; hypertension and gross hematuria are uncommon. The differential diagnosis of the child with marked edema includes protein-losing enteropathy, hepatic failure, congestive heart failure, acute or chronic glomerulonephritis, and protein malnutrition. The urinalysis reveals 3+ or 4+ proteinuria; microscopic hematuria may be present in 20% of children. Urinary protein excretion exceeds 3.5?g/24?hr in adults and 40?mg/m2 /hr in children. Spot urine protein to creatinine ratio exceeds 2.0. The serum creatinine value is usually normal, but it may be increased because of diminished renal perfusion resulting from contraction of the intravascular volume. The serum albumin level is generally less than 2.5 g/dL, and the serum cholesterol and triglyceride levels are elevated. C3 and C4 levels are normal. Renal biopsy is not required for diagnosis in most children.

738.- La detención y desaceleración del crecimiento por retraso de la maduración ósea es causada por:

a) Hipotiroidismob) Desnutrición de III grado

MX-Biomedical Research Group / JD-MD Bioinformatics LabsJD-MD Medicine and Technology

16

Page 17: Enarm 8a Parte

ENARM by JD-MD

c) Raquitismo d) Osteogenesis imperfectae) Hiperparatiroidismo

Vitamin D deficiency results in rickets, a term signifying failure to mineralize growing bone or osteoid tissue, or osteomalacia. The early changes of rickets are seen radiographically at the ends of long bones, but evidence of demineralization in the shafts also is present. Subsequently, if healing is not initiated, clinical manifestations appear. New bone formation is initiated by osteoblasts, which are responsible for matrix deposition and subsequent mineralization. Osteoblasts secrete collagen, and changes in polysaccharides, phospholipids, alkaline phosphatase, and pyrophosphatase follow until mineralization occurs. However, mineralization cannot occur unless adequate calcium and phosphorus are present. Resorption of bone occurs when osteoclasts secrete enzymes on the bone surface, which dissolve and remove both matrix and mineral. Osteocytes covered by bone both resorb and redeposit bone. The many factors that affect bone growth are poorly understood, but phosphorus, calcium, fluoride, and growth hormone are known to be involved. In rickets, defective bone growth results from retardation or suppression of normal epiphyseal cartilage growth and calcification. These changes result from a deficiency of calcium and phosphorus salts in the serum. Cartilage cells fail to complete their normal cycle of proliferation and degeneration, with subsequent failure of capillary penetration, which occurs in a patchy manner. The result is a frayed, irregular epiphyseal line at the end of the shaft. Failure of osseous and cartilaginous matrix to mineralize in the zone of preparatory calcification, followed by deposition of newly formed uncalcified osteoid, results in a wide, irregular, frayed zone of nonrigid tissue (the rachitic metaphysis). This zone is responsible for many of the skeletal deformities of rickets. It becomes compressed and bulges laterally, producing flaring of the ends of the bones and the rachitic rosary. Mineralization is also lacking in subperiosteal bone. Pre-existing cortical bone is resorbed in a normal manner, but it is replaced over the entire shaft by osteoid tissue that fails to mineralize. If this process continues, the shaft loses its rigidity and the softened and rarefied cortical bone is readily distorted by stress, resulting in deformities as well as fractures. With healing, degeneration of cartilage cells along the metaphyseal-diaphyseal border occurs, capillary penetration of the resultant spaces is resumed, and calcification in the zone of preparatory calcification takes place. This calcification occurs approximately where normal calcification would have occurred had the rachitic process not supervened. It produces a line that is clearly visible on radiographs. As healing progresses, the osteoid tissue between this line of preparatory calcification and the diaphysis also becomes mineralized, as does the osteoid tissue in the cortex and about the trabeculae in the shaft.

739.- El fármaco más útil para la prevención de las crisis hipoxicas en los niños que padecen tetralogía de Fallot es:

a) Digoxinab) Disopiraminac) Dobutaminad) Propanolol e) Isoproterenol

Para el tratamiento de las crisis hipóxicas por esta causa se utiliza O2 con mascarilla o couppette y fenobarbital IM; si la crisis no cede se administra HCO3 ya que la acidosis es la causa de la prolongación del cuadro. El propranolol se ha encontrado muy útil, tanto para prevenir las crisis como para yugularse (evita el espasmo del infundíbulo pulmonar).

740.- Una indicación absoluta para efectuar la intubación endotraqueal al momento de reanimar a un recien nacido es la presencia de:

a) Líquido amniótico meconial espeso b) Frecuencia cardíaca menor de 60 por minutoc) Hernia diafragmáticad) Gastroclisise) Apnea primaria

La única indicación clara y urgente de intubación endotraqueal es que exista meconio en el líquido amniótico que el recién nacido podría broncoaspirar.

741.- La hormona necesaria para el desarrollo del sistema nervioso central es:

a) Somatotropinab) Lactogeno placentarioc) Lipotropina

MX-Biomedical Research Group / JD-MD Bioinformatics LabsJD-MD Medicine and Technology

17

Page 18: Enarm 8a Parte

ENARM by JD-MD

d) Levotiroxina e) Adrenocorticotropina

La T3 es fundamental para el desarrollo del sistema nervioso central durante la embriogénesis.

742.- Para el tratamiento de los lactantes qué se presentan alergia a las proteínas de la leche de vaca se debe utilizar la leche:

a) Deslactosadab) Soyac) Hidrolizada d) Modificada en proteínase) Polímeros de glucosa

La leche hidrolizada es la formula de elección ante la alergia a las proteínas de la leche de vaca.

743.- Una adolescente de 15 años ha perdido 15 kg de peso en los últimos 5 meses y, además, tiene amenorrea, niega vomito, diarrea y dolor abdominal. En la exploración física se encuentra caquexia y frecuencia cardíaca de 40 por minuto. El laboratorio informa: potasio de 3.0 mg/dl y bicarbonato de 30 mEq/l sedimentación globular de 3 y hematocrito de 30. El diagnóstico clínico más probable es:

a) Enfermedad inflamatoria intestinalb) Anorexia nerviosa c) Bulimiad) Enfermedad de Addisone) Adenoma hipofisiario

Anorexia nervosa and bulimia nervosa are characterized by severe disturbances of eating behavior. The salient feature of anorexia nervosa (AN) is a refusal to maintain a minimally normal body weight. Bulimia nervosa (BN) is characterized by recurrent episodes of binge eating followed by abnormal compensatory behaviors, such as self-induced vomiting. AN and BN are distinct clinical syndromes but share certain features in common. Both disorders occur primarily among previously healthy young women who become overly concerned with body shape and weight. Many patients with BN have past histories of anorexia nervosa, and many patients with anorexia nervosa engage in binge eating and purging behavior. In the current diagnostic system, the critical distinction between AN and BN depends on body weight: patients with AN are, by definition, significantly underweight, whereas patients with BN have body weights in the normal range or above. Binge eating disorder (BED) is a more recently described syndrome characterized by repeated episodes of binge eating, similar to those of BN, in the absence of inappropriate compensatory behavior. Patients with BED are typically middle-aged men or women with significant obesity. They have an increased frequency of anxiety and depression compared to similarly obese patients without BED. It is not known whether patients with BED are at increased risk for medical complications or what treatment strategies are indicated. AN2 typically begins in mid to late adolescence, sometimes in association with a stressful life event such as leaving home for school. The disorder occasionally develops in early puberty, before menarche, but seldom begins after age 40. Despite being underweight, patients with AN are irrationally afraid of gaining weight, often out of a concern that weight gain will get "out of control." They also exhibit a distortion of body image, which may express itself in several ways. For example, despite being emaciated, patients with AN may believe that their body as a whole, or some part of their body, is too fat. Further weight loss is viewed by the patient as a fulfilling accomplishment, while weight gain is seen as a personal failure. Patients with AN rarely complain of hunger or fatigue and often exercise extensively. Despite the denial of hunger, one-quarter to one-half of patients with AN engage in eating binges. Patients tend to become socially withdrawn and increasingly committed to work or study, dieting, and exercise. As weight loss progresses, thoughts of food dominate mental life and idiosyncratic rules develop around eating. Patients with AN may obsessively collect cookbooks and recipes and be drawn to food-related occupations. Patients with AN typically have few physical complaints but may note cold intolerance. Gastrointestinal motility is diminished, leading to reduced gastric emptying and constipation. Some women who develop AN after menarche report that their menses ceased before significant weight loss occurred. Weight and height should be measured to allow calculation of body mass index (BMI; kg/m2). Vital signs may reveal bradycardia, hypotension, and mild hypothermia. Soft, downy hair growth (lanugo) sometimes occurs, and alopecia may be seen. Salivary gland enlargement, which is associated with starvation as well as with binge eating and vomiting, may make the face appear surprisingly full in contrast to the marked general wasting. Acrocyanosis of the digits is common, and peripheral edema can be seen in the absence of hypoalbuminemia, particularly when the patient begins to regain weight. Some patients who consume large amounts of vegetables containing vitamin A develop a yellow tint to the skin (hypercarotenemia), which is especially notable on the palms. Mild normochromic, normocytic anemia is frequent, as is mild to moderate leukopenia, with a disproportionate reduction of polymorphonuclear

MX-Biomedical Research Group / JD-MD Bioinformatics LabsJD-MD Medicine and Technology

18

Page 19: Enarm 8a Parte

ENARM by JD-MD

leukocytes. Dehydration may result in slightly increased levels of blood urea nitrogen and creatinine. Serum transaminase levels may increase, especially during the early phases of refeeding. The level of serum proteins is usually normal. Blood sugar is often low and serum cholesterol may be moderately elevated. Hypokalemic alkalosis suggests self-induced vomiting or the use of diuretics. Hyponatremia is common and may result from excess fluid intake and disturbances in the secretion of antidiuretic hormone. The diagnosis of AN2 is based on the presence of characteristic behavioral, psychological, and physical attributes. Widely accepted diagnostic criteria are provided by the American Psychiatric Association's Diagnostic and Statistical Manual of Mental Disorders (DSM-IV). These criteria include weight <85% of that expected for age and height, which is roughly equivalent to a BMI of 18.5 kg/m2 for adult women. This weight criterion is somewhat arbitrary, so that a patient who meets all other diagnostic criteria but weighs between 85 and 90% of expected would still merit the diagnosis of AN. The current diagnostic criteria require that women with AN not have spontaneous menses, but occasional patients with the characteristics and complications of AN describe regular menstruation. Two mutually exclusive subtypes of AN are specified in DSM-IV. Patients whose weight loss is maintained primarily by caloric restriction, perhaps augmented by excessive exercise, are considered to have the "restricting" subtype of AN. The "binge eating/purging" subtype is characterized by binge eating, self-induced vomiting and/or laxative abuse. Patients with the binge/purge subtype are more prone to develop electrolyte imbalances, are more emotionally labile, and are more likely to have other problems with impulse control, such as drug abuse. The diagnosis of AN can usually be made confidently on the basis of history when significant weight loss is accomplished by restrictive dieting and excessive exercise and is accompanied by a marked reluctance to gain weight. Patients with AN often deny that they have a serious problem and may be brought to medical attention by concerned family or friends. In atypical presentations, other causes of significant weight loss in previously healthy young people should be considered, including inflammatory bowel disease, gastric outlet obstruction, diabetes mellitus, central nervous system (CNS) tumors, and neoplasm.

744.- Una complicación de la insuficiencia renal aguda secundaria a deshidratación es la:

a) Acidosis metabólica b) Hipernatremiac) Hipercalcemiad) Acidosis mixtae) Alcalosis metabólica

Prerenal ARF1 is the most common form of ARF and represents a physiologic response to mild to moderate renal hypoperfusion. Prerenal ARF is by definition rapidly reversible upon restoration of renal blood flow and glomerular ultrafiltration pressure. Renal parenchymal tissue is not damaged; indeed, kidneys from individuals with prerenal ARF function well when transplanted into recipients with normal cardiovascular function. More severe hypoperfusion may lead to ischemic injury of renal parenchyma and intrinsic renal ARF (see below). Thus, prerenal ARF and intrinsic renal ARF due to ischemia are part of a spectrum of manifestations of renal hypoperfusion. Prerenal ARF can complicate any disease that induces hypovolemia, low cardiac output, systemic vasodilatation, or selective intrarenal vasoconstriction. Hypovolemia leads to a fall in mean systemic arterial pressure, which is detected as reduced stretch by arterial (e.g., carotid sinus) and cardiac baroreceptors. Activated baroreceptors trigger a coordinated series of neural and humoral responses designed to restore blood volume and arterial pressure. These include activation of the sympathetic nervous system and renin-angiotensin-aldosterone system and release of arginine vasopressin (AVP; formerly called antidiuretic hormone). Norepinephrine, angiotensin II, and AVP act in concert in an attempt to preserve cardiac and cerebral perfusion by stimulating vasoconstriction in relatively "nonessential" vascular beds, such as the musculocutaneous and splanchnic circulations, by inhibiting salt loss through sweat glands, by stimulating thirst and salt appetite, and by promoting renal salt and water retention. Glomerular perfusion, ultrafiltration pressure, and filtration rate are preserved during mild hypoperfusion through several compensatory mechanisms. Stretch receptors in afferent arterioles, in response to a reduction in perfusion pressure, trigger afferent arteriolar vasodilatation through a local myogenic reflex (autoregulation). Biosynthesis of vasodilator prostaglandins (e.g., prostaglandin E2 and prostacyclin) is also enhanced, and these compounds preferentially dilate afferent arterioles. In addition, angiotensin II induces preferential constriction of efferent arterioles. As a result, intraglomerular pressure is maintained, the fraction of plasma flowing through glomerular capillaries that is filtered is increased (filtration fraction), and glomerular filtration rate (GFR) is preserved. During states of more severe hypoperfusion, these compensatory responses are overwhelmed and GFR falls, leading to prerenal ARF. Autoregulatory dilatation of afferent arterioles is maximal at mean systemic arterial blood pressures of ~80 mmHg, and hypotension below this level is associated with a precipitous decline in GFR2. Lesser degrees of hypotension may provoke prerenal ARF1 in the elderly and in patients with diseases affecting the integrity of afferent arterioles (e.g., hypertensive nephrosclerosis, diabetic vasculopathy). In addition, drugs that interfere with adaptive responses in the renal microcirculation may convert compensated renal hypoperfusion into overt prerenal ARF or trigger progression of prerenal ARF to ischemic intrinsic renal ARF. Pharmacologic inhibitors of either renal prostaglandin biosynthesis [cyclooxygenase inhibitors; nonsteroidal anti-inflammatory drugs (NSAIDs)] or angiotensin-converting enzyme (ACE) activity (ACE inhibitors) and angiotensin II receptor blockers are the major culprits and should be used judiciously in the setting of

MX-Biomedical Research Group / JD-MD Bioinformatics LabsJD-MD Medicine and Technology

19

Page 20: Enarm 8a Parte

ENARM by JD-MD

suspected renal hypoperfusion. NSAIDs do not compromise GFR in healthy individuals but may precipitate prerenal ARF in patients with volume depletion or in those with chronic renal insufficiency in whom GFR is maintained, in part, through prostaglandin-mediated hyperfiltration by the remaining functional nephrons. ACE inhibitors should be used with special care in patients with bilateral renal artery stenosis or unilateral stenosis in a solitary functioning kidney. In these settings glomerular perfusion and filtration may be exquisitely dependent on the actions of angiotensin II. Angiotensin II preserves glomerular filtration pressure distal to stenoses by elevating systemic arterial pressure and by triggering selective constriction of efferent arterioles. ACE inhibitors blunt these responses and precipitate ARF, usually reversible, in ~30% of these patients. ARF impairs renal excretion of sodium, potassium, and water and perturbs divalent cation homeostasis and urinary acidification mechanisms. As a result, ARF is frequently complicated by intravascular volume overload, hyponatremia, hyperkalemia, hyperphosphatemia, hypocalcemia, hypermagnesemia, and metabolic acidosis. In addition, patients are unable to excrete nitrogenous waste products and are prone to develop the uremic syndrome. The speed of development and the severity of these complications reflect the degree of renal impairment and catabolic state of the patient.

745.- Un lactante de 6 meses, eutrófico, con antecedente de infección de las vías aéreas hace una semana, inicia su padecimiento actual hace 3 días con irritabilidad, palidez de tegumentos, vomito en cuatro ocasiones, así como evacuaciones diarreicas. Al tacto rectal se palpa una tumoración. El diagnóstico más probable es:

a) Divertículo de Meckelb) Oclusión intestinalc) Volvulus intestinald) Invaginación intestinal e) Apendicitis aguda

Intussusception is the most frequent cause of intestinal obstruction in the first 2 years of life. It is three times more common in males than in females. In most cases (85%) the cause is not apparent, although polyps, Meckel diverticulum, Henoch-Schönlein purpura, lymphomas, lipomas, parasites, foreign bodies, or adenovirus or rotavirus infections with hypertrophy of Peyer patches are predisposing factors. Intussusception of the small intestine occurs in patients with celiac disease and cystic fibrosis—related to the bulk of stool in the terminal ileum. In children over age 6 years, lymphoma is the most common cause. Intermittent small bowel intussusception is a rare cause of recurrent abdominal pain. The intussusception usually starts just proximal to the ileocecal valve, so that invagination is ileocolic. Other forms include ileoileal and colocolic. Swelling, hemorrhage, incarceration with necrosis of the intussuscepted bowel, and eventual perforation and peritonitis occur as a result of impairment of venous return. Characteristically, a thriving infant aged 3–12 months develops paroxysmal abdominal pain with screaming and drawing up of the knees. Vomiting and diarrhea occur soon afterward (90% of cases), and bloody bowel movements with mucus appear within the next 12 hours (50%). Prostration and fever supervene. The abdomen is tender and becomes distended. On palpation, a sausage-shaped mass may be found usually in the upper mid abdomen. Some patients show signs of altered consciousness—particularly lethargy between spasms of pain—or have seizures. The intussusception can persist for several days when obstruction is not complete, and patients may present with separate attacks of enterocolitis. In older children, sudden attacks of abdominal pain may be related to chronic recurrent intussusception with spontaneous reduction. In July 1999, the American Academy of Pediatrics (AAP) recommended suspending oral rotavirus immunization in the United States because of an increased risk of intussusception among young infants in the weeks following oral immunization. Barium enema is both diagnostic and therapeutic. Reduction by barium enema should not be attempted if signs of strangulated bowel, perforation, or severe toxicity are present. Air insufflation of the colon under fluoroscopic guidance is a safe alternative to barium enema with excellent diagnostic sensitiviy and specificity without the risk of contaminating the abdominal cavity with barium. Great care must be observed in either air or barium enema as ischemic damage to the colon secondary to vascular compromise increases the risk of perforation. In extremely ill patients and in patients who have evidence of bowel perforation or in whom hydrostatic or pneumatic reduction has been unsuccessful (25%), surgery is required. Surgery has the advantage of identifying any lead point such as a Meckel diverticulum. Surgical reduction of intussusception is associated with a lower recurrence rate than pneumatic reduction. The prognosis relates directly to the duration of the intussusception before reduction. The mortality rate with treatment is 1–2%. The patient should be observed carefully after hydrostatic or pneumatic reduction because intussusception recurs within 24 hours in 3–4% of patients.

746.- Un hombre de 52 años, qué ha tenido tabaquismo intenso durante 20, presenta desde hace una semana, fiebre, expectoración verde-amarillenta y dolor toracico de tipo pungitivo, qué aumenta con la espiración. En la exploración física se encuentra temperatura de 38º C, taquipnea, sibilancias en ambos hemotórax, y estertores crepitantes, incremento de las vibraciones vocales y submatidez en la región infraescapular derecha. El diagnóstico clínico más probable es:

MX-Biomedical Research Group / JD-MD Bioinformatics LabsJD-MD Medicine and Technology

20

Page 21: Enarm 8a Parte

ENARM by JD-MD

a) Neumonía lobar b) Neumonía de focos múltiplesc) Bronquiectasias infectadasd) Neumonía intersticiale) Cáncer broncogeno

The pathology of pneumonia manifests as four general patterns: lobar pneumonia, bronchopneumonia, interstitial pneumonia, and miliary pneumonia. Lobar Pneumonia. Lobar pneumonia classically involves an entire lung lobe relatively homogeneously, although in some patients a small portion of the lobe may be unaffected or at an earlier stage of involvement. Four stages of lobar pneumonia may exist simultaneously in the same lung, as the tendency of the progression to be synchronous is not absolute. The first stage — congestion — occurs during the first 24 h and is characterized grossly by redness and a doughy consistency and microscopically by vascular congestion and alveolar edema. At this stage, many bacteria are present and are swept by the rapid expansion of edema fluid throughout the lobe via the pores of Kohn. Only a few neutrophils are seen at this stage. The second stage — termed red hepatization because of the color of the lung and the similarity of its airless, noncrepitant firmness to the consistency of liver — is characterized microscopically by the presence of many erythrocytes, neutrophils, desquamated epithelial cells, and fibrin in the alveolar spaces. In the third stage — gray hepatization — the lung is dry, friable, and gray-brown to yellow as a consequence of a persistent fibrinopurulent exudate, a progressive disintegration of red blood cells, and the variable presence of hemosiderin. The exudate contains macrophages as well as neutrophils, but bacteria are seldom visible. The second and third stages last for 2 to 3 days each, with a 2- to 6-day duration of maximal consolidation. The final stage — resolution — is characterized by enzymatic digestion of the alveolar exudate; resorption, phagocytosis, or coughing up of the residual debris; and restoration of the pulmonary architecture. Fibrinous inflammation may extend to and across the pleural space, causing a rub heard by auscultation, and may lead to resolution or to organization and pleural adhesions. Bronchopneumonia. Bronchopneumonia, a patchy consolidation involving one or several lobes, usually involves the dependent lower and posterior portions of the lung — a pattern attributable to the distribution of aspirated oropharyngeal contents by gravity. The consolidated areas are usually poorly demarcated, although in some cases there is an abrupt delimitation of the pneumonia at interlobular septa. The neutrophilic exudate is centered in bronchi and bronchioles, with centrifugal spread to the adjacent alveoli and diminishing cellular exudate; often there is only edema in the periphery of the lesion. Interstitial Pneumonia. Interstitial pneumonia is defined by histopathologic identification of an inflammatory process predominantly involving the interstitium, including the alveolar walls and the connective tissue around the bronchovascular tree. The inflammation may be patchy or diffuse. The alveolar septa contain an infiltration of lymphocytes, macrophages, and plasma cells. The alveoli do not contain a significant exudate, but protein-rich hyaline membranes similar to those found in adult respiratory distress syndrome (ARDS) may line the alveolar spaces. Some viruses with tropism for epithelial cells of the airways and alveoli may cause necrosis of the epithelium. In some instances, there may be a significant inflammatory exudate, with extensive degradation of inflammatory cells. Bacterial superinfection of viral pneumonia can also produce a mixed pattern of interstitial and alveolar airspace inflammation. Miliary Pneumonia. The original description of miliary pneumonia was based on the resemblance of the diffusely distributed 2- to 3-mm lesions of hematogenous tuberculosis to millet seeds. The current concept of miliary pneumonia is based on its numerous discrete lesions resulting from the spread of the pathogen to the lungs via the bloodstream. The varying degrees of immunocompromise in miliary tuberculosis, histoplasmosis, and coccidioidomycosis manifest as variations in the tissue reaction (from granulomas with caseous necrosis to foci of necrosis); the fibrinous exudate; and the weak, poorly formed cellular reaction. Miliary herpesvirus, cytomegalovirus, or varicella-zoster virus infection in severely immunocompromised patients results in numerous acute necrotizing hemorrhagic lesions.

747.- La causa más frecuente de infertilidad es la:

a) Obstrucción tubariab) Anovulación c) Incompetencia istmico-cervicald) Poliposis endometriale) Endometriosis

Infertility is generally defined as the inability of a couple to conceive within a certain period of time, usually 1 year. Infertility defies categorization as a single disease entity; there are few symptoms and few definitive tests. The goal of treatment is absolute—a successful delivery—so a sense of progress is seldom felt. Time is seen as the enemy and often the couple feels a sense of personal loss and frustration. Sterility implies an intrinsic inability to achieve pregnancy, whereas infertility implies a decrease in the ability to conceive; infertility is synonymous with subfertility. Primary infertility applies to those who have never conceived, whereas secondary infertility designates those who have conceived at some

MX-Biomedical Research Group / JD-MD Bioinformatics LabsJD-MD Medicine and Technology

21

Page 22: Enarm 8a Parte

ENARM by JD-MD

time in the past. Approximately 90% of couples with unprotected intercourse will conceive within 1 year. Fecundity is the probability of achieving a live birth in 1 menstrual cycle. Fecundability is expressed as the likelihood of conception per month of exposure. In normal fertile couples having frequent intercourse, the chances of pregnancy are estimated to be approximately 20% per month. The prevalence of infertility ranges from 7–28%, depending on the age of the woman. Sterility affects 1–2% of couples. An inverse relationship exists between fecundability and the age of the woman. The decline in fecundability begins in the early thirties and progresses rapidly in the late thirties and early forties. The number of infertility visits has increased in the last few decades. The reasons for the increase in attention given infertility are multiple. Couples in some cases have voluntarily delayed childbearing in favor of establishing careers and may experience an age-related decline in fertility. In some cases the choice of prior contraceptive method may have contributed to infertility, as with the use of some intrauterine devices (IUDs); having an increased number of sexual partners leads to a greater potential for exposure to sexually transmitted diseases, which may contribute to infertility; and couples are less willing to simply accept childlessness and are increasingly aware of the available services and options for resolving infertility. Both partners in a relationship contribute to potential fertility, and both may be subfertile. A primary diagnosis of a male factor is made in about 30% of infertile couples, and the man may be contributory in another 20–30%. An abnormality in the woman is responsible for the remaining 40–50% of cases. A conscientious evaluation of the factors contributing to fertility usually indicates a probable cause in 85–90% of couples. It is important to remember that about 15% of normal fertile couples require more than 1 year to conceive and some of the “unexplained” causes of infertility may simply be a part of this normal 15%. The success rates of treatment for infertility depends on a variety of factors, including cause of infertility, woman's age, duration of infertility, and many times cost of treatment. Health insurance plans vary a great deal in the amount and type of infertility treatments that are covered. For those couples without infertility coverage, treatment choices are frequently dictated by financial rather than medical considerations. Many times infertility treatment does not actually make the difference between conceiving and not conceiving, but in sooner rather than later conception. An ovulatory dysfunction is responsible for approximately 20–25% of infertility cases . The problem should be investigated first by review of historical factors, including the onset of menarche; present cycle length (intermenstrual interval); and presence or absence of premenstrual symptoms (molimina), such as breast tenderness, bloating, or dysmenorrhea. Signs and symptoms of systemic disease, particularly of hyperthyroidism or hypothyroidism, and physical signs of endocrine disease (eg, hirsutism, galactorrhea, and obesity) should be noted. The degree and intensity of exercise, a history of weight loss, and complaints of hot flushes all are clinical clues to possible endocrine or ovulatory dysfunction. If regular menses with molimina and mild dysmenorrhea occur at intervals of 28–32 days, and particularly if the patient notes reliable mittelschmerz, then the initial evaluation can focus on confirming ovulation with a serum progesterone assay performed in the mid-luteal phase, or the third week of the cycle. The value accepted as confirming ovulation must be determined in each endocrine laboratory. Using very specific assay reagents, the follicular-phase progesterone will be less than 1 ng/mL, and values from 3–10 ng/mL are consistent with ovulation having occurred, depending on the lab. In the case of oligomenorrhea, amenorrhea, or short or very irregular menstrual cycles, evaluation of the hypothalamic-pituitary-ovarian axis is warranted, beginning with determination of the serum concentrations of luteinizing hormone (LH), follicle-stimulating hormone (FSH), and prolactin. In some women with delayed childbearing who are seeking evaluation in their fifth decade, an evaluation of the level of FSH and estradiol in the early follicular phase may provide helpful guidance in terms of the likelihood of achieving success, as mild elevations in either FSH or estradiol may precede overt ovulatory dysfunction but still indicate a poor prognosis for successful pregnancy. The specific cause of oligo-ovulation or anovulation is determined by the history, physical examination, and appropriate laboratory studies.

748.- Las infecciones nosocomiales qué se presentan con mayor frecuencia en los pacientes quirúrgicos son las:

a) Heridas quirúrgicasb) Vías urinarias c) Vías respiratoriasd) Vascularese) Cutáneas

Infection of the lower urinary tract is the most frequently acquired nosocomial infection. Preexisting contamination of the urinary tract, urinary retention, and instrumentation are the principal contributing factors. Bacteriuria is present in about 5% of patients who undergo short-term (< 48 hours) bladder catheterization, though clinical signs of urinary tract infection occur in only 1%. Cystitis is manifested by dysuria and mild fever and pyelonephritis by high fever, flank tenderness, and, occasionally, ileus. Diagnosis is made by examination of the urine and confirmed by cultures. Prevention involves treating urinary tract contamination before surgery, prevention or prompt treatment of urinary retention, and careful instrumentation when needed. Treatment includes adequate hydration, proper drainage of the bladder, and specific antibiotics.

MX-Biomedical Research Group / JD-MD Bioinformatics LabsJD-MD Medicine and Technology

22

Page 23: Enarm 8a Parte

ENARM by JD-MD

749.- Un hombre de 35 años presenta una dermatosis diseminada en la cara, el tronco y las extremidades, constituida por ampollas flácidas ardorosas y dolorosas con conservación normal del resto de la piel. Adicionalmente hay ulceras en las mucosas. En la biopsia de piel se encuentra una ampolla intradérmica acantolítica. El diagnóstico más probable es:

a) Penfigoideb) Epidermolísis ampollosac) Pénfigo vulgar d) Pénfigo foliáceoe) Dermatitis herpetiforme

Pemphigus vulgaris (PV) is a blistering skin disease seen predominantly in elderly patients. Patients with PV have an increased incidence of the HLA-DR4 and -DRw6 serologically defined haplotypes. This disorder is characterized by the loss of cohesion between epidermal cells (a process termed acantholysis) with the resultant formation of intraepidermal blisters. Clinical lesions of PV typically consist of flaccid blisters on either normal-appearing or erythematous skin. These blisters rupture easily, leaving denuded areas that may crust and enlarge peripherally. Substantial portions of the body surface may be denuded in severe cases. Manual pressure to the skin of these patients may elicit the separation of the epidermis (Nikolsky's sign). This finding, while characteristic of PV, is not specific to this disorder and is also seen in toxic epidermal necrolysis, Stevens-Johnson syndrome, and a few other skin diseases. Lesions in PV typically present on the oral mucosa, scalp, face, neck, axilla, and trunk. In most patients, lesions begin in the mouth; involvement of other mucosal surfaces (e.g., pharyngeal, laryngeal, esophageal, conjunctival, vulval, or rectal) can occur in severe disease. Pruritus may be a feature of early pemphigus lesions; extensive denudation may be associated with severe pain. Lesions usually heal without scarring, except at sites complicated by secondary infection or mechanically induced dermal wounds. Nonetheless, postinflammatory hyperpigmentation is usually present at sites of healed lesions for some time. Biopsies of early lesions demonstrate intraepidermal vesicle formation secondary to loss of cohesion between epidermal cells (i.e., acantholytic blisters). Blister cavities contain acantholytic epidermal cells, which appear as round homogeneous cells containing hyperchromatic nuclei. Basal keratinocytes remain attached to the epidermal basement membrane, hence blister formation is within the suprabasal portion of the epidermis. Lesional skin may contain focal collections of intraepidermal eosinophils within blister cavities; dermal alterations are slight, often limited to an eosinophil-predominant leukocytic infiltrate. Direct immunofluorescence microscopy of lesional or intact patient skin shows deposits of IgG on the surface of keratinocytes; in contrast, deposits of complement components are typically found in lesional but not uninvolved skin. Deposits of IgG on keratinocytes are derived from circulating autoantibodies directed against cell-surface antigens. Circulating autoantibodies can be demonstrated in 80 to 90% of PV1 patients by indirect immunofluorescence microscopy; monkey esophagus is the optimal substrate for these studies. Patients with PV have IgG autoantibodies directed against desmogleins (Dsgs), transmembrane desmosomal glycoproteins that belong to the cadherin supergene family of calcium-dependent adhesion molecules. Such autoantibodies can now be precisely quantitated by enzyme-linked immunosorbent assay (ELISA). Most patients with early PV (i.e., only mucosal involvement) have only anti-Dsg3 autoantibodies; most patients with advanced disease (i.e., involvement of skin and mucosa) have both anti-Dsg3 and anti-Dsg1 autoantibodies. Recent studies have shown that the anti-Dsg autoantibody profile in these patients' sera as well as the tissue distribution of Dsg3 and Dsg1 determine the site of blister formation in patients with pemphigus. Experimental studies have also shown that these autoantibodies are pathogenic (i.e., responsible for blister formation) and that their titer correlates with disease activity. PV can be life-threatening. Prior to the availability of glucocorticoids, the mortality ranged from 60 to 90%; the current mortality is approximately 5%. Common causes of morbidity and mortality are infection and complications of treatment with glucocorticoids. Bad prognostic factors include advanced age, widespread involvement, and the requirement for high doses of glucocorticoids (with or without other immunosuppressive agents) for control of disease. The course of PV in individual patients is variable and difficult to predict. Some patients achieve remission (40% of patients in some series), but others may require long-term treatment or succumb to complications of their disease or its treatment. The mainstay of treatment is systemic glucocorticoids. Patients with moderate to severe disease are usually started on prednisone, 60 to 80 mg/d. If new lesions continue to appear after 1 to 2 weeks of treatment, the dose may need to be increased. Many regimens combine an immunosuppressive agent with systemic glucocorticoids for control of PV. The most frequently used are either azathioprine (1 to 2 mg/kg per day), or mycophenolate mofetil (20 to 35 mg/kg per day), cyclophosphamide (1 to 2 mg/kg per day). It is important to bring severe or progressive disease under control quickly to lessen the severity and/or duration of this disorder.

750.- La intoxicación más frecuente en los recién nacidos es la administración de:

a) Teofilinab) Anticolinergicosc) Paracetamol d) Salicilatos

MX-Biomedical Research Group / JD-MD Bioinformatics LabsJD-MD Medicine and Technology

23

Page 24: Enarm 8a Parte

ENARM by JD-MD

e) Antihistamínicos

Overdosage of acetaminophen is the most common pediatric poisoning and can produce severe hepatotoxicity. The incidence of hepatotoxicity in adults and adolescents has been reported to be ten times higher than in children under age 5 years. In the latter group, less than 0.1% develop hepatotoxicity after acetaminophen overdose. In children, toxicity most commonly results from repeated overdosage arising from confusion about the age-appropriate dose or use of multiple products that contain acetaminophen. Acetaminophen is normally metabolized in the liver. A small percentage of the drug goes through a pathway leading to a toxic metabolite. Normally, this electrophilic reactant is removed harmlessly by conjugation with glutathione. In overdosage, the supply of glutathione becomes exhausted, and the metabolite may bind covalently to components of liver cells to produce necrosis. Some authors have proposed that therapeutic doses of acetaminophen may be toxic to children with depleted glutathione stores. However, no quality evidence of this phenomenon has been reported. Treatment is to supply a surrogate glutathione by giving acetylcysteine. In the United States, it may only be given orally. If the patient has ingested acetaminophen in a liquid preparation, blood levels obtained 2 hours after ingestion will give an accurate reflection of the toxicity to be expected relative to the standard nomogram. Acetylcysteine is administered to patients whose acetaminophen levels plot in the toxic range on the nomogram. Acetylcysteine is effective even when given more than 24 hours after ingestion.

751.- En condiciones normales la válvula iliocecal se cierra cuando:

a) Ocurre una onda peristálticab) Incrementa la presión ilealc) Presenta una contracción en masad) Aumenta la presión colónica e) Estimulacion vagal

Una de las funciones principales de la válvula iliocecal consiste en evitar el reflujo del contenido cecal desde el colon hacia el intestino delgado. Las valvas de la válvula iliocecal sobresalen hacia la luz del ciego, por lo que el aumento de la presión en éste que trate de empujar su contenido hacia fuera de ellas hará que la válvula se cierre con fuerza.

752.- Un preescolar de 5 años fue llevado al servicio de urgencias tras haber ingerido de manera accidental anticongelante automotriz en cantidad desconocida. La sustancia qué más probablemente pudo haber causado la intoxicación es el:

Hidróxido de potasioHipoclorito de sodioEtilenglicolMetilenglicolGlicerol

El etilenglicol se utiliza ampliamente como anticongelante en radiadores de automóviles, y es precisamente en esta formulación como ocasiona la mayor parte de casos de intoxicación, por ingestión del producto.

753.- El mediador más importante en la producción de la fiebre en los preescolares es:

Factor inhibitorio de la inflamaciónFactor de necrosis tumoralFactor estimulante de plaquetasInterleucina 1Interleucina 6

Cytokines are small proteins (molecular mass, 10,000 to 20,000 Da) that regulate immune, inflammatory, and hematopoietic processes. For example, stimulation of lymphocyte proliferation during an immune response to vaccination is the result of the cytokines interleukin (IL) 2, IL-4, and IL-6. Another cytokine, granulocyte colony-stimulating factor, stimulates granulocytopoiesis in the bone marrow. Some cytokines cause fever and hence are called pyrogenic cytokines. From a historic point of view, the field of cytokine biology began in the 1940s with laboratory investigations into fever induction by products of activated leukocytes. These fever-producing molecules were called endogenous pyrogens. The known pyrogenic cytokines include IL-1, IL-6, tumor necrosis factor (TNF), ciliary neurotropic factor (CNTF), and interferon (IFN)a. Others probably exist, although IL-18 — a member of the IL-1 family — does not appear to be a pyrogenic cytokine. Each cytokine is encoded by a separate gene, and each pyrogenic cytokine has been shown to cause fever in laboratory animals and in humans. When injected into humans, IL-1, IL-6, and TNF produce fever at low doses

MX-Biomedical Research Group / JD-MD Bioinformatics LabsJD-MD Medicine and Technology

24

Page 25: Enarm 8a Parte

ENARM by JD-MD

(10 to 100 ng/kg). The synthesis and release of endogenous pyrogenic cytokines are induced by a wide spectrum of exogenous pyrogens, most of which have recognizable bacterial or fungal sources. Viruses also induce pyrogenic cytokines by infecting cells. However, in the absence of microbial infection, inflammation, trauma, tissue necrosis, or antigen-antibody complexes can induce the production of IL-1, TNF, and/or IL-6, which — individually or in combination — trigger the hypothalamus to raise the set point to febrile levels. The cellular sources of pyrogenic cytokines are primarily monocytes, neutrophils, and lymphocytes, although many other types of cells can synthesize these molecules when stimulated. During fever, levels of prostaglandin E2 (PGE2) are elevated in hypothalamic tissue and the third cerebral ventricle. The concentrations of PGE2 are highest near the circumventricular vascular organs (organum vasculosum of lamina terminalis) — networks of enlarged capillaries surrounding the hypothalamic regulatory centers. Destruction of these organs reduces the ability of pyrogens to produce fever. Most studies in animals have failed to show, however, that pyrogenic cytokines pass from the circulation into the brain itself. Thus, it appears that both exogenous and endogenous pyrogens interact with the endothelium of these capillaries and that this interaction is the first step in initiating fever — i.e., in raising the set point to febrile levels. As has been mentioned, several cell types can produce pyrogenic cytokines. Pyrogenic cytokines such as IL1-1, IL-6, and TNF2 are released from the cells and enter the systemic circulation. Although the systemic effects of these circulating cytokines lead to fever by inducing the synthesis of PGE23, they also induce PGE2 in peripheral tissues. The increase in PGE2 in the periphery accounts for the nonspecific myalgias and arthralgias that often accompany fever. However, it is the induction of PGE2 in the brain that starts the process of raising the hypothalamic set point for core temperature. There are four receptors for PGE2, and each signals the cell in different ways. Of the four receptors, the third (EP-3) is essential for fever: when the gene for this receptor is deleted in mice, no fever follows the injection of IL1-1 or endotoxin. Deletion of the other PGE2 receptor genes leaves the fever mechanism intact. Although PGE2 is essential for fever, it is not a neurotransmitter. Rather, the release of PGE2 from the brain side of the hypothalamic endothelium triggers the PGE2 receptor on glial cells, and this stimulation results in the rapid release of cyclic adenosine 5´-monophosphate (cyclic AMP), which is a neurotransmitter. As shown in Fig. 16-1, the release of cyclic AMP from the glial cells activates neuronal endings from the thermoregulatory center that extend into the area. The elevation of cyclic AMP is thought to account for changes in the hypothalamic set point either directly or indirectly by inducing the release of neurotransmitters. Distinct receptors for microbial products (such as endotoxins) from gram-negative bacteria and for teichoic acids from gram-positive bacteria are located on the hypothalamic endothelium. These receptors are called Toll-like receptors and are similar in many ways to IL-1 receptors. The direct activation of Toll-like receptors also results in PGE2 production and fever.

754.- El factor qué favorece la adaptación al bajo consumo de oxigeno es:

Elevación del 2,3-difosfogliceratoDisminución del 2,3-difosfogliceratoDisminución de la hemoglobinaElevación de la hemoglobinaAumento del trabajo respiratorio

El nivel de saturación de la hemoglobina depende de la curva de disociación de oxihemoglobina, en el cual la saturación de oxígeno es función de la PO2. La afinidad de hemoglobina es alta para el oxígeno a niveles altos de saturación (más de 90%, PO2 superior a 60 mmHg) y menor con saturaciones pequeñas. Los factores que desplazan la curva hacia la izquierda son lo que incrementan la afinidad de la hemoglobina por el oxígeno, mientras que aquellos que la desplazan hacia la derecha son los que reducen dicha afinidad.

MX-Biomedical Research Group / JD-MD Bioinformatics LabsJD-MD Medicine and Technology

25

Page 26: Enarm 8a Parte

ENARM by JD-MD

755.- El antagonista para el tratamiento de la intoxicación aguda por teofilina en un lactante de 7 meses es:

AtropinaFisostigminaCarbón activadoNaloxonaN-acetilcisteina

Theophylline toxicity may be caused by several of its pharmacologic effects, including inhibition of phosphodiesterase and adenosine and release of catecholamines. Theophylline may cause intoxication after an acute single overdose, or intoxication may occur as a result of chronic accidental repeated overmedication or reduced elimination resulting from hepatic dysfunction or interacting drug (eg, cimetidine, erythromycin). The usual serum half-life of theophylline is 4-6 hours, but this may increase to more than 20 hours after overdose. Caffeine and caffeine-containing herbal products can produce similar toxicity. Mild intoxication causes nausea, vomiting, tachycardia, and tremulousness. Severe intoxication is characterized by ventricular and supraventricular tachyarrhythmias, hypotension, and seizures. Status epilepticus is common and often intractable to usual anticonvulsants. After acute overdose (but not chronic intoxication), hypokalemia, hyperglycemia, and metabolic acidosis are common. Seizures and other manifestations of toxicity may be delayed for several hours after acute ingestion, especially if a sustained-release preparation such as Theo-Dur was taken. Diagnosis is based on measurement of the serum theophylline concentration. Seizures and hypotension are likely to develop in acute overdose patients with serum levels greater than 100 mg/L. Serious toxicity may develop at lower levels (ie, 40-60 mg/L) in patients with chronic intoxication. After acute ingestion, administer activated charcoal. Repeated doses of activated charcoal may enhance theophylline elimination by "gut dialysis." Addition of whole bowel irrigation should be considered for large ingestions involving sustained-release preparations. Hemodialysis is effective in removing theophylline and is indicated for patients with status epilepticus or markedly elevated serum theophylline levels (eg, > 100 mg/L after acute overdose or > 60 mg/L with chronic intoxication). Treat seizures with benzodiazepines (lorazepam, 2-3 mg intravenously, or diazepam, 5-10 mg intravenously) or phenobarbital (10-15 mg/kg intravenously). Phenytoin is not effective. Hypotension and tachycardia¾which are mediated through excessive b-adrenergic stimulation¾may respond to b-blocker therapy even in low doses: Administer esmolol, 25-50 mcg/kg/min by intravenous infusion, or propranolol, 0.5-1 mg intravenously.

756.- Para el tratamiento de la tiña de la cabeza esta indicado el uso de:

a) Champú con ketoconazolb) Miconazol en aplicaciones tópicasc) Ketoconazol por vía orald) Micostatina por vía orale) Griseofulvina por vía oral

Dermatophytes are fungi that infect skin, hair, and nails and include members of the genera Trichophyton, Microsporum, and Epidermophyton. Tinea corporis, or infection of the relatively hairless skin of the body (glabrous skin), may have a variable appearance depending on the extent of the associated inflammatory reaction. It may have the typical annular appearance of "ringworm" or appear as deep inflammatory nodules or granulomas. Involvement of the groin (tinea cruris) is more common in males than females. It presents as a scaling, erythematous eruption that spares the scrotum. Infection of the foot (tinea pedis) is the most common dermatophyte infection and is often chronic; it is characterized by variable erythema, edema, scaling, pruritus, and occasionally vesiculation. Involvement may be widespread or localized, but almost invariably involves the web space between the fourth and fifth toes. Infection of the nails (tinea unguium or onychomycosis) occurs in many patients with tinea pedis and is characterized by opacified, thickened nails and subungual debris. Dermatophyte infection of the scalp (tinea capitis) has returned in epidemic proportions, particularly affecting inner-city children, but it also affects adults. The predominant organism is T. tonsurans, which can produce a relatively noninflammatory infection with mild scale and hair loss that is diffuse or localized. Localized disease may be well defined or irregular. T. tonsurans can also cause a markedly inflammatory dermatosis with edema and nodules. This latter presentation is a kerion. The diagnosis of tinea can be made from skin scrapings, nail scrapings, or hair by culture or direct microscopic examination with potassium hydroxide (KOH). Hair, nail scrapings, and scrapings from markedly inflamed skin may fail to show hyphae on direct examination. Both topical and systemic therapies may be used to treat dermatophyte infections. Treatment depends on the site involved and the type of infection. Topical therapy is generally effective for uncomplicated tinea corporis, tinea cruris, and limited tinea pedis. It is not effective as a monotherapy for tinea capitis or onychomycosis. Topical imidazoles, triazoles, and allylamines may all be effective topical therapies for dermatophyte infections. Haloprogin, undecylenic acid, ciclopirox olamine, and tolnaftate are also effective, but nystatin is not active against dermatophytes. Topicals are generally applied twice daily, and treatment should continue 1 week beyond clinical resolution of the infection. Tinea pedis often requires

MX-Biomedical Research Group / JD-MD Bioinformatics LabsJD-MD Medicine and Technology

26

Page 27: Enarm 8a Parte

ENARM by JD-MD

longer treatment courses and frequently relapses. Oral antifungal agents may be required for recalcitrant tinea pedis or tinea corporis with a nodular (granulomatous) component. Oral antifungal agents are required for dermatophyte infections involving the hair and nails and for other infections unresponsive to topical therapy. A fungal etiology should be confirmed by direct microscopic examination or by culture prior to prescribing oral antifungal agents. All of the oral agents may cause hepatotoxicity and should not be used in women who are pregnant or breast-feeding. Griseofulvin is the only oral agent approved in the United States for dermatophyte infections involving the skin, hair, or nails. Itraconazole and terbinafine are approved for onychomycosis; however, the literature cites multiple examples of their effective use in other dermatophyte infections. When griseofulvin is used, a daily dose of 500 mg of microsized or 350 mg of ultramicrosized griseofulvin administered with a fatty meal is an adequate dose for most dermatophyte infections. Higher doses are required for tinea pedis and onychomycosis. The duration of therapy may be 2 weeks for uncomplicated tinea corporis or as long as 6 to 18 months for nail infections. Due to high relapse rates, griseofulvin is seldom used for nail infections. The usual adult dose of griseofulvin for tinea capitis is 1 g of microsized or 0.5 g of ultramicrosized given daily for 6 to 8 weeks or until cultures are negative. The adjunctive use of topical antifungal agents may be useful, but topical therapy alone is not adequate for tinea capitis. Markedly inflammatory tinea capitis may result in scarring and hair loss, and systemic or topical glucocorticoids may be helpful in preventing these sequelae. Common side effects of griseofulvin include gastrointestinal distress, headache, and urticaria. Oral itraconazole and terbinafine are approved for onychomycosis. Itraconazole is given as either continuous daily therapy (200 mg/d) or pulses (200 mg twice daily for 1 week per month) administered with food. Fingernails require 2 months of continuous therapy or two pulses. Toenails require 3 months of continuous therapy or three pulses. Itraconazole has the potential for serious interactions with other drugs requiring the P450 enzyme system for metabolism. Terbinafine (250 mg/d) is also effective for onychomycosis. Therapy with terbinafine is continued for 6 weeks for fingernail infections and 12 weeks for toenail infections. Terbinafine has fewer drug-drug interactions, but caution should be used when patients are on multiple medications.

757.- Un paciente oncológico que presenta dolor neuropático intenso debe ser tratado por medio de la administración de:

Buprenorfina y nalbufinaTramadol y metamizolAmitriptilina y morfinaTramado y morfinaClonazepam y morfina

Opioids. Opioids are still the drugs of choice for the treatment of acute episodes and acute exacerbations of neuropathic pain. Opioids strongly inhibit central nociceptive neurons mainly through interaction with mu-opioid receptors producing neuronal membrane hyperpolarization. Opioids are clearly effective in postoperative, inflammatory, and cancer pain. In some neuropathic pain syndromes, intravenous morphine is clearly analgesic when compared with placebo. However, no long-term studies have been performed on oral opioids used in the treatment of neuropathic pain. Surveys and uncontrolled studies indicate that a small fraction of the patients suffering from postherpetic neuralgia or other neuropathic pain syndromes can obtain and maintain adequate pain relief with oral opioids. Even without solid scientific evidence, the opinion of many pain clinicians is that opioids can and should be used as a part of comprehensive pain treatment program. Given that some patients with neuropathic pain may obtain considerable pain relief, opioids should be tested early in the treatment of neuropathic pains and a trial of opioids should not be delayed as a last resort. Administration of opioids requires specific treatment programs for patients with a history of chemical dependence and caution in patients with pulmonary disease. Prophylactic treatment of common side effects, notably nausea or constipation, can improve patient compliance and satisfaction with opioid therapy. Tricyclic Antidepressants. TCAs are the best-studied group of pharmacologic agents and the most widely prescribed drugs for the treatment of neuropathic pain disorders. They have been shown to have analgesic effects, although often modest and not in all patients. Controlled studies have demonstrated their efficacy in painful diabetic neuropathy independent of their effect on depression. Solid evidence suggests that the serotonin and noradrenaline reuptake blocker amitriptyline and the selective noradrenaline blocker desipramine produce pain relief in diabetic or postherpetic neuropathy. A systematic review of antidepressants for the treatment of neuropathic pain concluded that these drugs are effective: “of 100 patients …30 will obtain more than 50% pain relief”. Contrary to prior teaching, when these agents are effective, they may relieve the constant burning and deep pains, as well as lancinating pains. TCAs inhibit reuptake of monoaminergic transmitter peptides. Selective serotonin reuptake blockers are no more effective than placebo; this would suggest that efficacy of TCAs for neuropathic pain depends in a significant part on a noradrenergic component (79). It has been postulated that the site of analgesic activity is the CNS brainstem–dorsal horn nociceptive modulating system, in which the drug may alter serotonin and norepinephrine activity. The TCA drugs bind to a variety of receptor sites, not only serotonin and norepinephrine, but also histaminergic, cholinergic, and adrenergic neurotransmitter sites. The significant variability in pain relief and side effects experienced by patients among the different TCAs may be explained by the fact that each tricyclic possesses a distinct profile of activity at each of these neurotransmitter sites. TCAs also have sodium channel blocking characteristics, and

MX-Biomedical Research Group / JD-MD Bioinformatics LabsJD-MD Medicine and Technology

27

Page 28: Enarm 8a Parte

ENARM by JD-MD

this could contribute to their efficacy. Treatment with a tricyclic should be initiated with a dose of 10 or 25 mg in the evening, a few hours before bedtime to avoid early morning sedation. The lower 10-mg dose should be prescribed for the elderly, frail, or side effect–prone patient. The dose is titrated by one tablet, 10 or 25 mg, every 5 to 7 days if the patient has poor pain relief and does not complain of intolerable side effects. If effective, the majority of patients report significant pain relief or intolerable side effects within the dose range of 30 to 150 mg. The mean dose that results in pain reduction for amitriptyline is 75 to 150 mg per day, and that dose is smaller than doses necessary to achieve antidepressant effects. Onset of the analgesic effect occurs within 1 to 2 weeks and peaks around 4 to 6 weeks. Improvement in sleep, mood, and anxiety can further add to the improvement of pain control. The most common side effects of the tricyclics are caused by their anticholinergic activity: constipation, dry mouth, blurred vision, slight cognitive changes, tachycardia, and urinary hesitancy. a-Adrenergic receptor blockade may result in orthostatic hypotension. Importantly, all of these potential side effects can be minimized by following the guideline of slow titration. Contraindications to the tricyclics include closed-angle glaucoma, benign prostatic hypertrophy, and acute myocardial infarction. Sedation and weight gain may occur from histaminic activity. The tricyclics are often prescribed solely for their positive effects on sleep. Sedation without weight gain is not possible, as both result from histaminergic activity. The clinical experience with the TCAs for pain relief is poor to fair. Many patients obtain clinically insignificant pain relief and more often than not they report intolerable side effects. Although TCAs have been important in the field of neuropathic pain, their clinical utility is marginal and more effective agents are needed.

758.- Una mujer de 20 años, en tratamiento con ibuprofeno porque padece tensión premenstrual, presenta un cuadro de dos días de evolución manifestado por pápulas en zonas de piel eritematosa localizadas en la cara, la V del escote, los antebrazos y el dorso de las manos. La paciente refiere prurito y ardor en las zonas afectadas. El diagnóstico más probable es:

FotodermatosisEritrodermiaDermatitis de atópicaNeurodermatitisUrticaria

Se trata de una dermatitis fotoalérgica que es una reacción cutánea papulovesicular, eccematosa o exudativa que se produce de 24 a 48 horas después de la exposición a la luz en una persona previamente sensibilizada. La sustancia sensibilizante se concentra en la piel y necesita experimentar una alteración química provocada por la luz para convertirse en un antígeno activo, al parecer el ibuprofeno no tiene relación con el padecimiento.

759.- El tratamiento de la amenorrea secundaria de origen ginecológico se debe iniciar con la administración de:

ProgesteronaEtinilestradiolBromocriptinaDanazolHormona folículoestimulante

Idealmente una prueba de reto de progesterona (inyección intramuscular de 100 a 200 mg de progesterona en aceite o 10 mg de acetato de medroxiprogesterona oral por 5 días) produciría hemorragia (incluso manchado) en una amenorrea secundaria de origen ginecológico.

760.- Un recién nacido de termino, de 3 kg de peso, se obtuvo por parto vaginal y presentó apnea secundaria. Se le aspiraron secreciones, es liberó la vía aérea y se inicio la ventilación con bolsa y oxigeno al 100%. Se le encuentra con cianosis y frecuencia cardíaca de 60 por minuto. En este momento esta indicado:

Iniciar la compresión cardíacaPracticar intubación endotraquealAdministrar naloxonaAdministrar adrenalinaAdministrar bicarbonato

STEPS IN THE RESUSCITATION1. Dry the infant well, and place him under the radiant heat source. Do not allow the infant to become hyperthermic.2. Gently suction the mouth, then the nose.

MX-Biomedical Research Group / JD-MD Bioinformatics LabsJD-MD Medicine and Technology

28

Page 29: Enarm 8a Parte

ENARM by JD-MD

3. Quickly assess the infant's condition. The best criteria are the infant's respiratory effort (apneic, gasping, regular) and heart rate (> 100 or < 100 beats/min). A depressed heart rate—indicative of hypoxic myocardial depression—is the single most reliable indicator of the need for resuscitation.4. Infants who are breathing and have heart rates over 100 beats/min usually require no further intervention. Infants with heart rates less than 100 beats/min and apnea or irregular respiratory efforts should be stimulated vigorously. The baby's back should be rubbed with a towel while oxygen is provided near the baby's face.5. If the baby fails to respond to tactile stimulation within a few seconds, begin bag and mask ventilation, using a soft mask that seals well around the mouth and nose. For the initial inflations, pressures of 30–40 cm H2 O may be necessary to overcome surface-active forces in the lungs. Adequacy of ventilation is assessed by observing expansion of the infant's chest accompanied by an improvement in heart rate, perfusion, and color. After the first few breaths, lower the peak pressure to 15–20 cm H2 O. The baby's chest movement should resemble that of an easy breath rather than a deep sigh. The rate of bagging should be 40–60 breaths per minute.6. Most neonates can be resuscitated effectively with a bag and mask. If the infant does not respond to bag and mask ventilation, try to reposition the head (slight extension), reapply the mask to achieve a good seal, consider suctioning the mouth and the oropharynx, and try ventilating with the mouth open. If the infant does not respond within 30 seconds, intubation is appropriate.Failure to respond to intubation and ventilation can result from (1) mechanical difficulties, (2) profound asphyxia with myocardial depression, and (3) inadequate circulating blood volume. Check to be sure the endotracheal tube passes through the vocal cords. A CO2 detector placed between the endotracheal tube and the bag can be very helpful as a rapid confirmation of tube position in the airway. Occlusion of the tube should be suspected when there is resistance to bagging and no chest wall movement. Very few neonates (approximately 0.1%) require either cardiac massage or drugs during resuscitation. Almost all newborns respond to ventilation with 100% oxygen if done effectively.7. If mechanical causes are ruled out and the heart rate remains below 60 beats/min after intubation and positive-pressure ventilation for 30 seconds, cardiac compression should be initiated. Simultaneous delivery of chest compressions and positive-pressure ventilation is likely to decrease the efficiency of ventilation. Therefore, chest compressions should be interspersed with ventilation at a 3:1 ratio (90 compressions and 30 breaths per minute).8. If drugs are needed (rarely), the drug and dose of choice is epinephrine, 1:10,000 solution, 0.1–0.3 mL/kg given via the endotracheal tube or through an umbilical venous line. Sodium bicarbonate, 1–2 mEq/kg of the neonatal dilution (0.5 mEq/mL), can be used in prolonged resuscitation efforts in which the response to other measures is poor. If volume loss is suspected, 10 mL/kg of a volume expander (normal saline) should be administered through an umbilical vein line.

761.- Una lactante de 6 meses, que ha tenido en los días recientes vomito e irritabilidad, recibió tratamiento no especificado e ingresa al servicio de urgencias con taquicardia, rubicundez, distensión abdominal y midriasis: El Diagnóstico más probable es de intoxicación por:

a) Dimenhidrinato b) Acetaminofenc) Dimeticonad) Difenidole) Loratadina

El dimenhidranato (dramamine) es un antagonista H1, cuya intoxicación causa efectos excitadores centrales. El síndrome incluye alucinaciones, excitación, ataxia, incoordinación, atetosis y convulsiones. Las pupilas fijas y midriáticas con hiperemia facial junto con taquicardia sinusal, retención urinaria, xerostomía y fiebre conforman un síndrome muy similar al de intoxicación por atropina. En la fase terminal, se advierte coma cada vez más profundo con colapso cardiorrespiratorio y muerte, por lo común, en término de 12 a 18 h. El tratamiento casi siempre es sintomático, con colocación de catéteres para introducir fármacos de sostén.

762.- El uso de cocaína durante la gestación condiciona con mayor frecuencia que en recién nacido se presente:

a) Encefalopatias hipoxico-isquémica b) Enterocolitis necrosantec) Insuficiencia renal agudad) Miocardiopatia hipoxico-isquémicae) Insuficiencia hepática

Cocaine is currently the most commonly abused illicit drug, identified in up to 20–40% of pregnant women on urban delivery services; moreover, cocaine is often used in association with other drugs. The obstetric effects include maternal hypertension, decreased uterine blood flow, fetal hypoxemia, and uterine contractions. The rates of stillbirth, placental

MX-Biomedical Research Group / JD-MD Bioinformatics LabsJD-MD Medicine and Technology

29

Page 30: Enarm 8a Parte

ENARM by JD-MD

abruption, and preterm labor are increased two- to fourfold over nonusers, as is the rate of intrauterine growth restriction. Other effects in the fetus include microcephaly, cerebral infarctions, and congenital malformations caused by vascular infarcts such as intestinal atresia. In high-risk populations (no prenatal care, placental abruptions, and preterm labor), urine toxicology screens should be performed in mothers and infants. Analysis of meconium enhances diagnosis by indicating cumulative drug use prior to delivery. As with other illegal drugs, cocaine seems to have long-term neurobehavioral effects, but multiple drug use and environmental factors preclude assigning specific effects to cocaine with certainty. The risk of SIDS is increased three to seven times over the risk in nonusers (0.5–1% of exposed infants).

763.- La causa más frecuente de estupor y coma en los adultos jóvenes es de origen:

ConvulsivoTraumáticoVascularMetabólicoFarmacológico

For the adolescent population, cultural and environmental rather than organic factors pose the greatest threats to life. The three leading causes of death in the adolescent population (aged 15–19 years) in 1998 were unintentional injury (46.1%) (78% of these were caused by motor vehicle crashes), suicides (14.8%), and homicides (19.9%). These three causes of violent death accounted for 80.7% of all adolescent deaths. Deaths resulting from homicide among black 15- to 19-year-olds increased 328% between 1983 and 1993. Although the homicide rate decreased significantly since the high in 1993, homicide is still the leading cause of death for black adolescents aged 15–19 years, accounting for 41.2% of all deaths. In 1998, over one third of all deaths among adolescent males aged 15–19 years were due to homicide and suicide. Although deaths from automobile crashes have decreased over the last decade, alcohol use remains the underlying cause of most teenage motor vehicle deaths. Almost two thirds of motor vehicle deaths involving young drinking drivers occur on Friday, Saturday, or Sunday, and 70% occur between 8:00 PM and 4:00 AM. There is growing concern in the United States with the problem of youth violence. This concern is rooted in the high rate of handgun homicides among young males, the number of firearm-related suicides, and school shootings. Violent crimes committed by juveniles comprise about 1 in 4 of all violent crimes. Nationally, 20% of high school students reported carrying a weapon at least once in the preceding 30 days. However, since the peak rate of serious violent crime involving juvenile offenders in 1993, the rate has decreased. The rate of serious violent crimes among youth victims decreased from 37.6 in 1980 to 20.4 in 1999, and the rate of crimes involving youths aged 12 to 17 decreased from 34.9 to 26.1 (crimes per 1000 youths aged 12 to 17). Demographic and economic changes in the American family since the mid-1970s have had a profound effect on children and adolescents. Between 1955 and 1990, the divorce rate rose from about 400,000 to nearly 1.2 million a year. Between 1960 and 1990, the number of children involved in divorce each year increased from 460,000 to 1.1 million. There has been a dramatic decrease in the percentage of children and adolescents living in 2-parent households, from 79% in 1980 to 68% in 1998. Sixty-four percent of Hispanic and 38% of black children and adolescents under the age of 18 lived in a 2-parent household in 1998. In 1998, 18.9% of children and adolescents under age 18 years in the United States were living below the poverty level, compared with 12% in 1970. Forty-two percent of children in mother-only families were living in poverty. The major causes of morbidity during adolescence are psychosocial: unintended pregnancy, sexually transmitted diseases (STDs), substance abuse, smoking, dropping out of school, depression, running away from home, physical violence, and juvenile delinquency. High-risk behavior in one area is often associated with problems in another. For example, teenagers who live in a dysfunctional family (eg, problems related to drinking or physical or sexual abuse) are much more likely than other teenagers to be depressed. A depressed teenager is at greater risk for drug and alcohol abuse, academic failure, inappropriate sexual activity, STDs, pregnancy, and suicide.

764.- Los pacientes con hipercalciuria absortiva y litiasis renoureteral secundaria deben ser tratados por medio de:

Fosfato de celulosaTorasemideD-penicilaminaAzatioprinaBumetanida

El fosfato de celulosa es un quelante que impide la absorción de calcio en intestino.

765.- La complicación mas frecuente de la neumonía por broncoaspiración en los pacientes alcohólicos es:

Formación de bronquiectasias

MX-Biomedical Research Group / JD-MD Bioinformatics LabsJD-MD Medicine and Technology

30

Page 31: Enarm 8a Parte

ENARM by JD-MD

Enfisema pulmonarAbsceso pulmonarNeumonía lobar recurrenteBronconeumonía

Most aspiration patients with necrotizing pneumonia, lung abscess, and empyema are found to be infected with multiple species of anaerobic bacteria. Most of the remainder are infected with both anaerobic and aerobic bacteria. Prevotella melaninogenica, Peptostreptococcus, Fusobacterium nucleatum, and Bacteroides species are commonly isolated anaerobic bacteria.

766.- La medida terapéutica qué esta contraindicada en los prematuros que padecen enterocolitis necrotizante comprobada es la:

a) Administración de antibióticos de amplio espectrob) Administración de indometacina c) Alimentación parenterald) Descompresión con sonda nasogástricae) Paracentesis

Necrotizing enterocolitis is the most common acquired gastrointestinal emergency in the newborn infant; it most often affects preterm infants, with an incidence of 10% in infants of birth weight less than 1500 g. In term infants, it occurs in association with polycythemia, congenital heart disease, and birth asphyxia. The pathogenesis of the disease is a multifactorial interaction between an immature gastrointestinal tract, mucosal injury, and potentially injurious factors in the lumen. Previous intestinal ischemia, bacterial or viral infection, and immunologic immaturity of the gut are thought to play a role in the genesis of the disorder. In up to 20% of affected infants, the only risk factor is prematurity. The most common presenting sign is abdominal distention. Other signs include vomiting or increased gastric residuals, heme-positive stools, abdominal tenderness, temperature instability, increased apnea and bradycardia, decreased urine output, and poor perfusion. The complete blood count may show an increased white blood cell count with an increased band count or, as the disease progresses, absolute neutropenia. Thrombocytopenia is often observed along with stress-induced hyperglycemia and metabolic acidosis. Diagnosis is confirmed by the presence of pneumatosis intestinalis (air in the bowel wall) on x-ray. There is a spectrum of disease, and milder cases may exhibit only distention of bowel loops with bowel wall edema (thickened-appearing walls on x-ray). Necrotizing enterocolitis is managed by decompression of the gut by nasogastric tube, maintenance of oxygenation, mechanical ventilation if necessary, and intravenous fluids (normal saline) to replace third-space gastrointestinal losses. Enough fluid should be given to restore a good urine output. Other measures consist of broad-spectrum antibiotics (including anaerobic coverage), close monitoring of vital signs, physical examination, and laboratory studies (blood gases, white blood cell count, platelet count, and x-rays). Como dato importante, la indometacina se relaciona con la inducción de perforación intestinal neonatal, por lo que podría estar contraindicada en la ECN, sin embargo, no encontré ninguna indicación, ni contraindicación de paracentesis en el mismo padecimiento.

767.- Un neonato de dos días de nacido desarrolla dificultad respiratoria progresiva. Tiene estertores inspiratorios finos y un soplo sistólico suave de eyección en la mitad de superior del borde esternal izquierdo; el borde del hígado se palpa a cuatro cm por debajo del borde costal derecho y presenta pulsos periféricos saltones. El cuadro clínico del paciente corresponde a la presencia de:

a) Comunicación interauricularb) Comunicación interventricularc) Coartación de la aortad) Conducto arterioso permeable e) Estenosis de la arteria pulmonar

Patent Ductus Arteriosus (PDA)Persistence of a normal fetal channel connecting the aorta and pulmonary artery. This normally closes 12-24 hours after birth by a rise in perivascular PO2. It may be re-opened however in response to a strong stimulus such as acidosis, hypoxemia, or prostaglandin. Pathophysiology. Acyanotic defect...usually left to right shunt as blood is shunted from areas of high to low resistance. This causes pulmonary overcirculation and frequently CHF results. When ductus fails to close normally, blood will shunt from the left to the right into the pulmonary artery and lungs resulting in pulmonary overcirculation leading to CHF. A large PDA will result in a low diastolic pressure and may result in poor coronary perfusion.

MX-Biomedical Research Group / JD-MD Bioinformatics LabsJD-MD Medicine and Technology

31

Page 32: Enarm 8a Parte

ENARM by JD-MD

Assessment. Presentation depends on the magnitude of the left to right shunt. Pre-term infants may present in sever low cardiac output states. Machinery like continuous murmur can be heard at the left upper sternal border. Hx of poor feeding, irritability, tachycardia, and tachypnea. Pulse pressure is wide. Peripheral pulses may be bounding. CXR shows enlarged cardiac silhouette. Pulmonary vascular markings may be accentuated in moderate to large shunts.InterventionIV, O2, Monitor. 3cc/kg/hr D10W for infants under 1 year of age, D5W if over 1 year.Medical therapy is control CHF, consider Lasix. Closure of the PDA with indomethican if kidney function and platelet levels are assured.Surgical Ligation.

768.- El anticonvulsivante de elección en presencia de convulsiones postraumáticas en preescolares es:

a) Fenobarbitalb) Carbamazepinac) Difenilhidantoinatod) Diazepam e) Valproato de magnesio

El anticonvulsivante en este caso son las benzodiazepinas, para el tratamiento agudo, si esto no resultara, entonces se utiliza fenitoina, fenobarbital, nuevas dosis del mismo medicamento y finalmente coma barbitúrico con propofol.

769.- Las ampollas que presentan los pacientes qué padecen dermatitis herpetiforme se localiza a nivel:

a) Subepidermicob) Dermis c) Estrato lucidod) Estrato granulosoe) Estrato basal

Dermatitis herpetiformis is a rare and fascinating entity characterized by urticaria and vesicles. Males tend to be affected more frequently than are females, and the age at onset is often in the third and fourth decades, although disease has been known to develop at any age after weaning. A major association is with celiac disease; both the vesicular dermatosis and the enteropathy respond to a diet free of gluten. The urticarial plaques and vesicles of dermatitis herpetiformis are extremely pruritic. They characteristically occur bilaterally and symmetrically, involving preferentially the extensor surfaces, elbows, knees, upper back, and buttocks. Vesicles are frequently grouped, as are those of true herpesvirus, and hence the name herpetiform. The early lesions of dermatitis herpetiformis are histologically characteristic. Fibrin and neutrophils accumulate selectively at the tips of dermal papillae, forming small "microabscesses". The basal cells overlying these microabscesses show vacuolization, and minute zones of dermoepidermal separation (microscopic blisters) may occur at the tips of involved papillae. In time, these zones coalesce to form a true subepidermal blister. Eosinophils may occur in the infiltrates of older lesions, creating confusion with the histologic picture of bullous pemphigoid. Attention to the early alterations at the blister edge, however, usually allows separation of these two disorders. By direct immunofluorescence, dermatitis herpetiformis shows granular deposits of IgA selectively localized in the tips of dermal papillae, where they are deposited on anchoring fibrils. Gluten is the protein moiety that persists subsequent to the removal of water and starch from defatted flour. Gliadin is a class of protein found in the gluten fraction of flour. Patients with dermatitis herpetiformis may develop antibodies of the IgA and IgG classes to gliadin and reticulin, a component of the anchoring fibrils that tether the epidermal basement membrane to the superficial dermis. In addition, individuals with certain histocompatibility types (HLA-B8 and HLA-DRw3) are particularly prone to this disease. It is thus thought that genetically predisposed persons may develop IgA antibodies in the gut to components of dietary gluten and that these antibodies (or immune complexes) then cross-react or are deposited in the dermal papillae of the skin, resulting in clinical disease. Some individuals with dermatitis herpetiformis and enteropathy respond to a gluten-free diet (as with celiac disease).

770.- En los casos de crioglobulineamia, el marcador cutáneo es:

a) Púrpura b) Urticariac) Esclerosisd) Eritemae) Nodulo

MX-Biomedical Research Group / JD-MD Bioinformatics LabsJD-MD Medicine and Technology

32

Page 33: Enarm 8a Parte

ENARM by JD-MD

Purpura are seen when there is an extravasation of red blood cells into the dermis and, as a result, the lesions do not blanch with pressure. This is in contrast to those erythematous or violet-colored lesions that are due to localized vasodilatation — they do blanch with pressure. Purpura (=3 mm) and petechiae (=2 mm) are divided into two major groups, palpable and nonpalpable. The most frequent causes of nonpalpable petechiae and purpura are primary cutaneous disorders such as trauma, solar purpura, and capillaritis. Less common causes are steroid purpura and livedoid vasculitis (see "Ulcers," below). Solar purpura are seen primarily on the extensor forearms, while glucocorticoid purpura secondary to potent topical steroids or endogenous or exogenous Cushing's syndrome can be more widespread. In both cases there is alteration of the supporting connective tissue that surrounds the dermal blood vessels. In contrast, the petechiae that result from capillaritis are found primarily on the lower extremities. In capillaritis there is an extravasation of erythrocytes as a result of perivascular lymphocytic inflammation. The petechiae are bright red, 1 to 2 mm in size, and scattered within annular or coin-shaped yellow-brown macules. The yellow-brown color is caused by hemosiderin deposits within the dermis. Systemic causes of nonpalpable purpura fall into several categories, and those secondary to clotting disturbances and vascular fragility will be discussed first. The former group includes thrombocytopenia (Chap. 101), abnormal platelet function as is seen in uremia, and clotting factor defects. The initial site of presentation for thrombocytopenia-induced petechiae is the distal lower extremity. Capillary fragility leads to nonpalpable purpura in patients with systemic amyloidosis (see "Papulonodular Skin Lesions," above), disorders of collagen production such as Ehlers-Danlos syndrome, and scurvy. In scurvy there are flattened corkscrew hairs with surrounding hemorrhage on the lower extremities, in addition to gingivitis. Vitamin C is a cofactor for lysyl hydroxylase, an enzyme involved in the posttranslational modification of procollagen that is necessary for cross-link formation. In contrast to the previous group of disorders, the purpura seen in the following group of diseases are associated with thrombi formation within vessels. It is important to note that these thrombi are demonstrable in skin biopsy specimens. This group of disorders includes disseminated intravascular coagulation (DIC), monoclonal cryoglobulinemia, thrombotic thrombocytopenic purpura, and reactions to warfarin. DIC is triggered by several types of infection (gram-negative, gram-positive, viral, and rickettsial) as well as by tissue injury and neoplasms. Widespread purpura and hemorrhagic infarcts of the distal extremities are seen. Similar lesions are found in purpura fulminans, which is a form of DIC associated with fever and hypotension that occurs more commonly in children following an infectious illness such as varicella, scarlet fever, or an upper respiratory tract infection. In both disorders, hemorrhagic bullae can develop in involved skin. Monoclonal cryoglobulinemia is associated with multiple myeloma, Waldenstrom's macroglobulinemia, lymphocytic leukemia, and lymphoma. Purpura, primarily of the lower extremities, and hemorrhagic infarcts of the fingers and toes are seen in these patients. Exacerbations of disease activity can follow cold exposure or an increase in serum viscosity. Biopsy specimens show precipitates of the cryoglobulin within dermal vessels. Similar deposits have been found in the lung, brain, and renal glomeruli. Patients with thrombotic thrombocytopenic purpura can also have hemorrhagic infarcts as a result of intravascular thromboses. Additional signs include thrombocytopenic purpura, fever, and microangiopathic hemolytic anemia. Administration of warfarin can result in painful areas of erythema that become purpuric and then necrotic with an adherent black eschar. This reaction is seen more often in women and in areas with abundant subcutaneous fat — breasts, abdomen, buttocks, thighs, and calves. The erythema and purpura develop between the third and tenth day of therapy, most likely as a result of a transient imbalance in the levels of anticoagulant and procoagulant vitamin K-dependent factors. Continued therapy does not exacerbate preexisting lesions, and patients with an inherited or acquired deficiency of protein C are at increased risk for this particular reaction as well as for purpura fulminans. Purpura secondary to cholesterol emboli are usually seen on the lower extremities of patients with atherosclerotic vascular disease. They often follow anticoagulant therapy or an invasive vascular procedure such as an arteriogram but also occur spontaneously from disintegration of atheromatous plaques. Associated findings include livedo reticularis, gangrene, cyanosis, subcutaneous nodules, and ischemic ulcerations. Multiple step sections of the biopsy specimen may be necessary to demonstrate the cholesterol clefts with the vessels. Petechiae are also an important sign of fat embolism and occur primarily on the upper body 2 to 3 days after a major injury. By using special fixatives, the emboli can be demonstrated in biopsy specimens of the petechiae. Emboli of tumor or thrombus are seen in patients with atrial myxomas and marantic endocarditis. In the Gardner-Diamond syndrome (autoerythrocyte sensitivity), female patients develop large ecchymoses within areas of painful, warm erythema. Intradermal injections of autologous erythrocytes or phosphatidyl serine derived from the red cell membrane can reproduce the lesions in some patients; however, there are instances where a reaction is seen at an injection site of the forearm but not in the midback region. The latter has led some observers to view Gardner-Diamond syndrome as a cutaneous manifestation of severe emotional stress. Waldenstrom's hypergammaglobulinemic purpura is a chronic disorder characterized by petechiae on the lower extremities. There are circulating complexes of IgG-anti-IgG molecules, and exacerbations are associated with prolonged standing or walking. Palpable purpura are further subdivided into vasculitic and embolic. In the group of vasculitic disorders, leukocytoclastic vasculitis (LCV), also known as allergic or small-vessel vasculitis, is the one most commonly associated with palpable purpura (Chap. 306). Underlying etiologies include drugs (e.g., antibiotics), infections (e.g., hepatitis C), and connective tissue diseases. Henoch-Schonlein purpura is a subtype of acute LCV that is seen primarily in children and adolescents following an upper respiratory infection. The majority of lesions are found on the lower

MX-Biomedical Research Group / JD-MD Bioinformatics LabsJD-MD Medicine and Technology

33

Page 34: Enarm 8a Parte

ENARM by JD-MD

extremities and buttocks. Systemic manifestations include fever, arthralgias (primarily of the knees and ankles), abdominal pain, gastrointestinal bleeding, and nephritis. Direct immunofluorescence examination shows deposits of IgA within dermal blood vessel walls. In polyarteritis nodosa, specific cutaneous lesions result from a vasculitis of arterial vessels rather than postcapillary venules as in LCV. The arteritis leads to ischemia of the skin, and this explains the irregular outline of the purpura. Several types of infectious emboli can give rise to palpable purpura. These embolic lesions are usually irregular in outline as opposed to the lesions of LCV25, which are circular in outline. The irregular outline is indicative of a cutaneous infarct, and the size corresponds to the area of skin that received its blood supply from that particular arteriole or artery. The palpable purpura in LCV are circular because the erythrocytes simply diffuse out evenly from the postcapillary venules as a result of inflammation. Infectious emboli are most commonly due to gram-negative cocci (meningococcus, gonococcus), gram-negative rods (Enterobacteriaceae), and gram-positive cocci (Staphylococcus). Additional causes include Rickettsia and, in immunocompromised patients, Candida and opportunistic fungi. The embolic lesions in acute meningococcemia are found primarily on the trunk, lower extremities, and sites of pressure, and a gunmetal-gray color often develops within them. Their size varies from 1 mm to several centimeters, and the organisms can be cultured from the lesions. Associated findings include a preceding upper respiratory tract infection, fever, meningitis, DIC26, and, in some patients, a deficiency of the terminal components of complement. In disseminated gonococcal infection (arthritis-dermatitis syndrome), a small number of papules and vesicopustules with central purpura or hemorrhagic necrosis are found on the distal extremities. Additional symptoms include arthralgias, tenosynovitis, and fever. To establish the diagnosis, a Gram stain of these lesions should be performed. Rocky Mountain spotted fever is a tick-borne disease that is caused by R. rickettsii. A several-day history of fever, chills, severe headache, and photophobia precedes the onset of the cutaneous eruption. The initial lesions are erythematous macules and papules on the wrists, ankles, palms, and soles. With time, the lesions spread centripetally and become purpuric. Lesions of ecthyma gangrenosum begin as edematous, erythematous papules or plaques and then develop central purpura and necrosis. Bullae formation also occurs in these lesions, and they are frequently found in the girdle region. The organism that is classically associated with ecthyma gangrenosum is Pseudomonas aeruginosa, but other gram-negative rods such as Klebsiella, Escherichia coli, and Serratia can produce similar lesions. In immunocompromised hosts, the list of potential pathogens is expanded to include Candida and opportunistic fungi.

771.- La exposición a radiaciones y a productos químicos es un factor de riesgo relacionado con:

a) Linfoma linfocitico de células peludasb) Enfermedad de Hodgkinc) Linfoma no Hodgkind) Leucemia granulocitica agudae) Leucemia mieloide aguda

The incidence of acute myeloid leukemia (AML) is ~3.6 per 100,000 people per year, and the age-adjusted incidence is higher in men than in women (4.4 versus 3.0). AML incidence increases with age; it is 1.7 in individuals <65 years and 16.2 in those >65. A significant increase in AML incidence has occurred over the past 10 years. ETIOLOGY Heredity, radiation, chemical and other occupational exposures, and drugs have been implicated in the development of AML1. No direct evidence suggests a viral etiology. Heredity: Certain syndromes with somatic cell chromosome aneuploidy, e.g., Down (chromosome 21 trisomy), Klinefelter (XXY and variants), and Patau (chromosome 13 trisomy), are associated with an increased incidence of AML1. Inherited diseases with excessive chromatin fragility, e.g., Fanconi anemia, Bloom syndrome, ataxia telangiectasia, and Kostmann syndrome, are also associated with AML. Radiation: Survivors of the atomic bomb explosions in Japan had an increased incidence of myeloid leukemias that peaked 5 to 7 years after exposure. Therapeutic radiation alone seems to add little risk of AML1 but can increase the risk in people exposed to alkylating agents. Chemical and Other Exposures: Exposure to benzene, which is used as a solvent in the chemical, plastic, rubber, and pharmaceutical industries, is associated with an increased incidence of AML1. Smoking and exposure to petroleum products, paint, embalming fluids, ethylene oxide, herbicides, and pesticides, have also been associated with an increased risk of AML. Drugs: Anticancer drugs are the leading cause of treatment-associated AML1. Alkylating agent-associated leukemias occur on average 4 to 6 years after exposure, and affected individuals have aberrations in chromosomes 5 and 7. Topoisomerase II inhibitor-associated leukemias occur 1 to 3 years after exposure, and affected individuals often have aberrations involving chromosome 11q23. Chloramphenicol, phenylbutazone, and, less commonly, chloroquine and methoxypsoralen can result in bone marrow failure that may evolve into AML.Symptoms: Patients with AML1 most often present with nonspecific symptoms that begin gradually or abruptly and are the consequence of anemia, leukocytosis, leukopenia or leukocyte dysfunction, or thrombocytopenia. Nearly half have had symptoms for =3 months before the leukemia was diagnosed.

MX-Biomedical Research Group / JD-MD Bioinformatics LabsJD-MD Medicine and Technology

34

Page 35: Enarm 8a Parte

ENARM by JD-MD

Half mention fatigue as the first symptom, but most complain of fatigue or weakness at the time of diagnosis. Anorexia and weight loss are common. Fever with or without an identifiable infection is the initial symptom in ~10% of patients. Signs of abnormal hemostasis (bleeding, easy bruising) are noted first in 5% of patients. On occasion, bone pain, lymphadenopathy, nonspecific cough, headache, or diaphoresis is the presenting symptom. Rarely patients may present with symptoms from a mass lesion located in the soft tissues, breast, uterus, ovary, cranial or spinal dura, gastrointestinal tract, lung, mediastinum, prostate, bone, or other organs. The mass lesion represents a tumor of leukemic cells and is called a granulocytic sarcoma, or chloroma. Typical AML1 may occur simultaneously, later, or not at all in these patients. This rare presentation is more common in patients with 8;21 translocations. Physical Findings: Fever, splenomegaly, hepatomegaly, lymphadenopathy, sternal tenderness, and evidence of infection and hemorrhage are often found at diagnosis. Significant gastrointestinal bleeding, intrapulmonary hemorrhage, or intracranial hemorrhage occur most often in acute promyelocytic leukemia (APL). Bleeding associated with coagulopathy may also occur in monocytic AML1 and with extreme degrees of leukocytosis or thrombocytopenia in other morphologic subtypes. Retinal hemorrhages are detected in 15% of patients. Infiltration of the gingivae, skin, soft tissues, or the meninges with leukemic blasts at diagnosis is characteristic of the monocytic subtypes (FAB5 M4 and M5). Hematologic Findings: Anemia is usually present at diagnosis and can be severe. The degree varies considerably irrespective of other hematologic findings, splenomegaly, or the duration of symptoms. The anemia is usually normochromic normocytic. Decreased erythropoiesis often results in a reduced reticulocyte count, and erythrocyte survival is decreased by accelerated destruction. Active blood loss also contributes to the anemia. The median presenting leukocyte count is about 15,000/ul. Between 25 and 40% of patients have counts <5000/ul, and 20% have counts >100,000/ul. Fewer than 5% have no detectable leukemic cells in the blood. Poor neutrophil function may be noted functionally by impaired phagocytosis and migration and morphologically by abnormal lobulation and deficient granulation. Platelet counts <100,000/ul are found at diagnosis in ~75% of patients, and about 25% have counts <25,000/ul. Both morphologic and functional platelet abnormalities can be observed, including large and bizarre shapes with abnormal granulation and inability of platelets to aggregate or adhere normally to one another.

772.- La acción terapéutica del surfactante pulmonar exógeno se debe a que:

a) Aumenta la interfase aire-líquidob) Incrementa la tensión superficial alveolarc) Aumenta la perfusión capilard) Favorece la dilatación de los bronquiolose) Actúa sobre los neumocitos tipo 1

The alveoli are lined by two types of epithelial cells. Type I cells are flat cells with large cytoplasmic extensions and are the primary lining cells. Type II cells (granular pneumocytes) are thicker and contain numerous lamellar inclusion bodies. These cells secrete surfactant. An important factor affecting the compliance of the lungs is the surface tension of the film of fluid that lines the alveoli. The magnitude of this component at various lung volumes can be measured by removing the lungs of an experimental animal from the body and distending them alternately with saline and with air while measuring the intrapulmonary pressure. Because saline reduces the surface tension to nearly zero, the pressure-volume curve obtained with saline measures only the tissue elasticity, whereas the curve obtained with air measures both tissue elasticity and surface tension. The difference between the two curves, the elasticity due to surface tension, is much smaller at small than at large lung volumes. The surface tension is also much lower than the expected surface tension at a water-air interface of the same dimensions. The low surface tension when the alveoli are small is due to the presence in the fluid lining the alveoli of surfactant, a lipid surface-tension-lowering agent. Surfactant is a mixture of dipalmitoylphosphatidylcholine (DPPC), other lipids, and proteins. If the surface tension is not kept low when the alveoli become smaller during expiration, they collapse in accordance with the law of Laplace (see Chapter 30). In spherical structures like the alveoli, the distending pressure equals 2 times the tension divided by the radius (P = 2T/r); if T is not reduced as r is reduced, the tension overcomes the distending pressure. Surfactant also helps to prevent pulmonary edema. It has been calculated that if it were not present, the unopposed surface tension in the alveoli would produce a 20 mm Hg force favoring transudation of fluid from the blood into the alveoli. Phospholipids, which have a hydrophilic "head" and two parallel hydrophobic fatty acids "tails", line up in the alveoli with their tails facing the alveolar lumen, and surface tension is inversely proportionate to their concentration per unit area. They move farther apart as the alveoli enlarge during inspiration, and surface tension increases, whereas it decreases when they move closer together during expiration. Surfactant is produced by type II alveolar epithelial cells. Typical lamellar bodies, membrane-bound organelles containing whorls of phospholipid, are formed in these cells and secreted into the alveolar lumen by exocytosis. Tubes of lipid called tubular myelin form from the extruded bodies, and the tubular myelin in turn forms the phospholipid film. Some of the protein-lipid complexes in surfactant are taken up by endocytosis in type II alveolar cells and recycled. Formation of the phospholipid film is greatly facilitated by the proteins in surfactant. This material contains four unique proteins, SP-A, SP-B, SP-C, and SP-D. SP-A is a large glycoprotein and has a collagen-like domain within its structure. It probably has

MX-Biomedical Research Group / JD-MD Bioinformatics LabsJD-MD Medicine and Technology

35

Page 36: Enarm 8a Parte

ENARM by JD-MD

multiple functions, including regulation of the feedback uptake of surfactant by the type II alveolar epithelial cells that secrete it. SP-B and SP-C are smaller proteins, which facilitate formation of the monomolecular film of phospholipid. Like SP-A, SP-D is a glycoprotein. Its function is uncertain. However, SP-A and SP-D are members of the collectin family of proteins that are involved in innate immunity (see Chapter 27) in other parts of the body. Surfactant is important at birth. The fetus makes respiratory movements in utero, but the lungs remain collapsed until birth. After birth, the infant makes several strong inspiratory movements and the lungs expand. Surfactant keeps them from collapsing again. Surfactant deficiency is an important cause of infant respiratory distress syndrome (IRDS; hyaline membrane disease), the serious pulmonary disease that develops in infants born before their surfactant system is functional. Surface tension in the lungs of these infants is high, and there are many areas in which the alveoli are collapsed (atelectasis). An additional factor in IRDS is retention of fluid in the lungs. During fetal life, Cl- is secreted with fluid by the pulmonary epithelial cells. At birth, there is a shift to Na+ absorption by these cells via the epithelial Na+ channels (ENaCs), and fluid is absorbed with the Na+. Prolonged immaturity of the ENaCs contributes to the pulmonary abnormalities in IRDS. Administration of phospholipid alone by inhalation has little value in the treatment of IRDS. However, a synthetic surfactant and a surfactant preparation derived from bovine lungs are available for use by inhalation. Used prophylactically at birth and as replacement therapy, they decrease the severity of IRDS and the severity but not the incidence of chronic lung disease in survivors. Maturation of surfactant in the lungs is also accelerated by glucocorticoid hormones. There is an increase in fetal and maternal cortisol near term, and the lungs are rich in glucocorticoid receptors. Patchy atelectasis is also associated with surfactant deficiency in patients who have undergone cardiac surgery involving use of a pump oxygenator and interruption of the pulmonary circulation. In addition, surfactant deficiency may play a role in some of the abnormalities that develop following occlusion of a main bronchus, occlusion of one pulmonary artery, or long-term inhalation of 100% O2. There is a decrease in surfactant in the lungs of cigarette smokers. An interesting recent finding is the presence of excess surfactant lipids and proteins in mice with the GM-CSF gene knocked out. The pathologic findings in the lungs of the knockout mice resemble those in the lungs of humans with pulmonary alveolar proteinosis.

773.- La hipoacusia neurosensorial esta asociada más frecuentemente a la presencia de:

a) Baro traumab) Diabetes mellitusc) Osteopetrosisd) Ototoxicidad e) Mastoiditis crónica

HEARING LOSSA. CONDUCTIVE HEARING LOSS:Conductive hearing loss results from dysfunction of the external or middle ear. There are four mechanisms, each resulting in impairment of the passage of sound vibrations to the inner ear: (1) obstruction (eg, cerumen impaction), (2) mass loading (eg, middle ear effusion), (3) stiffness effect (eg, otosclerosis), and (4) discontinuity (eg, ossicular disruption). Conductive hearing loss is generally correctable with medical or surgical therapy—or in some cases both.B. SENSORY HEARING LOSS:Sensory hearing loss results from deterioration of the cochlea, usually due to loss of hair cells from the organ of Corti. Among the many common causes are noise trauma, ototoxicity, and aging (presbyacusis). Sensory hearing loss is not correctable with medical or surgical therapy but often may be prevented or stabilized.C. NEURAL HEARING LOSS:Neural hearing loss occurs with lesions involving the eighth nerve, auditory nuclei, ascending tracts, or auditory cortex. It is the least common clinically recognized cause of hearing loss. Causes include acoustic neuroma, multiple sclerosis, and cerebrovascular disease.

774.- La hipocalcemia qué se presenta en la mayoría de los pacientes con insuficiencia renal aguda es secundaria a:

Alteraciones de la liberación de la hormona paratiroideaAcidosis metabólicaHiperfosfatemiaHipermagnesemiaPerdida tubular de calcio

ARF impairs renal excretion of sodium, potassium, and water and perturbs divalent cation homeostasis and urinary acidification mechanisms. As a result, ARF is frequently complicated by intravascular volume overload, hyponatremia, hyperkalemia, hyperphosphatemia, hypocalcemia, hypermagnesemia, and metabolic acidosis. In addition, patients are

MX-Biomedical Research Group / JD-MD Bioinformatics LabsJD-MD Medicine and Technology

36

Page 37: Enarm 8a Parte

ENARM by JD-MD

unable to excrete nitrogenous waste products and are prone to develop the uremic syndrome. The speed of development and the severity of these complications reflect the degree of renal impairment and catabolic state of the patient. Expansion of extracellular fluid volume is an inevitable consequence of diminished salt and water excretion in oliguric or anuric individuals. Whereas milder forms are characterized by weight gain, bibasilar lung rales, raised jugular venous pressure, and dependent edema, continued volume expansion may precipitate life-threatening pulmonary edema. Hypervolemia may be particularly problematic in patients receiving multiple intravenous medications and enteral or parenteral nutrition. Excessive administration of free water either through ingestion and nasogastric administration or as hypotonic saline or isotonic dextrose solutions (dextrose being metabolized) can induce hypoosmolality and hyponatremia, which, if severe, lead to cerebral edema and neurologic abnormalities, including seizures. Hyperkalemia is a frequent complication of ARF1. Serum potassium typically rises by 0.5 mmol/L per day in oliguric and anuric patients due to impaired excretion of ingested or infused potassium and potassium released from injured tissue. Coexistent metabolic acidosis may exacerbate hyperkalemia by promoting potassium efflux from cells. Hyperkalemia may be particularly severe, even at the time of diagnosis, in patients with rhabdomyolysis, hemolysis, and tumor lysis syndrome. Mild hyperkalemia (<6.0 mmol/L) is usually asymptomatic. Higher levels may trigger electrocardiographic abnormalities and/or arrythmias. Metabolism of dietary protein yields between 50 and 100 mmol/d of fixed nonvolatile acids that are normally excreted by the kidneys. Consequently, ARF1 is typically complicated by metabolic acidosis, often with an increased serum anion gap. Acidosis can be particularly severe when endogenous production of hydrogen ions is increased by other mechanisms (e.g., diabetic or fasting ketoacidosis; lactic acidosis complicating generalized tissue hypoperfusion, liver disease, or sepsis; metabolism of ethylene glycol or methanol). Mild hyperphosphatemia is an almost invariable complication of ARF. Severe hyperphosphatemia may develop in highly catabolic patients or following rhabdomyolysis, hemolysis, or tumor lysis. Metastatic deposition of calcium phosphate can lead to hypocalcemia, particularly when the product of serum calcium (mg/dL) and phosphate (mg/dL) concentrations exceeds 70. Other factors that contribute to hypocalcemia include tissue resistance to the actions of parathyroid hormone and reduced levels of 1,25-dihydroxyvitamin D. Hypocalcemia is often asymptomatic but can cause perioral paresthesia, muscle cramps, seizures, hallucinations and confusion, and prolongation of the QT interval and nonspecific T-wave changes on electrocardiography. Anemia develops rapidly in ARF and is usually mild and multifactorial in origin. Contributing factors include impaired erythropoiesis, hemolysis, bleeding, hemodilution, and reduced red cell survival time. Prolongation of the bleeding time and leukocytosis are also common. Common contributors to the bleeding diathesis include mild thrombocytopenia, platelet dysfunction, and/or clotting factor abnormalities (e.g., factor VIII dysfunction), whereas leukocytosis usually reflects sepsis, a stress response, or other concurrent illness. Infection is a common and serious complication of ARF, occurring in 50 to 90% of cases and accounting for up to 75% of deaths. It is unclear whether patients with ARF have a clinically significant defect in host immune responses or whether the high incidence of infection reflects repeated breaches of mucocutaneous barriers (e.g., intravenous cannulae, mechanical ventilation, bladder catheterization). Cardiopulmonary complications of ARF include arrhythmias, myocardial infarction, pericarditis and pericardial effusion, pulmonary edema, and pulmonary embolism. Mild gastrointestinal bleeding is common (10 to 30%) and is usually due to stress ulceration of gastric or small intestinal mucosa. Protracted periods of severe ARF are invariably associated with the development of the uremic syndrome. A vigorous diuresis can occur during the recovery phase of ARF, which may on occasions be inappropriate and lead to intravascular volume depletion and delayed recovery of GFR2 by causing secondary prerenal ARF. Hypernatremia can also complicate recovery if water losses via hypotonic urine are not replaced or if losses are inappropriately replaced by relatively hypertonic saline solutions. Hypokalemia, hypomagnesemia, hypophosphatemia, and hypocalcemia are less common metabolic complications during this period.

775.- El intento de suicidio se presenta más frecuentemente en:

Niños preescolaresAdolescentes varonesMujeres de 20 a 30 añosHombres de 60 a 70 añosMujeres de 60 a 70 años

For the adolescent population, cultural and environmental rather than organic factors pose the greatest threats to life. The three leading causes of death in the adolescent population (aged 15–19 years) in 1998 were unintentional injury (46.1%) (78% of these were caused by motor vehicle crashes), suicides (14.8%), and homicides (19.9%). These three causes of violent death accounted for 80.7% of all adolescent deaths. Deaths resulting from homicide among black 15- to 19-year-olds increased 328% between 1983 and 1993. Although the homicide rate decreased significantly since the high in 1993, homicide is still the leading cause of death for black adolescents aged 15–19 years, accounting for 41.2% of all deaths. In 1998, over one third of all deaths among adolescent males aged 15–19 years were due to homicide and suicide. Although

MX-Biomedical Research Group / JD-MD Bioinformatics LabsJD-MD Medicine and Technology

37

Page 38: Enarm 8a Parte

ENARM by JD-MD

deaths from automobile crashes have decreased over the last decade, alcohol use remains the underlying cause of most teenage motor vehicle deaths. Almost two thirds of motor vehicle deaths involving young drinking drivers occur on Friday, Saturday, or Sunday, and 70% occur between 8:00 PM and 4:00 AM. There is growing concern in the United States with the problem of youth violence. This concern is rooted in the high rate of handgun homicides among young males, the number of firearm-related suicides, and school shootings. Violent crimes committed by juveniles comprise about 1 in 4 of all violent crimes. Nationally, 20% of high school students reported carrying a weapon at least once in the preceding 30 days. However, since the peak rate of serious violent crime involving juvenile offenders in 1993, the rate has decreased. The rate of serious violent crimes among youth victims decreased from 37.6 in 1980 to 20.4 in 1999, and the rate of crimes involving youths aged 12 to 17 decreased from 34.9 to 26.1 (crimes per 1000 youths aged 12 to 17).

776.- Los pacientes que padecen síndrome hipercinético se observan:

Tics fónicos y motores multiformes y cambiantesIrregularidad de la atención e impulsividadTrastornos por somatizaciónEstados emocionales cambiantes sin causa aparentePatrones de comportamiento antisocial

Síndrome que afecta a los niños, a los adolescentes y, en raras ocasiones, a los adultos, caracterizado por la existencia de alteraciones de conducta y en la capacidad de aprendizaje. Los síntomas comprenden trastornos de percepción, conceptualización, lenguaje, memoria y habilidades motoras, así como una disminución del tiempo de atención, un incremento de la impulsividad y de la labilidad emocional, y generalmente, aunque no siempre, una hiperactividad.

777.- El humor acuoso es producido por:

Liquefacción del vítreoCrestas del cuerpo ciliarLinfaCanal de SchlemmPars plana

The space between the lens and the retina is filled primarily with a clear gelatinous material called the vitreous (vitreous humor). Aqueous humor, a clear liquid which nourishes the cornea and lens, is produced in the ciliary body by diffusion and active transport from plasma. It flows through the pupil and fills the anterior chamber of the eye. It is normally reabsorbed through a network of trabeculae into the canal of Schlemm, a venous channel at the junction between the iris and the cornea (anterior chamber angle). Obstruction of this outlet leads to increased intraocular pressure. Increased intraocular pressure does not cause glaucoma, a degenerative disease in which there is loss of retinal ganglia cells, and a substantial minority of the patients with this disease have normal intraocular pressure (10-20 mm Hg). However, increased pressure makes glaucoma worse, and treatment is aimed at lowering the pressure. One cause of increased pressure is decreased permeability through the trabeculae (open-angle glaucoma), and another is forward movement of the iris, obliterating the angle (angle-closure glaucoma). Glaucoma can be treated with b-adrenergic blocking drugs or carbonic anhydrase inhibitors, both of which decrease the production of aqueous humor, or with cholinergic agonists, which increase aqueous outflow.

778.- Los agentes que con mayor frecuencia producen intoxicación en los preescolares son los:

MedicamentosDetergentesInsecticidasHidrocarburosAgentes biológicos

En primer lugar los medicamentos con 50%, en segundo los agentes de limpieza (detergentes con 12% e insecticidas con 12%).

779.- Un dato de intoxicación por metilxantina es un niño de ocho años es el desarrollo de:

a) Alucinacionesb) Delirioc) Ataxia cerebelosa

MX-Biomedical Research Group / JD-MD Bioinformatics LabsJD-MD Medicine and Technology

38

Page 39: Enarm 8a Parte

ENARM by JD-MD

d) Taquicardia e) Crisis convulsivas

Theophylline toxicity may be caused by several of its pharmacologic effects, including inhibition of phosphodiesterase and adenosine and release of catecholamines. Theophylline may cause intoxication after an acute single overdose, or intoxication may occur as a result of chronic accidental repeated overmedication or reduced elimination resulting from hepatic dysfunction or interacting drug (eg, cimetidine, erythromycin). The usual serum half-life of theophylline is 4-6 hours, but this may increase to more than 20 hours after overdose. Caffeine and caffeine-containing herbal products can produce similar toxicity. Mild intoxication causes nausea, vomiting, tachycardia, and tremulousness. Severe intoxication is characterized by ventricular and supraventricular tachyarrhythmias, hypotension, and seizures. Status epilepticus is common and often intractable to usual anticonvulsants. After acute overdose (but not chronic intoxication), hypokalemia, hyperglycemia, and metabolic acidosis are common. Seizures and other manifestations of toxicity may be delayed for several hours after acute ingestion, especially if a sustained-release preparation such as Theo-Dur was taken. Diagnosis is based on measurement of the serum theophylline concentration. Seizures and hypotension are likely to develop in acute overdose patients with serum levels greater than 100 mg/L. Serious toxicity may develop at lower levels (ie, 40-60 mg/L) in patients with chronic intoxication. After acute ingestion, administer activated charcoal. Repeated doses of activated charcoal may enhance theophylline elimination by "gut dialysis." Addition of whole bowel irrigation should be considered for large ingestions involving sustained-release preparations. Hemodialysis is effective in removing theophylline and is indicated for patients with status epilepticus or markedly elevated serum theophylline levels (eg, > 100 mg/L after acute overdose or > 60 mg/L with chronic intoxication). Treat seizures with benzodiazepines (lorazepam, 2-3 mg intravenously, or diazepam, 5-10 mg intravenously) or phenobarbital (10-15 mg/kg intravenously). Phenytoin is not effective. Hypotension and tachycardia¾which are mediated through excessive b-adrenergic stimulation¾may respond to b-blocker therapy even in low doses: Administer esmolol, 25-50 mcg/kg/min by intravenous infusion, or propranolol, 0.5-1 mg intravenously.

780.- La conducta a seguir en el caso de un recién nacido con datos radiológicos de neumotórax menor de 5% es la:

a) Instalación de un minisellob) Administración de oxigeno c) Ventilación mecánicad) Intubación orotraqueale) Colocación de sello pleural

Este tipo de neumotórax (de menor tamaño) se resuelven espontáneamente.

781.- Las hemorragias intracerebrales, los ictus isquemicos y la hemorragia subaracnoidea pueden aparecer como consecuencia del abuso de:

a) Cocaina b) Ácido lisergicoc) Alcohold) Heroínae) Metanefrinas

There has been an increase in both intravenous administration and inhalation of pyrolyzed cocaine via smoking. Following intranasal administration, changes in mood and sensation are perceived within 3 to 5 min, and peak effects occur at l0 to 20 min. The effects rarely last more than 1 h. Inhalation of pyrolyzed materials includes inhaling crack/cocaine or smoking coca paste, a product made by extracting cocaine preparations with flammable solvents, and cocaine free-base smoking. Free-base cocaine, including the free base prepared with sodium bicarbonate (crack), has become increasingly popular because of the relative high potency of the compound and its rapid onset of action (8 to 10 s following smoking). Cocaine produces a brief, dose-related stimulation and enhancement of mood and an increase in cardiac rate and blood pressure. Body temperature usually increases following cocaine administration, and high doses of cocaine may induce lethal pyrexia or hypertension. Because cocaine inhibits reuptake of catecholamines at adrenergic nerve endings, the drug potentiates sympathetic nervous system activity. Cocaine has a short plasma half-life of ~45 to 60 min. Cocaine is metabolized by plasma esterases, and cocaine metabolites are excreted in urine. The very short duration of the euphorigenic effects of cocaine observed in chronic abusers is probably due to both acute and chronic tolerance. Frequent self-administration of the drug (two to three times per hour) is often reported by chronic cocaine abusers. Alcohol is used to modulate both the cocaine high and the dysphoria associated with the abrupt disappearance of cocaine's effects. A metabolite of cocaine, cocaethylene, has been detected in blood and urine of persons who

MX-Biomedical Research Group / JD-MD Bioinformatics LabsJD-MD Medicine and Technology

39

Page 40: Enarm 8a Parte

ENARM by JD-MD

concurrently abuse alcohol and cocaine. Cocaethylene induces changes in cardiovascular function similar to those of cocaine alone, and the pathophysiologic consequences of alcohol abuse plus cocaine abuse may be additive when both are used together. The prevalent assumption that cocaine inhalation or intravenous administration is relatively safe is contradicted by reports of death from respiratory depression, cardiac arrhythmias, and convulsions associated with cocaine use. In addition to generalized seizures, neurologic complications may include headache, ischemic or hemorrhagic stroke, or subarachnoid hemorrhage. Disorders of cerebral blood flow and perfusion in cocaine-dependent persons have been detected with magnetic resonance spectroscopy (MRS) studies. Severe pulmonary disease may develop in individuals who inhale crack cocaine; this effect is attributed both to the direct effects of cocaine and to residual contaminants in the smoked material. Hepatic necrosis has been reported to occur following crack cocaine use. Although men and women who abuse cocaine may report that the drug enhances libidinal drive, chronic cocaine use causes significant loss of libido and adversely affects reproductive function. Impotence and gynecomastia have been observed in male cocaine abusers, and these abnormalities often persist for long periods following cessation of drug use. Women who abuse cocaine have reported major derangements in menstrual cycle function including galactorrhea, amenorrhea, and infertility. Chronic cocaine abuse may cause persistent hyperprolactinemia as a consequence of disordered dopaminergic inhibition of prolactin secretion by the anterior pituitary. Cocaine abuse by pregnant women, particularly the smoking of crack, has been associated with both an increased risk of congenital malformations in the fetus and perinatal cardiovascular and cerebrovascular disease in the mother. However, cocaine abuse per se is probably not the sole cause of these perinatal disorders, since many problems associated with maternal cocaine abuse, including poor nutrition and health care status as well as polydrug abuse, also contribute to risk for perinatal disease. Protracted cocaine abuse may cause paranoid ideation and visual and auditory hallucinations, a state that resembles alcoholic hallucinosis. Psychological dependence on cocaine, indicated by inability to abstain from frequent compulsive use, has also been reported. Although the occurrence of withdrawal syndromes involving psychomotor agitation and autonomic hyperactivity remains controversial, severe depression ("crashing") following cocaine intoxication may accompany drug withdrawal.

782.- Si el líquido de un derrame pleural contiene 3.5 gramos de proteínas por decilitro, sugiere la existencia de:

Insuficiencia cardíacaCáncer pulmonarSíndrome nefróticoInsuficiencia renalTuberculosis pleural

When a patient is found to have a pleural effusion, an effort should be made to determine the cause. The first step is to determine whether the effusion is a transudate or an exudate. A transudative pleural effusion occurs when systemic factors that influence the formation and absorption of pleural fluid are altered. The leading causes of transudative pleural effusions in the United States are left ventricular failure, pulmonary embolism, and cirrhosis. An exudative pleural effusion occurs when local factors that influence the formation and absorption of pleural fluid are altered. The leading causes of exudative pleural effusions are bacterial pneumonia, malignancy, viral infection, and pulmonary embolism. The primary reason to make this differentiation is that additional diagnostic procedures are indicated with exudative effusions to define the cause of the local disease. Transudative and exudative pleural effusions are distinguished by measuring the lactate dehydrogenase (LDH) and protein levels in the pleural fluid. Exudative pleural effusions meet at least one of the following criteria, whereas transudative pleural effusions meet none: 1. pleural fluid protein/serum protein >0.5 2. pleural fluid LDH1/serum LDH >0.6 3. pleural fluid LDH more than two-thirds normal upper limit for serum(La cantidad de proteínas (albumina) en plasma es de 3.5 a 5.5 g/dL en condiciones normales y de acuerdo a la regla numero uno antes mencionada, corresponde a un derrame pleural del tipo transudado, y la causa más común es la insuficiencia cardiada, que se explica mas adelante).The above criteria misidentify approximately 25% of transudates as exudates. If one or more of the exudative criteria are met and the patient is clinically thought to have a condition producing a transudative effusion, the difference between the albumin levels in the serum and the pleural fluid should be measured. If this gradient is greater than 12 g/L (1.2 g/dL), the exudative categorization by the above criteria can be ignored because almost all such patients have a transudative pleural effusion.Effusion due to Heart Failure: The most common cause of pleural effusion is left ventricular failure. The effusion occurs because the increased amounts of fluid in the lung interstitial spaces exit in part across the visceral pleura. This overwhelms the capacity of the lymphatics in the parietal pleura to remove fluid. A diagnostic thoracentesis should be performed if the effusions are not bilateral and comparable in size, if the patient is febrile, or if the patient has pleuritic chest pain to verify that the patient has a transudative effusion. Otherwise the patient is best treated with diuretics. If the effusion persists despite diuretic therapy, a diagnostic thoracentesis should be performed.

MX-Biomedical Research Group / JD-MD Bioinformatics LabsJD-MD Medicine and Technology

40

Page 41: Enarm 8a Parte

ENARM by JD-MD

Hepatic Hydrothorax: Pleural effusions occur in approximately 5% of patients with cirrhosis and ascites. The predominant mechanism is the direct movement of peritoneal fluid through small holes in the diaphragm into the pleural space. The effusion is usually right-sided and frequently is large enough to produce severe dyspnea. If medical management does not control the ascites and the effusion, the best treatment is a liver transplant. If the patient is not a candidate for this, the best alternative is insertion of a transjugular intrahepatic portal systemic shunt. Parapneumonic Effusion: Parapneumonic effusions are associated with bacterial pneumonia, lung abscess, or bronchiectasis and are probably the most common exudative pleural effusion in the United States. Empyema refers to a grossly purulent effusion. Patients with aerobic bacterial pneumonia and pleural effusion present with an acute febrile illness consisting of chest pain, sputum production, and leukocytosis. Patients with anaerobic infections present with a subacute illness with weight loss, a brisk leukocytosis, mild anemia, and a history of some factor that predisposes them to aspiration. The possibility of a parapneumonic effusion should be considered whenever a patient with a bacterial pneumonia is initially evaluated. The presence of free pleural fluid can be demonstrated with a lateral decubitus radiograph, computed tomography (CT) of the chest, or ultrasound. If the free fluid separates the lung from the chest wall by more than 10 mm on one of these examinations, a therapeutic thoracentesis should be performed. Factors indicating the likely need for a procedure more invasive than a thoracentesis (in increasing order of importance) include: 1. loculated pleural fluid 2. pleural fluid pH < 7.20 3. pleural fluid glucose <3.3 mmol/L (<60 mg/dL) 4. positive Gram stain or culture of the pleural fluid 5. the presence of gross pus in the pleural space If the fluid recurs after the initial therapeutic thoracentesis, a repeat thoracentesis should be performed if any of the above characteristics are present. If the fluid recurs a second time, tube thoracostomy should be performed if any of the poor prognostic factors are present. If the fluid cannot be completely removed with the therapeutic thoracentesis, consideration should be given to inserting a chest tube and instilling a thrombolytic (streptokinase, 250,000 units) or performing thoracoscopy with the breakdown of adhesions. Decortication should be considered when the above are ineffective. Effusion Secondary to Malignancy: Malignant pleural effusions secondary to metastatic disease are the second most common type of exudative pleural effusion. The three tumors that cause approximately 75% of all malignant pleural effusions are lung carcinoma, breast carcinoma, and lymphoma. Most patients complain of dyspnea, which is frequently out of proportion to the size of the effusion. The pleural fluid is an exudate, and its glucose level may be reduced if the tumor burden in the pleural space is high. BR> The diagnosis is usually made via cytology of the pleural fluid. If the initial cytologic examination is negative, then thoracoscopy is the best next procedure if malignancy is strongly suspected. At the time of thoracoscopy, a procedure such as pleural abrasion should be performed to effect a pleurodesis. If thoracoscopy is unavailable, then needle biopsy of the pleura should be performed. Patients with a malignant pleural effusion are treated symptomatically for the most part, since the presence of the effusion indicates disseminated disease and most malignancies associated with pleural effusion are not curable with chemotherapy. The only symptom that can be attributed to the effusion itself is dyspnea. If the patient's lifestyle is compromised by dyspnea, and if the dyspnea is relieved with a therapeutic thoracentesis, then one of the following procedures should be considered: (1) tube thoracostomy with the instillation of a sclerosing agent such as doxycycline, 500 mg; or (2) outpatient insertion of a small indwelling catheter. Mesothelioma: Malignant mesotheliomas are primary tumors that arise from the mesothelial cells that line the pleural cavities. Most are related to asbestos exposure. Patients with mesothelioma present with chest pain and shortness of breath. The chest radiograph reveals a pleural effusion, generalized pleural thickening, and a shrunken hemithorax. Thoracoscopy or open pleural biopsy is usually necessary to establish the diagnosis. Various treatment modalities, including radical surgery, chemotherapy, and radiation therapy, have been tried, but none has been proven to be more effective than symptomatic therapy. It is recommended that chest pain be treated with opiates and that shortness of breath be treated with oxygen and/or opiates. Effusion Secondary to Pulmonary Embolization: The diagnosis most commonly overlooked in the differential diagnosis of a patient with an undiagnosed pleural effusion is pulmonary embolism. Dyspnea is the most common symptom. The pleural fluid is usually exudative but can be transudative. The diagnosis is established by spiral CT2 scan or pulmonary arteriography. Treatment of the patient with a pleural effusion secondary to pulmonary embolism is the same as for any patient with pulmonary emboli. If the pleural effusion increases in size after anticoagulation, the patient probably has recurrent emboli or another complication such as a hemothorax or a pleural infection. Tuberculous Pleuritis: In many parts of the world, the most common cause of an exudative pleural effusion is tuberculosis (TB), but this is relatively uncommon in the United States. Tuberculous pleural effusions are thought to be due primarily to a hypersensitivity reaction to tuberculous protein in the pleural space. Patients with tuberculous pleuritis present with fever, weight loss, dyspnea, and/or pleuritic chest pain. The pleural fluid is an exudate with predominantly small lymphocytes. The diagnosis is established by demonstrating high levels of TB markers in the pleural fluid (adenosine deaminase > 45 IU/L, interferon ? > 140 pg/mL, or positive polymerase chain reaction (PCR) for tuberculous

MX-Biomedical Research Group / JD-MD Bioinformatics LabsJD-MD Medicine and Technology

41

Page 42: Enarm 8a Parte

ENARM by JD-MD

DNA). Alternatively, the diagnosis can be established by culture of the pleural fluid, needle biopsy of the pleura, or thoracoscopy. The recommended treatment of pleural and pulmonary tuberculosis is identical.Effusion Secondary to Viral Infection: Viral infections are probably responsible for a sizable percentage of undiagnosed exudative pleural effusions. In many series, no diagnosis is established for approximately 20% of exudative effusions, and these effusions resolve spontaneously with no long-term residua. The importance of these effusions is that one should not be too aggressive in trying to establish a diagnosis for the undiagnosed effusion, particularly if the patient is improving clinically. AIDS: Pleural effusions are uncommon in such patients. The most common cause is Kaposi's sarcoma, followed by parapneumonic effusion. Other common causes are TB3, cryptococcosis, and primary effusion lymphoma. Pleural effusions are very uncommon with Pneumocystis carinii infection. Chylothorax: A chylothorax occurs when the thoracic duct is disrupted and chyle accumulates in the pleural space. The most common cause of chylothorax is trauma, but it also may result from tumors in the mediastinum. Patients with chylothorax present with dyspnea, and a large pleural effusion is present on the chest radiograph. Thoracentesis reveals milky fluid, and biochemical analysis reveals a triglyceride level that exceeds 1.2 mmol/L (110 mg/dL). Patients with chylothorax and no obvious trauma should have a lymphangiogram and a mediastinal CT4 scan to assess the mediastinum for lymph nodes. The treatment of choice for most chylothoraces is implantation of a pleuroperitoneal shunt. Patients with chylothoraces should not undergo prolonged tube thoracostomy with chest tube drainage because this will lead to malnutrition and immunologic incompetence. Hemothorax: When a diagnostic thoracentesis reveals bloody pleural fluid, a hematocrit should be obtained on the pleural fluid. If the hematocrit is >50% that of the peripheral blood, the patient has a hemothorax. Most hemothoraces are the result of trauma; other causes include rupture of a blood vessel or tumor. Most patients with hemothorax should be treated with tube thoracostomy, which allows continuous quantification of bleeding. If the bleeding emanates from a laceration of the pleura, apposition of the two pleural surfaces is likely to stop the bleeding. If the pleural hemorrhage exceeds 200 mL/h, consideration should be given to thoracotomy. Miscellaneous Causes of Pleural Effusion: There are many other causes of pleural effusion (Table 245-1). Key features of some of these conditions are as follows: If the pleural fluid amylase level is elevated, the diagnosis of esophageal rupture or pancreatic disease is likely. If the patient is febrile, has predominantly polymorphonuclear cells in the pleural fluid, and has no pulmonary parenchymal abnormalities, an intraabdominal abscess should be considered. The diagnosis of an asbestos pleural effusion is one of exclusion. Benign ovarian tumors can produce ascites and a pleural effusion (Meigs' syndrome), as can the ovarian hyperstimulation syndrome. Several drugs can cause pleural effusion; the associated fluid is usually eosinophilic. Pleural effusions commonly occur following coronary artery bypass surgery. Effusions occurring within the first weeks are typically left-sided and bloody, with large numbers of eosinophils, and respond to one or two therapeutic thoracenteses. Effusions occurring after the first few weeks are typically left-sided and clear yellow, with predominantly small lymphocytes, and tend to recur. Other medical manipulations that induce pleural effusions include abdominal surgery, endoscopic variceal sclerotherapy, radiation therapy, liver or lung transplantation, or the intravascular insertion of central lines.

783.- Una paciente lúpica, que tiene lesiones purpuricas diseminadas, toma diariamente 5 mg de prednisona. El laboratorio informa: velocidad de sedimentación globular, 50 mm/hora; C3 y C4, muy bajos; hemoglobina, 9.3 g; plaquetas, 18,000. En estas condiciones, se debe:

a) Incrementar la dosis de prednisona b) Suspender la administración de prednisonac) Transfundir concentrado eritrocitariod) Iniciar plasmaferesise) Agregar vitamina K al tratamiento

The mainstay of treatment for any inflammatory life-threatening or organ-threatening manifestations of SLE1 is systemic glucocorticoids (0.5 to 2 mg/kg per day orally or 1000 mg of methylprednisolone sodium succinate intravenously daily for 3 days followed by 0.5-1 mg/kg of daily prednisone or equivalent). survival is significantly better in people with DPGN3 treated with high-dose daily glucocorticoids (40 to 60 mg of prednisone daily for 4 to 6 months) versus lower doses. Currently, high doses are recommended for much shorter periods; recent trials of interventions for severe SLE employ 4 to 6 weeks of these doses. Thereafter, doses are tapered as rapidly as the clinical situation permits, usually to a maintenance dose varying from 5 to 10 mg of prednisone, prednisolone, or equivalent per day or 10 to 20 mg every other day. Most patients with an episode of severe lupus require many years of maintenance therapy with low-dose glucocorticoids, which can be increased to prevent or treat disease flares.

784.- Un hombre de 30 años sufrio una fractura craneal deprimida hace 12 meses. Desde entonces ha experimentado episodios de crisis convulsivas parciales simples con signos motores, que actualmente han

MX-Biomedical Research Group / JD-MD Bioinformatics LabsJD-MD Medicine and Technology

42

Page 43: Enarm 8a Parte

ENARM by JD-MD

incrementado de frecuencia. En la exploración física se le encuentra orientado y conciente. El paciente debe ser tratado mediante la administración de:

Ácido valproicoEtosuccimidaLamotriginaDifenilhidantoinaCarbamacepina

Head trauma is a common cause of epilepsy in adolescents and adults. The head injury can be caused by a variety of mechanisms, and the likelihood of developing epilepsy is strongly correlated with the severity of the injury. A patient with a penetrating head wound, depressed skull fracture, intracranial hemorrhage, or prolonged posttraumatic coma or amnesia has a 40 to 50% risk of developing epilepsy, while a patient with a closed head injury and cerebral contusion has a 5 to 25% risk. Recurrent seizures usually develop within 1 year after head trauma, although intervals of =10 years are well known. In controlled studies, mild head injury, defined as a concussion with amnesia or loss of consciousness of <30 min, was found to be associated with only a slightly increased likelihood of epilepsy. Nonetheless, most epileptologists know of patients who have partial seizures within hours or days of a mild head injury and subsequently develop chronic seizures of the same type; such cases may represent rare examples of chronic epilepsy resulting from mild head injury. Carbamazepine, phenytoin, or lamotrigine is currently the initial drug of choice for the treatment of partial seizures, including those that secondarily generalize. Overall they have very similar efficacy, but differences in pharmacokinetics and toxicity are the main determinants for use in a given patient. Phenytoin has a relatively long half-life and offers the advantage of once or twice daily dosing compared to two or three times daily dosing for carbamazepine (although a more expensive, extended-release form of carbamazepine is now available) and lamotrigine. An advantage of carbamazepine is that its metabolism follows first-order pharmacokinetics, and the relationship between drug dose, serum levels, and toxicity is linear. By contrast, phenytoin shows properties of saturation kinetics, such that small increases in phenytoin doses above a standard maintenance dose can precipitate marked side effects. This is one of the main causes of acute phenytoin toxicity. Long-term use of phenytoin is associated with untoward cosmetic effects (e.g., hirsutism, coarsening of facial features, and gingival hypertrophy), and effects on bone metabolism, so it is often avoided in young patients who are likely to require the drug for many years. Carbamazepine can cause leukopenia, aplastic anemia, or hepatotoxicity and would therefore be contraindicated in patients with predispositions to these problems. A major concern with lamotrigine is the occurrence of skin rash during the initiation of therapy. This can be extremely severe and lead to Stevens-Johnson syndrome if unrecognized and if the medication is not discontinued immediately. This risk can be reduced by slow introduction and titration. Lamotrigine must be started slowly when used as add-on therapy with valproic acid, since valproic acid can inhibit its metabolism, thereby substantially prolonging its half-life. Valproic acid is an effective alternative for some patients with partial seizures, especially when the seizures secondarily generalize. Gastrointestinal side effects are fewer when using the valproate semisodium formulation (Depakote). Valproic acid also rarely causes reversible bone marrow suppression and hepatotoxicity, and laboratory testing is required to monitor toxicity. This drug should generally be avoided in patients with preexisting bone marrow or liver disease. Irreversible, fatal hepatic failure appearing as an idiosyncratic rather than dose-related side effect is a relatively rare complication; its risk is highest in children <2 years old, especially those taking other antiepileptic drugs or with inborn errors of metabolism. Topiramate, tiagabine, levetiracetam, zonisamide, gabapentin, and oxcarbazepine are additional drugs currently used for the treatment of partial seizures with or without secondary generalization. Until recently, phenobarbital and other barbiturate compounds were commonly used as first-line therapy for many forms of epilepsy. However, the barbiturates frequently cause sedation in adults, hyperactivity in children, and other more subtle cognitive changes; thus, their use should be limited to situations in which no other suitable treatment alternatives exist.

785.- Lo más efectivo para tratar un derrame pleural recidivante en un paciente con una neoplasia maligna es la:

a) Quimioterapiab) Pleurectomiac) Radioterapiad) Pleurodesis química e) Abrasión mecánica

Effusion Secondary to Malignancy: Malignant pleural effusions secondary to metastatic disease are the second most common type of exudative pleural effusion. The three tumors that cause approximately 75% of all malignant pleural effusions are lung carcinoma, breast carcinoma, and lymphoma. Most patients complain of dyspnea, which is frequently out of proportion to the size of the effusion. The pleural fluid is an exudate, and its glucose level may be reduced if the tumor burden in the pleural space is high. BR>

MX-Biomedical Research Group / JD-MD Bioinformatics LabsJD-MD Medicine and Technology

43

Page 44: Enarm 8a Parte

ENARM by JD-MD

The diagnosis is usually made via cytology of the pleural fluid. If the initial cytologic examination is negative, then thoracoscopy is the best next procedure if malignancy is strongly suspected. At the time of thoracoscopy, a procedure such as pleural abrasion should be performed to effect a pleurodesis. If thoracoscopy is unavailable, then needle biopsy of the pleura should be performed. Patients with a malignant pleural effusion are treated symptomatically for the most part, since the presence of the effusion indicates disseminated disease and most malignancies associated with pleural effusion are not curable with chemotherapy. The only symptom that can be attributed to the effusion itself is dyspnea. If the patient's lifestyle is compromised by dyspnea, and if the dyspnea is relieved with a therapeutic thoracentesis, then one of the following procedures should be considered: (1) tube thoracostomy with the instillation of a sclerosing agent such as doxycycline, 500 mg; or (2) outpatient insertion of a small indwelling catheter.

786.- El fármaco más útil para tratar los síntomas propios de los episodios de migraña es:

a) Ergotamina b) Sumatriptanc) Cafeinad) Ketorolacoe) Flunarizina

The mainstay of pharmacologic therapy is the judicious use of one or more of the many drugs that are effective in migraine. The selection of the optimal regimen for a given patient depends on a number of factors, the most important of which is the severity of the attack. Mild migraine attacks can usually be managed by oral agents; the average efficacy rate is 50 to 70%. Severe migraine attacks may require parenteral therapy. Most drugs effective in the treatment of migraine are members of one of three major pharmacologic classes: anti-inflammatory agents, 5-HT agonists, and dopamine antagonists. In general, an adequate dose of whichever agent is chosen should be used as soon as possible after the onset of an attack. If additional medication is required within 60 min because symptoms return or have not abated, the initial dose should be increased for subsequent attacks. Migraine therapy must be individualized for each patient; a standard approach for all patients is not possible. A therapeutic regimen may need to be constantly refined and personalized until one is identified that provides the patient with rapid, complete, and consistent relief with minimal side effects. NONSTEROIDAL ANTI-INFLAMMATORY AGENTSBoth the severity and duration of a migraine attack can be reduced significantly by anti-inflammatory agents. Indeed, many undiagnosed migraineurs are self-treated with nonprescription anti-inflammatory agents (Table 14-4). A general consensus is that NSAIDs8 are most effective when taken early in the migraine attack. However, the effectiveness of anti-inflammatory agents in migraine is usually less than optimal in moderate or severe migraine attacks. The combination of acetaminophen, aspirin, and caffeine (Excedrin Migraine) has been approved for use by the U.S. Food and Drug Administration (FDA) for the treatment of mild to moderate migraine. The combination of aspirin and metoclopramide has been shown to be equivalent to a single dose of sumatriptan. Major side effects of NSAIDs include dyspepsia and gastrointestinal irritation. 5-HT AGONISTSOral: Stimulation of 5-HT101 receptors can stop an acute migraine attack. Ergotamine and dihydroergotamine are nonselective receptor agonists, while the series of drugs known as triptans are selective 5-HT1 receptor agonists. A variety of triptans (e.g., naratriptan, rizatriptan, sumatriptan, zolmitriptan, almotriptan, frovatriptan) are now available for the treatment of migraine. Each of the triptan class of drugs has similar pharmacologic properties but varies slightly in terms of clinical efficacy. Rizatriptan and almotriptan are the fastest acting and most efficacious of the triptans currently available in the United States. Sumatriptan and zolmitriptan have similar rates of efficacy as well as time to onset, whereas naratriptan and frovatriptan are the slowest acting and the least efficacious. Clinical efficacy appears to be related more to the tmax (time to peak plasma level) than to the potency, half-life, or bioavailability (Table 14-10). This observation is in keeping with a significant body of data indicating that faster-acting analgesics are more efficacious than slower-acting agents. Unfortunately, monotherapy with a selective oral 5-HT111 agonist does not result in rapid, consistent, and complete relief of migraine in all patients. Triptans are not effective in migraine with aura unless given after the aura is completed and the headache initiated. Side effects, although often mild and transient, occur in up to 89% of patients. Moreover, 5-HT1 agonists are contraindicated in individuals with a history of cardiovascular disease. Recurrence of headache is a major limitation of triptan use, and occurs at least occasionally in 40 to 78% of patients. Ergotamine preparations offer a nonselective means of stimulating 5-HT121 receptors. A nonnauseating dose of ergotamine should be sought since a dose that provokes nausea is too high and may intensify head pain. Except for a sublingual formulation of ergotamine (Ergomar), oral formulations of ergotamine also contain 100 mg caffeine (theoretically to enhance ergotamine absorption and possibly to add additional vasoconstrictor activity). The average oral ergotamine dose for a migraine attack is 2 mg. Since the clinical studies demonstrating the efficacy of ergotamine in migraine predated the clinical trial methodologies used with the triptans, it is difficult to assess the clinical efficacy of

MX-Biomedical Research Group / JD-MD Bioinformatics LabsJD-MD Medicine and Technology

44

Page 45: Enarm 8a Parte

ENARM by JD-MD

ergotamine versus the triptans. In general, ergotamine appears to have a much higher incidence of nausea than triptans, but less headache recurrence. Nasal: The fastest acting nonparenteral antimigraine therapies that can be self-administered include nasal formulations of dihydroergotamine (Migranal) or sumatriptan (Imitrex Nasal). The nasal sprays result in substantial blood levels within 30 to 60 min. However, the nasal formulations suffer from inconsistent dosing, poor taste, and variable efficacy. Although in theory the nasal sprays might provide faster and more effective relief of a migraine attack than oral formulations, their reported efficacy is only approximately 50 to 60%. Parenteral: Parenteral administration of drugs such as dihydroergotamine (DHE-45 Injectable) and sumatriptan (Imitrex SC) is approved by the FDA13 for the rapid relief of a migraine attack. Peak plasma levels of dihydroergotamine are achieved 3 min after intravenous dosing, 30 min after intramuscular dosing and 45 min after subcutaneous dosing. If an attack has not already peaked, subcutaneous or intramuscular administration of 1 mg dihydroergotamine suffices for about 80 to 90% of patients. Sumatriptan, 6 mg subcutaneously, is effective in approximately 70 to 80% of patients. DOPAMINE ANTAGONISTSOral: Oral dopamine antagonists should be considered as adjunctive therapy in migraine. Drug absorption is impaired during migrainous attacks because of reduced gastrointestinal motility. Delayed absorption occurs in the absence of nausea and is related to the severity of the attack and not its duration. Therefore, when oral NSAIDs8 and/or triptan agents fail, the addition of a dopamine antagonist such as metoclopramide, 10 mg, should be considered to enhance gastric absorption. In addition, dopamine antagonists decrease nausea/vomiting and restore normal gastric motility. Parenteral: Parenteral dopamine antagonists (e.g., chlorpromazine, prochlorperazine, metoclopramide) can also provide significant acute relief of migraine; they can be used in combination with parenteral 5-HT141 agonists. A common intravenous protocol used for the treatment of severe migraine is the administration over 2 min of a mixture of 5 mg of prochlorperazine and 0.5 mg of dihydroergotamine.

787.- Una de las complicaciones hematológicas mas frecuentes de la mononucleosis infecciosa es la anemia:

a) Hemolitica autoinmune b) Ferropenicac) Perniciosad) Megaloblásticae) Drepanocitica

Most cases of IM are self-limited. Deaths are very rare and most often are due to central nervous system (CNS) complications, splenic rupture, upper airway obstruction, or bacterial superinfection. When CNS complications develop, they usually do so during the first 2 weeks of EBV infection; in some patients, especially children, they are the only clinical manifestations of IM2. Heterophile antibodies and atypical lymphocytes may be absent. Meningitis and encephalitis are the most common neurologic abnormalities, and patients may present with headache, meningismus, or cerebellar ataxia; acute hemiplegia and psychosis have also been described. The cerebrospinal fluid (CSF) contains mainly lymphocytes, with occasional atypical lymphocytes. Most cases resolve without neurologic sequelae. Acute EBV infection has also been associated with cranial nerve palsies (especially ones involving cranial nerve VII), Guillain-Barre syndrome, acute transverse myelitis, and peripheral neuritis. Autoimmune hemolytic anemia occurs in ~2% of cases during the first 2 weeks. In most cases the anemia is Coombs'-test positive, with cold agglutinins directed against the red blood cell antigen. Most patients with hemolysis have mild anemia that lasts for 1 or 2 months, but some patients have severe disease with hemoglobinuria and jaundice. Nonspecific antibody responses may also include rheumatoid factor, antinuclear antibodies, anti-smooth muscle antibodies, antiplatelet antibodies, and cryoglobulins. IM2 has been associated with red-cell aplasia, severe granulocytopenia, thrombocytopenia, pancytopenia, and hemophagocytic syndrome. The spleen ruptures in fewer than 0.5% of cases. Splenic rupture is more common among males than among females and may be manifest as abdominal pain, referred shoulder pain, or hemodynamic compromise. Hypertrophy of lymphoid tissue in the tonsils or adenoids can result in upper airway obstruction, as can inflammation and edema of the epiglottis, pharynx, or uvula. About 10% of patients with IM develop streptococcal pharyngitis after their initial sore throat resolves. Other rare complications associated with acute EBV infection include hepatitis (which can be fulminant), myocarditis or pericarditis with electrocardiographic changes, pneumonia with pleural effusion, interstitial nephritis, genital ulcerations, and vasculitis.

788.- Caso clínico seriado. Un lactante de 3 meses presenta piel fría, estreñimiento, ictericia y edema en las extremidades. Además de corroborar los datos anteriores, en la exploración física se encuentra fontanela anterior amplia, llanto ronco y hernia umbilical. Primer enunciado. El diagnostico más probable es:

Hepatitis neonatal

MX-Biomedical Research Group / JD-MD Bioinformatics LabsJD-MD Medicine and Technology

45

Page 46: Enarm 8a Parte

ENARM by JD-MD

Trisomia 21 Síndrome nefrótico del primer año de vida Hipotiroidismo congénito Toxoplasmosis congénita

789.- Segundo enunciado. Una vez hecho el diagnostico el siguiente paso consiste en:

Observar una conducta expectante y esperar la resolución espontánea del problema Administrar esteroides Inciar el tratamiento sustitutivo hormonal tiroideo Administrar pirimetamina Realizar la estimulación temprana y prevenir las complicaciones

790.- Tercer enunciado. El tratamiento inadecuado de este lactante puede dar lugar a la aparición de:

Insuficiencia renal crónica Desarrollo de hepatoblastoma Microcefalia y epilepsia Leucemia Retraso psicomotor

Most newborns with congenital hypothyroidism appear normal at birth and gain weight normally, even if not receiving treatment, for the first 3–4 months of life. Because congenital hypothyroidism must be treated as early as possible to avoid intellectual impairment, the diagnosis should be based on the newborn screening test and not on abnormal physical findings. Cabbage, soybeans, aminosalicylic acid, thiourea derivatives, resorcinol, phenylbutazone, cobalt, and iodides have been reported to cause goiter and hypothyroidism during pregnancy. Because many of these agents cross the placenta freely, they should be used with great caution during pregnancy. Even with congenital absence of the thyroid gland, the first finding may not appear for several days or weeks. Consequently, any suspected abnormality in the results of the newborn screen should be investigated promptly. Findings include physical and mental sluggishness; pale, gray, cool, or mottled skin; nonpitting myxedema; constipation; large tongue; poor muscle tone giving rise to a protuberant abdomen, umbilical hernia, and lumbar lordosis; hypothermia; bradycardia; diminished sweating (variable); decreased pulse pressure; hoarse voice or cry; delayed transient deafness; and a slow relaxation component of deep tendon reflexes (best appreciated in the ankles). Nasal obstruction and discharge and persistent jaundice may be present in the neonatal period. The skin may be dry, thick, scaly, and coarse, with a yellowish tinge due to excessive deposition of carotene. The hair is dry, coarse, and brittle (variable) and may be excessive. Lateral thinning of the eyebrows may occur. The axillary and supraclavicular fat pads may be prominent in infants. Muscular hypertrophy (Kocher-Debré-Sémélaigne syndrome) is an unusual association with congenital hypothyroidism. Growth changes include short stature; infantile skeletal proportions with relatively short extremities; infantile naso-orbital configuration (bridge of nose flat, broad, and underdeveloped; eyes seem to be widely spaced); delayed epiphysial development; delayed closure of fontanelles; and retarded dental eruption. Treatment of acquired hypothyroidism may not result in the predicted final adult target height. Menometrorrhagia may occur in older girls, and galactorrhea resulting from the stimulation of prolactin secretion or elevated thyrotropin-releasing hormone (TRH) has been reported. In hypothyroidism resulting from enzymatic defects, ingestion of goitrogens, or chronic lymphocytic thyroiditis, the thyroid gland may be enlarged. Thyroid enlargement in children is usually symmetric, and the gland is moderately firm and without nodularity. In chronic lymphocytic thyroiditis, however, the thyroid frequently has a cobblestone surface; size and shape are readily apparent on inspection in children. Slowing of mental responsiveness and retardation of development of the brain may occur in neonates and infants, and a coincidental congenital malformation of the brain is present in some patients. Levothyroxine is the drug of choice in a dosage of 75–100 µg/m2/d as a single dose. In newborns and infants, the dose is 10–12 µg/kg. The hypothyroid patient may be very responsive to thyroid and may be sensitive to slight excesses of thyroid hormone. A dose of 0.0375 µg (37.5 µg) of levothyroxine is often recommended initially. Serum FT4 concentrations should be used to monitor the adequacy of therapy initially, because the elevated TSH may not fall into the normal range for several days to weeks. Subsequently, elevations of serum TSH are sensitive early indicators of the need for increased medication (or increased compliance). In the treatment of neonatal goiter with or without hypothyroidism resulting from drugs and goitrogens taken by the pregnant woman, temporary use of levothyroxine may be helpful in decreasing the size of the goiter.

791.- Un lactante de un año ocho meses presenta en forma súbita sangrado fresco por el recto, que le produce taquicardia, hipotensión y cifras bajas de hematocrito. El diagnostico más probable es:

Invaginación intestinal Volvulus intestinal

MX-Biomedical Research Group / JD-MD Bioinformatics LabsJD-MD Medicine and Technology

46

Page 47: Enarm 8a Parte

ENARM by JD-MD

Divertículo de Meckel Colitis pseudomembranosa Pólipos en el colon

El cuadro de invaginación intestinal debido a que un segmento intestinal se introduce en otro inmediatamente distal a el, es la causa más frecuente de obstrucción intestinal entre los 3 y los 6 años de edad. Su clínica tiene dos fases, una inicial caracterizada por la aparición de dolor abdominal brusco de tipo cólico, crisis de llanto, encogimiento. Esta fase se sigue de otra en la que el niño se encuentra cada vez más aletargado, más débil y somnoliento. En las primeras fases es frecuente que el cuadro se acompañe de vómitos y eliminación de heces, que hasta en el 60% de los casos presentan sangre roja fresca y moco (“heces en jalea de grosella”). Recuerda la imagen en rosquilla diagnóstica característica de este cuadro en la ecografía. En cuanto al tratamiento, si el cuadro no esta muy evolucionado se puede intentar una reducción hidrostática con enemas y control ecográfico, pero si ya hay signos de perforación intestinal, shock, neumatosis o distensión abdominal es de elección la corrección quirúrgica.

792.- un lactante de 8 meses comienza en forma abrupta un cuadro de vomito, diarrea acuosa con moco, fiebre, dolor abdominal y deshidratación moderada. A la exploración física se encuentra irritable, con eritema faringeo y abdomen timpanico. El diagnóstico más probable es:

a) Colera b) Infección por rotavirus c) Amibiasis d) Giardiasis e) Salmonelosis

La diarrea por rotavirus es la causa más frecuente de diarrea infantil, pudiendo llegar a ser un cuadro grave causando deshidración y necesidad de atención médica de urgencia. El mecanismo de transmisión del rotavirus principalmente es a través del contacto oral-fecal. Los síntomas de éste cuadro pueden incluir fiebre, nauseas y vómitos, dolor abdominal diarrea acuosa, deshidratación (especialmente importante en lactantes)… Para el diagnóstico, además de la clínica y la exploración física puede realizarse un cultivo de heces para la detección del virus. En cuanto al tratamiento, no hay ninguno específico, por lo que éste será de soporte y sintomático.

793.- La vacunación oral contra la poliomielitis esta contraindicada cuando existe:

a) Alimentacion al seno materno b) Diarrea c) Ingestion de antimicrobianos d) Fiebre e) Inmunodeficiencia primaria

Las contraindicaciones absolutas de la vacuna de la polio oral (tipo Sabin) incluyen las inmunodeficiencias humorales y celulares, incluida la infección por el VIH o contacto con personas inmunodeprimidas, por el riesgo de diseminación fecal-oral. Debe evitarse también esta vacuna en caso de embarazo, enfermedad digestiva aguda o en pacientes hospitalizados. Uno de sus efectos secundarios más graves es la parálisis flácida, mientras que la vacuna de la polio intramuscular (tipo Salk) tiene efectos secundarios mínimos y de poca importancia.

794.- El tratamiento de elección para la tiña de la cabeza en los pacientes pediátricos es:

a) La nistatina b) El ketoconazol c) El fluconazol d) La griseofulvina e) El clotrimazol

Las dermatofitosis o tiñas son infecciones micóticas que afectan a la piel y estructuras queratinizadas como pelos y uñas. La tiña del cuero cabelludo (también denominada Tinea capitis o Tinea tonsurans) es un tipo de tipo de tiña no inflamatoria son propias de la infancia cursando con placas alopécicas con pelos rotos y descamación. Por lo general, al llegar la pubertad suelen curar espontáneamente sin dejar cicatriz. El tratamiento de elección de la tiña del cuero cabelludo en niños es la griseofulvina. La griseofulvina es de primera elección en el tratamiento de las tiñas por su bajo precio, pero nunca está indicada en el tratamiento de una candidiasis cutánea. Otros fármacos útiles en el tratamiento de las dermatofitosis son los imidazoles y la terbinafina.

MX-Biomedical Research Group / JD-MD Bioinformatics LabsJD-MD Medicine and Technology

47

Page 48: Enarm 8a Parte

ENARM by JD-MD

795.- En niños alérgicos a la penicilina, el tratamiento de elección para la prevención de la fiebre reumática es la:

a) Eritromicina b) Kanamicina c) Dicloxacilina d) Ampicilina e) Gentamicina

Para el tratamiento de la fiebre reumática requiere antibioterapia de inmediato. En adultos es de elección el tratamiento con 500 mg de penicilina V o bien bencilpenicilina benzatina IM en dosis única. En caso de alergia a betalactámicos, se puede administrar eritromicina. Posteriormente, está indicado hacer una profilaxis de nuevos episodios de fiebre reumática ya que puede haber recidivas, sobre todo en los casos con afectación cardiaca y en los cinco primeros años tras el episodio inicial. Esta se debe hacer con una inyección intramuscular de 1,2 millones de UI de penicilina G benzatina. En caso de alergia puede utilizarse eritromicina o sulfamidas. Esta profilaxis está indicada al menos hasta los 21 años en niños, y en adultos puede ser necesaria de por vida.

796.- El medicamento mas útil para tratar las manifestaciones inflamatorias de las articulaciones producidas por la fiebre reumática en los escolares es:

a) La indometacina b) El metamizol c) El pirazolam d) El acido acetilsalicilico e) El metocarbamol

El tratamiento de la fase aguda de la fiebre reumática requiere administrar tratamiento antibiótico de inmediato, siendo de elección la penicilina V o la bencilpenicilina benzatina. Además durante la fase aguda es importante el reposo hasta que desaparezcan las alteraciones analíticas. Para el tratamiento antiinflamatorio el fármaco más empleado es la aspirina, a dosis crecientes hasta que lleguen a aparecer efectos secundarios (tinnitus, cefalea, hiperpnea). El tratamiento con salicilatos hace desaparecer los signos inflamatorios de las articulaciones, que son sin duda la manifestación clínica más frecuente de la fiebre reumática, mientras que el tratamiento antiinflamatorios con glucocorticoides sólo se utiliza si con la aspirina no basta o si hay datos de carditis con insuficiencia cardiaca moderada-grave.

797.- El medicamento inhalado que se debe utilizar para el tratamiento de la laringotraqueobronquitis grave que se presenta en los lactantes es:

La adrenalina El salbutamol El cromoglicato de sodio El bromuro de ipratropio La epinefrina racemica

El tratamiento de la laringotraqueitis aguda debe incluir: 1) Ambiente húmedo y frío, 2) Oxigenoterapia, 3) Corticoides inhalados (budesonida) con efecto antiinflamatorio local, 3) Corticoides sistémicos, 4) Adrenalina inhalada con efecto antiinflamatorio por su acción vasoconstrictora. La adrenalina inhalada está indicada en casos moderados-graves y su acción dura 4-6 horas, tras las cuales reaparece la clínica, por lo que está indicada la observación durante ese período. En casos grave que no respondan al tratamiento puede ser necesario la intubación y la ventilación mecánica. Recuerda que no están indicados los atitusígenos, no antihistamínicos, ni sedantes, y sólo están indicados los antibióticos en caso de sobreinfección bacteriana.

798.- El antimicótico polienico de primer elección en infecciones profundas producidas por Candida albicans es:

La anfotericina BLa griseofulvina La nistatina El ketoconazol El fluconazol

MX-Biomedical Research Group / JD-MD Bioinformatics LabsJD-MD Medicine and Technology

48

Page 49: Enarm 8a Parte

ENARM by JD-MD

El fluconazol es un antifúngico del grupo de los azoles, que se puede utilizar por vía oral o intravenosa, siendo de elección en el tratamiento de las infecciones sistémicas por Cándida albicans, así como en el tratamiento de la meningitis por Cryptococo neoformans, ya que atraviesa muy bien barrera hematoencefálica. También es útil en el tratamiento de micosis profundas por cándida la Anfotericina B (funguicida poliénico), asociado o no a la 5-Fluorocitosina, por via IV, pero sólo es de primera elección en los casos graves debido a sus graves efectos tóxicos (hipertermia, nefrotoxicidad, síndrome febril).

799.- Una niña de once años ingirió hace 7 días alimentos enlatados de dudosa calidad y presenta un cuadro clínico compatible con el diagnostico de botulismo. La sustancia que explica la sintomatología de la paciente es una:

a) Toxina alfa b) Termotoxina c) Neurotoxina d) Enterotoxina e) Endotoxina

La neurotoxina producida por el Clostridium botulinum actúa en el sistema nervioso periférico, concretamente a nivel presináptico en la placa motora, inhibiendo la liberación de acetilcolina, impidiendo la contracción del músculo y dando lugar al cuadro de parálisis motora que caracteriza al botulismo. El cuadro típico de botulismo tiene su origen en la ingesta de la toxina preformada en los alimentos, sobre todo alimentos enlatados o conservas caseras. Recuerda además que existen ocho tipos de toxina botulínica, siendo la A la causante de la enfermedad más grave.

800.- El diurético que produce reducción de la producción de liquido cefalorraquídeo a nivel de los plexos coroides con decremento de la presión hidrostática cerebral es:

El furosemide La acetazolamida La espironolactona La tiazida El manitol

La acetazolamida es un diurético proximal que actúa inhibiendo la anhidrasa carbónica, de forma que impide la reabsorción de bicarbonato, tanto a nivel del túbulo contorneado proximal como a nivel de los procesos ciliares en el ojo, inhibiendo la formación de humor acuoso. También ejerce su acción a nivel del SNC, viéndose implicado en la formación del líquido cefalorraquídeo. Así, sus principales indicaciones son el glaucoma de ángulo abierto y la hipertensión intracraneal benigna. En esta última, la primera medida terapéutica es la retirada de los fármacos que puedan ser responsables del cuadro o el tratamiento de la enfermedad asociada. Después está indicada la restricción hidrosalina, el tratamiento diurético con acetazolamida y los corticoides. Si esto no fuese suficiente sería necesario el drenaje del líquido cefalorraquideo.

MX-Biomedical Research Group / JD-MD Bioinformatics LabsJD-MD Medicine and Technology

49